You are on page 1of 161

微分流形笔记

Grantsome

2020 年 2 月 9 日
2
目录

I 无关紧要的话 5

1 前言 7

2 学习的认知心理学 9

3 数学学习方法论 21

II 笔记 29

4 多重线性函数 31
4.1 对偶空间 . . . . . . . . . . . . . . . . . . . . . . . . . . . . . . . 31
4.2 张量代数 . . . . . . . . . . . . . . . . . . . . . . . . . . . . . . . 36
4.3 外代数 . . . . . . . . . . . . . . . . . . . . . . . . . . . . . . . . . 45
4.4 外形式空间 . . . . . . . . . . . . . . . . . . . . . . . . . . . . . . 53

5 微分流形初步 67
5.1 微分流形 . . . . . . . . . . . . . . . . . . . . . . . . . . . . . . . 67
5.2 流形的例子 . . . . . . . . . . . . . . . . . . . . . . . . . . . . . . 79
5.3 切向量 . . . . . . . . . . . . . . . . . . . . . . . . . . . . . . . . . 83
5.4 切映射 . . . . . . . . . . . . . . . . . . . . . . . . . . . . . . . . . 92

6 子流形理论 101
6.1 截断函数 . . . . . . . . . . . . . . . . . . . . . . . . . . . . . . . 101
6.2 光滑向量场 . . . . . . . . . . . . . . . . . . . . . . . . . . . . . . 106
6.3 子流形 . . . . . . . . . . . . . . . . . . . . . . . . . . . . . . . . . 115

3
目录 目录

6.4 Frobenius 定理 . . . . . . . . . . . . . . . . . . . . . . . . . . . . 121

7 流形上的微积分 131
7.1 光滑张量场 . . . . . . . . . . . . . . . . . . . . . . . . . . . . . . 131
7.2 外微分与外微分式 . . . . . . . . . . . . . . . . . . . . . . . . . . 137
7.3 外微分形式的积分 . . . . . . . . . . . . . . . . . . . . . . . . . . 147

8 后记 159

4
Part I

无关紧要的话

5
第1章 前言

本笔记是云南师范大学数学学院 2019 年秋季郭震教授所开研究生基础课


程“微分流形”的笔记, 笔记中大部分内容均来自于老师上课的讲述, 其中很大
部分笔记参考了刘泓斓小姐姐的笔记和同门师姐吴元芬的笔记以及同门同届马
江涛小哥哥的笔记, 并且也部分的参考了平锐小姐姐的笔记和张徐小姐姐的笔
记以及于秀君小姐姐的笔记, 在这里谨向她 (他) 们表示感谢.
笔记主要包括如下内容: 多重线性函数基础、微分流形初步、子流形理论、
流形上的微积分.
之所以想要整理一份 LATEX 排版, 起源于自己上课不记笔记的这种懒惰恶
习, 偶尔抄一抄笔记也只是用 A4 纸, 但 A4 又不方便保存, 容易丢失. 从而有了
做一份 PDF 的笔记方便自己需要查阅的时候在手机上翻看和复习的想法. 另一
方面, 希望对那些和我一样懒以及对微分流形有兴趣的同学有些帮助.
在本笔记中, 我设法用直观的角度去看待抽象的概念. 花了大量的笔墨从本
科的基础知识过渡到微分几何的一些基本思想方法和工具上 (比如从微积分的
思想研究几何过渡到微分几何, 流形这个概念的来龙去脉, 流形不过是曲面和曲
线的推广. 用鱼和与鱼鳞的比喻来帮助大家理解流形这个概念. 张量不过是线性
变换的推广, 张量分量坐标的变换公式不过是矩阵相似的推广, 从流形的外围空
间与内蕴的角度上去看待为什么切向量需要更为本质的定义, 从拓扑本身是研
究图形的科学, 连续是图形的基本性质引出了连续是依赖于邻域的, 而邻域的三
条性质推广便是拓扑结构的概念, 切映射不过是一个光滑映射在某一个点的线
性化, 截断函数是为了把局部的东西延拓成整体的东西, 切从只不过是切空间的
整体化, 目的在于整体的去看向量场, 所谓子流形是三维欧式空间中曲线曲面几
何学的推广, 浸入子流形的标准型定理为了让子流形有一个好的坐标表示, 从积
分曲线的存在性过渡到积分曲面的存在性问题, 为了以及高维的积分流形的存
在性问题而得到了著名的 F robenius 定理, 为了更好的判断一个量是否是张量
引出了函数线性性.), 旨在给初学者一个比较低的门槛.
另外, 笔记中附加了两篇博客, 分别是关于认知心理学和数学学习方法的文

7
CHAPTER 1. 前言

章, 这大多是在我考研完了之后的一点点观察, 希望对那些对学习数学有疑惑的
人的一点点帮助
由于笔记整理时间匆忙, 加之理解能力有限, 学识不足, 并且对 LATEX 排版
不熟悉, 故笔记必然有诸多纰漏之处, 希望各位多多批评指正. 欢迎各位读者朋
友就书中的内容与我讨论, 我的邮箱 1278603466@qq.com.

8
第2章 学习的认知心理学

本文摘自一篇博客 (blog), 因某些隐私原因不贴出原文地址. 主要是写的关


于如何学习的认知心理学研究, 有部分改动原文的内容.

考研二三事——悟道学习存心法

为什么那些平时看似很懒惰的同学却成绩优异?
为什么那些上课经常记笔记或者各种在书上各种荧光笔标记的同学成绩却
比不上那些根本不愿意记笔记的同学?同一个老师在同一个教室上着相同的课,
同学们做着相同的作业为何成绩却大相径庭?人们做了看上去相同的事情,为
何会结果不同?到底什么是有效学习?学习的核心是什么?有没有什么放任四
海之内的准则?我们应该怎么学习以更好的掌控自己的人生呢?
渴望挑战的刺激
让暴风雨来得更猛烈些吧!——高尔基《海燕》
我渴望有价值的对手! ——英雄联盟剑姬的台词
荧光笔、下划线、记笔记和反复阅读是最容易做的重复性事情,也是最没
有学习效果的事情。 在大部分时间里,我们都在用错误的方式学习着,那些在
书上写满了各种笔记和荧光笔标记的同学,笔记越多,留下给我们思考的时间
也就越少。并且你看着笔记会有一种熟练度错觉,以为自己已经掌握了这个知
识点。但实际上我们对自己的盲目乐观与自信加上不恰当的学习策略,导致了
我们最终的结局。
听上去很残酷?嗯,是的,我想说得是,学习越轻松,效果越不好,越等
于没学。只有测试和检索,才能正确的评估自己的理解层次和知识素养。很多
时候,学生们不知道自己的问题在哪儿,这才是最大的问题。
天性懒惰孕育了认知规律和心智模式,大脑逃避思考,喜欢幻想和 YY,
逃避挑战和困难。
学习中不可避免的挑战,重新 固记忆:练习从记忆中检索新知识或者新
技能是有效的学习工具,也是保持长久记忆的唯一武器。努力检索有助于人们

9
CHAPTER 2. 学习的认知心理学

获得更好的学习效果,产生更持久的记忆。值得一提的是,频繁的集中检索和
练习只能产生短期记忆,而间隔检索、穿插练习、组合练习、多样化练习能够
使知识储存得更加牢固。
思考与理解:学习不可避免的会遇到各种问题,解不出来的题、看不明白
的证明过程、思考很久也没有什么头绪问题。这时候首先要做得是把问题无解
视为一种常态。
学习是挑战天性的必修课。学习的必要难度原则: 你的大脑越是费尽力气
地挖出某项记忆,你对其再次学得的程度也就越深,因为提取能力和存储能力
都被增强了。相反的,你复习你刚刚学过的内容对你没有任何意义,那并不会
增加任何记忆强度。所以,我们平时重复看书、抄笔记等工作并没有带给大脑
任何挑战,没有引起大脑的思考和提取,长此以往我们慢慢的扼杀了自己的学
习和思考能力。变成了一个看上去在学习的“机器”。
Poincare 的马车:顿悟
沃拉斯《思考的艺术》和阿达玛《数学领域的发明心理学》中引用的那些
思考者的描述相当有趣。特别是 Poincare 的例子。
法国数学家亨利. 庞加莱 (Henri Poincare) 对自己的一段描述,很详尽地记
载了他在琢磨富克斯函数的特性的种种经历。“一个人若要专研某个难题,第
一次往往会一无所得,”庞加莱写下了他对自己的观察,“这人会或长或短的休
息一下,再坐下来专研那道难题,跟上次一样,半个小时过去了仍然毫无头绪,
可是突然之间,一个成形的想法就蓦然出现在脑海中。”
“……我可以说一句在如此的环境下发现了这个定理的证明,这个定理有
个很生疏的名字,恐怕我们大多数人都不熟悉它,但这一点无关紧要,对于心
理学家来说,重要的不是定理本身,而是发现这个定理的种种情形。”
“……这样,我们就说到富克斯函数了。起先,我对这种函数冥思苦想了
整整两个星期,企图证明它不存在,但这个想法被后来的事实证明是错误的,
我想要把这类函数表示成两个级数的商,思想是非常自然且有明确目标的,这
时我想起了类似于此的椭圆函数的情形。我就自己设想,如果这两个级数存在,
它们会有什么样的性质呢?寻此向前,我并没有遇到任何困难,我构造出了这
两个级数,并称之为富克斯。”
“就在此时,我离开了我所居住的地方卡昂,在矿业学院的资助下,开始
了地质考察的旅行生活。途中的许多事情使我忘记了我的数学工作。到达康斯
坦茨湖,我们要乘一辆马车到其他地方去,就在我把脚放到马车踏板的那一瞬
间,一个思想突然闪现在我脑海中,而在此之前,我还从来没有想到的,这个
思想就是,我可以定义富克斯函数的变换与非欧几何的变换是等价的。当时我
没有马上去考虑证明这个思想,因为当时我没有时间去考虑这件事,我继续和

10
CHAPTER 2. 学习的认知心理学

马车里的旅伴海阔天空的谈论着其他事情,然而我能感觉到这个思想是完全确
实的。在旅行结束之后我回到了我的居住地卡昂之后,为了能够问心无愧,我
还是抽空给出了这个思想的证明。”
“此后我就把注意力转移到于此有关的一些算术运算的问题上,但并没有
取得任何成功,并且看起来也不像与我以前的工作有什么联系,由于对自己的
失败感到厌烦,我去海边度过了几天,并且考虑了一些其他的事情,有一天早
上,我正在悬岩上散步,一种新思想和上次一样突然闪现而来,而且同样是简
洁而确实的一种猜想,这个思想就是不定三元二次型的算术变换和菲欧变换是
等价的。”首先最令人惊奇的是,这种”顿悟”的出现。
同样的话德国物理学家亥姆霍兹也说过,他被难题卡住而四处碰壁的时候,
一个新的念头如何冒出来的。”令人豁然开朗的念头往往意外来临,看似不费吹
灰之力,就跟灵感一样忽然出现”,他写道,“我以前的体会而言,在我的脑子
里面已经累了的时候,或是坐在办公室前正儿八经工作的时候,灵感是不会跑
出来的……可是要是在阳光灿烂的日子里沿着斜坡走向山林,却是他们最喜欢
跑出来的时候。”
这些描述在沃拉斯的眼里却并非如此:他能看出它们都有一种基础结果。
这些思考者首先是被一个问题卡住,然后放下来到处走走。这时,他已经穷尽
了脑子里面所有的思路却看不出任何门道。关键性的顿悟可能往往会在他放手
之后、在他并非专心思考的时候,忽然意外地出现。
第一个步骤叫做准备期。这一阶段可能以小时计、天计,甚至更久。是一
个人用来琢磨某个逻辑难题或者创新课题的时间。比如庞加莱,他花了 15 天
的时间想要证明富克斯函数不存在。“每天,我会让自己坐在办公桌前,花上一
两个小时,反复尝试各种不同的公式组合,却总是一无所获。”他写道。装阶段
不仅包括弄懂你要琢磨的难题,弄清楚你手上有什么线索、别人怎么提示你的,
还包括做去各种各样的尝试,直到用尽你头脑中所有的线索。换句话讲,你不
是止步不前,而是黔驴技穷了。于是,准备期到此结束。
第二个步骤叫做孵化期。这要从你把问题搁到一边去的时候开始算起。以
亥姆霍兹为例,他就是在放下忙了一上午的工作,出去顺着山林往上走,不再
想刚才的问题时出现的顿悟。沃拉斯还发现,有些人这一阶段出现在睡觉的时
候、吃饭的时候,甚至是和朋友聚会的时候。沃拉斯认为:大脑在离线状态下
还在围绕那个课题继续工作,不时加上一两个老早就装在大脑里面但一时没能
想到但可调用出来的想法。
第三个步骤叫做顿悟期。这就是你啊哈的那一刻。
第四个步骤叫做验证期。这一步骤是要复核并确认得到的结果是否真的行
得通。

11
CHAPTER 2. 学习的认知心理学

谈谈自己的一些体会。有一次, 记得是我们班一个女生 (杨秀) 为了参加数


学竞赛而问了我一些数学分析的比较困难的题目。当时我有两个题根本没有思
路做出来。持续了一周左右的时间我每天都会做那两个题,每天做一个小时左
右。无果遂放弃。直到后面 11 月的一天,我当时正在学校 15 栋前面的马路上
走着,忽然想到了用泰勒展开和三次方公式去化简那两个题目。然后我赶紧到
了图书馆,做了一下午加一晚上终于干掉了那两个题目。我现在都还记得那一
瞬间的感觉和情形。
还有一次, 是在学实变函数的时候遇到的一个问题,在大三上的时候,一
个关于鲁津定理、Riemann 可积与 Lesbuegue 可积的问题。当时困扰了我很久
很久,一直持续到下学期开学,记得那一天做数学分析做累了,然后又没有带
其他的书籍,感觉无聊的时候突然就有了一个想法:那是一个垃圾定理,偷梁
换柱改拓扑而得到一个诡异的结论。然后我便去图书馆看了看那个定理的证明
过程,看了好几本不同的教材,发现都是差不多的。和我之前的想法一模一样。
并且在夏道行的教材上看见了和我疑惑有点类似的一个注解。才明白那个地方
的缘由,不过虽然明白了那个地方的,但新的问题又冒出来了,新的问题至今
为止也没有得以解决。从那以后边对鲁津定理没有任何好感了。
考研的时候,记得在计算一个行列式思考了一两个小时无果之后出去上课,
由于二教距离四教还有一段距离,我在路上漫不经心的走着,突然就明白了怎
么做:加边法。然后到了教室我用这个思路做出来了那个题目。但考研时大多
数的思考都是失败的,我到现在还有印象的思考过的题目并且最终也是靠看答
案而明白的题目至少有十个左右。考研时一般没有什么思路出去吃个饭或者上
个厕所回来也许有新的思路,也许这个思路最终也是不行的。个人觉得因为考
研解决不了的问题会影响即将到来的考试,不敢轻易像平时的问题那样放在那
里一段时间,所以实在想不出来就索性看答案。但好像效果并不是特别好。
也是在考研的时候,我对学习方法和学习习惯开始有了自己的体会和看法。
最近一些时候, 一次我们班班长 (海宝) 问了我一个问题,当时我想明白了,
但是后面我又忘了,导致给他讲的时候没有说清楚细节,但思路是清楚的,细
节需要加工一下。当天晚上我回去洗澡的时候,突然又明白了那个细节的地方,
这次可不会再忘记了。
从云南师范复试回来,当时因为我抽到的题目比较简单,然后我很快就做
完了。但后面老师又给了我一个问题:E+AB 可逆,求证 E+BA 也逆。当时
我的第一想法是初等行变换然后两边同时取行列式。但我个人觉得太麻烦了,
复试的时候我也就说了一下思路,并没有去做。复试完了之后,我在思考有没
有什么简单的方法,好像没有什么想法。于是就放下了。一直到回学校了之后,
有一天晚上我的手机要没电了,正准备回寝室,刚刚伸出脚的时候,我突然就

12
CHAPTER 2. 学习的认知心理学

想到了 AB 与 BA 具有完全相同的非零特征值,(初等变换已经被我放弃了,我
准备是在做不动的时候再用这个放弃,但这个结论用打洞 (龙生龙,凤生凤,
华罗庚的弟子会打洞) 原理的一个推论是显然的) 于是我马上验证这个想法的正
确性,给出了如下证明:证明: 用反证法,因为 AB 与 BA 具有相同的非零特
征值,如果 E+ BA 的行列式等于 0,说明 E-BA 的行列式等于 0,这说明 -1
必然是 BA 的特征值,而 -1 是 BA 的非零特征值,这说明 -1 也是 AB 的特征
值,即 -E-AB 的行列式等于 0,从而 E+AB 的行列式也等于 0,这与 E+AB
可逆相矛盾,证毕
应当说,从复试结束以来,我自认为我想明白了好一些以前所不太理解的
内容,包括 Galois 理论、微分流形、代数拓扑里面的基础性内容。也许是比考
研之前更能静下心来好好理解与思考有关。
所以,先把难题放到一边不必为此感到恼怒或者垂头丧气,实在做不下去
的时候,休息一下。很多时候这种关键性的休息时刻能够有助于最终解决问题。
另外,值得一提的是,把问题无解视为一种常态,是至关重要的。不去因
为问题无解而各种发散自我评价和各种归因,对问题的解决有着一定的帮助。
Vinci 的啄木鸟:好奇
“一定要清楚啄木鸟的舌头结构是怎样的。为什么它一直在啄树啄地很响,
却没有引起任何不良反应 (比如脑震荡等)。”– 达芬奇
“一定要知道鱼为什么在水里 的速度看起来为什么比鸟儿飞得还要快。”
– 达芬奇
保持好奇心,仔细观察。所谓知知者不如好知者,好知者不如乐知者。
达芬奇就是达芬奇,为什么鱼儿看起来的速度比鸟儿还快?为什么啄木鸟
没有脑震荡?多么有意思的问题。
正是好奇,驱动着我们前进,不断逼近世界的真相。推动着科学的发展。
达芬奇还有一句很有意思的话:“谈话、走路、骑马一定要风度翩翩。”
费曼技巧
原理:对自己也好。对他人也好,把你学过的东西表述出来,这种简单的
做法并非是传统意义上的一种“自考”方式,它更是一种“学习”的方式,一
种更高效学习方式。这比你继续坐在那里盯着书本的效果肯定更好,而且这样
的练习还能消除“熟练度错觉”,让你能真正看清哪些地方你还不知道,哪些
地方你还有疑惑,哪些地方你已经忘记,立杆见影。
这也在一定程度上说明了为什么老师基本对各种概念理解的清清楚楚,而
学生学的时候总是迷迷糊糊。
渗滤:拥抱你的拖延症
创造性飞跃是如何诞生的。

13
CHAPTER 2. 学习的认知心理学

小说家约瑟夫. 海勒如何描述他那些奇思妙想的由来:“我必须独自一人。
公共汽车不错,遛遛狗也行,刷牙则更好了,尤其是思路以及无处可走的时候。
常常就是我在累极了、打算休息的时候,我的脑子缺格外清楚起来……这时常
常会钻出来一句名团需要琢磨的句子,或者是下一步构思的念头。在我最好的
思路都不是在我落笔的时候冒出来的。”
诗人豪斯曼也有类似的描述。他们给我们展现了这么一幅构图:创造性的
飞跃通常出现创作者沉浸于某个故事或者主题一段时间后,暂时放下之时。而
且常常以零碎的方式出现,既没有一定先后顺序,也没有什么重要程度的讲究。
可能是宏大的、结构严谨的想法,也有可能只是一个小小的改动.
所以我们用渗滤这个词来描述不同于之前的顿悟的短期啊哈。该词的本意
为:像煮咖啡那样,先将咖啡粉浸泡,再透过滤纸的点点滴滴。取其为浸泡过
滤渗透之意。
那些电视剧的编剧们,总是在剧情最为精彩的时候结束一集。让观众们看
完这一集就期待下一集。这样的中断会在观众心理留下一个悬念。但这只是阴
的一面,阳的一面呢?假如我们正在完成一项耗时耗心血的工作?那这项工作
会不会一直在我们脑子中打转。
人一旦被某件事情吸引,便自然会生出一种动力来想逃一口气做完那件事,
而这种动力会随着事情临近尾声而变得越发强烈。“想要完成某件事情的欲望,
在刚开始的时候还算不上是欲望,但是到了后来,当你投入到忘我境界之中时,
那就变成了货真价实的欲望。”
渗滤的第一要素是打断 一旦某个目标被激活,它便能盖过其他一切,调动
起我们的觉知、思维、甚至是心态,去关注身边的一切。这便是渗滤的第二要
素。渗滤的第三要素是有意识的反思
机会总是留给那些去找呗感知力调动起来的大脑。
一次访谈中,主持人问擅长微小说的小说家韦尔蒂,她在小说中的那些对
话都是怎样得来的。韦尔蒂回答说:
“一旦你投入一篇故事的创作中,便会觉得
所见所闻的一切都可以写下笔。比如说,你在市区搭乘公交车时听到的对话,
就完全可以用到你今天写的那段故事里。随便走到哪儿,都能遇到可写入小说
的情节。也许这样该算是对准了频道了吧,或者说是你的耳洞变成了磁石,你
需要的事情就都被你吸引了过来。”
她在这里还有一件事情没有说到:那些从公交车上听来的对话如今让她笔
下的某个角色活了下来,帮她把故事铺展了开去。那些被“吸引”到我们耳朵
里面的对话不但可以归入到我们脑子里面的旁人对话分裂手册里面去,更能眼
神我们对一篇故事的思考。

14
CHAPTER 2. 学习的认知心理学

所谓渗滤,其关键就在于一直保持这心神的警觉,不断的想办法调动我们
的头脑来,关注手中那一份未完成的项目相关联的一切,随时采集各种对外界
的感知,并感悟内心的反思。
所谓数学家,其实是说的这么一种人,他走到哪里脑子里面都惦记着一个
概念,直到有一天他回过头来,发现自己已经对那个概念已经很熟了。
交替:谢谢你的厌烦
每次我学习或者看书,同一本书看不了多久就感到厌烦了。
但正是这种需要变化,需要交替和新鲜感的本能,这种厌烦的感觉恰恰有
着非常积极的影响。
学习时把不同的物件、技巧、概念等穿插在一起来练习,经过一段时间的
积累之后,我们不但能够更加清楚地到了解它们之间的联系,而且对它们自身
也有了更好的掌握。
交替学习最为关键的地方在于,你把新的科目和已经学过但有一段时间没
有接触过的东西混在一起。而这比长时间反复联系一个技巧要好得多。
怀疑一切的力量
对于科学而言,没有什么比质疑 (怀疑) 更为重要。但质疑需要强大的自
信!未经思考的相信本质与迷信无异,质疑和思考才是科学的本质。
也许这是一个时代,在应试教育和听话主义下必然的结果。但唯有自信,
才是解药。
每一个作者眼中的事实,都有可能是扭曲的。我的文章,恐怕亦有许多偏
见。有位数学家曾说:那些你的前辈赠予你的知识,如果你想要获取他们,你
还得重新探索一遍。
基于去探索作者的描述,去体会背后所表达的事实,而不是去相信,相信
是没有意义的,相反,如果去质疑,则拥有着更多的可能,收获更为深刻的理
解和更为 锐的洞察力。
不要去盲目的相信和接受,无论任何知识,既然它是前人的脑力劳动成果。
既然前人能够做出来,我们就应当有信心去超越前人。
在整个高中和本科时代,学生都有这种倾向: 遇到问题和困难总是寄希望
与别人来告诉自己如何往下进行,始终陷入一种被动的、踌躇不决的思路框架
之中,害怕标新立异的心理状态,没有什么明确的见地。其结果就是,学生们
忘记了最容易请教的人:自己。学生们总是只顾着到处看别人,到处找那些更
好、更聪明的观点,我不认为自己能想出来什么、写出来什么。对自己往往没
有任何信心。伊利诺伊州的一名博士生在给学生们授课的时候也注意到了这个
情况,她的学生所学的文章也给人这种畏畏缩缩、毕恭毕敬的感觉。龙达. 代
夫利当时在这个州的大学攻读英语博士学位,同时还负责本科生的写作课教学,

15
CHAPTER 2. 学习的认知心理学

课程主题是如何以权威性的资料为依据,在学术期刊上发表具有说服力的文章。
可每次到了期末,她却倍感失望,在那一学期里,她要求学生写六篇文章,每
篇文章都得以社会、政治、文化中的争议性主题为论点,篇幅 3-5 页。她希望
看到旁征博引而论据犀利的文章,可是按照她的描述,收回来的文章无非是前
任发表过的学术文章的剪贴编辑而已。最让她难过的是,期末的作品和刚开课
时候的作品,没有任何进步。
她决定改善这种做法,第二学期一开始,她放弃了这种小儿多动症般的忙
碌教学法。她只要求学生在期末的时候交一篇涵盖单一主题的文章。但在学期
的进程中,作为如何做调研的训练,他们需要另外完成 5 篇“练习作业”,目
标均锁定在做调研/做科研的体验上面。
比如,一篇练习描述如何采访一名专家,另一篇练习如何确定核心词汇,
并在辩论和讨论中加以应用。第三篇则是针对自己所选主题面临的有争议的各
个思想流派,他们应该如何加以应答。不仅如此,她还要求学生阿奇整个学习
调研的过程中做笔记,记录下他们面对调研中的各种人物或者资料时,自己心
中有什么感受,比如是否认为 篇文章的论述合情合理?是否认同 重中的主
要观点?某位名人大流的文章里面有没有什么错误?
学生们的反馈如下: 随着时光的推移,我做的调研越来越多,很多信息都
已经印在了我的脑海里,如今,我已经开始质疑某些作者宣称的所谓事实,我
发现现在不见得会认同发表在专业学术杂志上的所有内容。
还有一位同学说:我对自己经手的资料有着更为清晰而彻底的了解,已经
能够针对期刊上面的文章提出一些自己的问题了。
还有一位同学嘲笑的说:发表在这篇颇有名望的杂志上的文章,简直就是
给这个领域的初学者的。我只会把这篇文章推荐给对这一领域毫无了解的人去
看。
也就是说:她的学生不再只会借用别人的观点了,他们已经知道努力探求
自己的观点了。
也就是说:她让学生们把自己调研中的感受都写下来,写下他们对所用的
调研材料、杂志文章、人物访谈等的个人看法。学生们的能力便随着他们慢慢
积攒起来的知识得到了提高。
这便是怀疑的力量,也是渗滤中反思那一步最为重要的东西。
为理解之终极而思考。
真实环境的重要性
特别强调的是,针对于考试这种事情,最好定时模拟考试几次,只有真实
的环境,才能考出你最为真实存在的问题,而不是在自己的思维中打转。

16
CHAPTER 2. 学习的认知心理学

模拟真实,而不是在自己大脑想象出来的真实场景中。有个案例:在警察
学院的大学生们每次练习缴枪是两个大学生相互练习,缴掉对方的枪之后马上
还给对方接着练习。但这也导致了这些警察第一次在出去办案缴枪的时候直接
把缴过来的枪还给了歹徒。这就是真实环境的重要性,不过好在歹徒也没有反
应过来这是什么操作,然后让警察再一次夺回了枪。
整体性学习
所谓的整体性学习策略, 是指
学习的常见误区及其死循环
人的思维格局,决定了人们做事的时候质的不同。特别是一些常见的思维
误区。
思维误区
1、害怕挑战,呆在自己的舒适区。
2、以为自己时间很多,就从头开始而不是抓住重点。
3、对未知的恐惧很对曾经不如意的愧疚一直在情绪和潜意识中飘来飘去。
4、认为下次成功的机会一定比上次高。
5、相信别人能够解决自己的问题而不是自己。
6、认为学习某个新东西一定得专门找个时间。事实上,当下,把一个问题
一个问题的弄清楚,你就已经踏上了学习这条路了。只有脑中有问题,脚所能
到达的每一寸土地都是思考的天堂。纯粹的探索着自己未知的东西,这句话比
学习更为本质。做一个当下的智者,而不是未来的信徒。
7、大脑习惯性的强迫性重复某些幻想的场景,重复着“单调如同死水”的
生命。而不去关注当下自己能够做的能够改变的东西。拒绝做一个当下的智者。
伪勤奋者的自白
每天假装学习,实际上什么困难都选择逃避,没有任何独立思考的能力。
假装很刻苦,实际从来不思考。这就是中学时期常见的那种,学习非常刻苦但
成绩也不怎么样的那种学生。
1、学习的时候没有挑战:这种人太常见了,因为这种人是大部分人的“实
干家”。
劳苦功不高:这一类人的典型代表是起早贪黑的考研人,每天起早贪黑的
做事情,“忙碌”到自己没有任何时间去思考、沉淀。每次遇到问题都几乎放
弃,要么问别人要么直接放弃,加之社交圈和信息的闭塞,他们最终的结果也
会受这些“看不见的大手”的影响。
当然这种人还有一些特点:� 上课喜欢记笔记而不是思考、理解。� 书看了
一遍又一遍实际上对书上的内容还是理解不到位。� 喜欢问别人问题,而没有
意识到“我”是最强大的智者。� “热衷”看书/学习而不是做题、提问、思考、

17
CHAPTER 2. 学习的认知心理学

创造等需要动脑的活,或者做题只做自己会的,自己不会的题目也不去想办法
弄懂。� 面对问题没有自信。(喜欢看答案,但实际上看了答案过以段时间回来
做题还是不会)。� 不会反思和总结。
与之相反的有一类人,看上去漫不经心,讨厌记笔记,性格偏内向。对所
有偏理解和思考性质的东西有着执著的追求,这种人走路看上都心不在焉,而
且行动非常缓慢,堪比乌龟。总是喜欢自己一个人散步和思考问题,不惧怕任
何问题,视问题无解为常态而不放弃问题,学习的时候懂得适度穿插做题、回
忆、思考、举例等 (常见理工科钢铁直男形象?)。
2、没有模拟真实环境:如同备考,在重大考试之前,可以模拟几次真实环
境,尤其是数学。不然你不会知道你要怎么安排时间,如何检查计算结果的正
确性,如何安排证明的步骤,如何使用草稿纸等,如何在考场上调整心态等,
在什么情况下使用答题技巧。这也是非常重要的东西,上面也提到过模拟真实
环境的重要性。
学习的心法
幸福的家庭都是相似的,不幸的家庭各有各的不同。– 列夫. 托尔斯泰《安
娜. 卡列尼娜》
说完理论来分享一些比较好的学习习惯。
1、每天开始学习之前拿一张 A4 纸,凭借理解和记忆默写昨天学过的内
容。(对数学而言,特别是重要定理及其证明,例子,应用等)整理自己还存有
疑惑和没有理解的地方。完全清楚和理解后再开始新的一天的学习。解释一下
其中的原理:大脑要从记忆中提取学过的定理、定义、公式、推论等任何都东
西,所付出的努力远比直接重读一遍或者重学一遍要好得多,而这份额外的努
力则加强了这些记忆的储存能力与提取能力。这样做之所以能够对知识的掌握
更加牢固,这是因为我们简单的温习了一遍,是自己把它们从脑海中“提取”
了出来。值得一提的是,错误而失败的记忆、理解提取比正确的提取更令人印
象深刻。
2、每个学科最多持续学习两个小时,交替不同的学科学习,将听课、笔
记、看书、做题、回忆有机结合起来。
3、重要学科在期末之前写一本笔记 (不是抄一本笔记,而是回忆和复述加
注解一本笔记)。
4、读完定理之后自己尝试着去证明而不是直接去看书上的证明。
不管谁写了多少心法,写了多少理论。可以肯定的是,这些只能治标而不
能治本。或者说,虽然有了那些传统的错误学习方法的适度纠正。但这种纠正
会制造下一个“传统学习方法不当”的错误,无论思维和解决方式现在看来多
么好,跌落是必然的。

18
CHAPTER 2. 学习的认知心理学

任何解决问题的方法,都会跌落成下一个产生问题的原因和背景。这个方
法会变成一个问题,直到我们已经找到下一个解决方法,如此循环。
另外,知识量不等于创造力。学习得再好,懂得东西再多。都要记住,我
们并没有比前人更富有洞察力去开创一个学科,学习在某种意义上是平凡的。
而创造是非平凡的。
参考书籍:
《辛雷学习方法》
《认知天性》
《如何学习》本尼迪克特. 凯里
《改变:问题形成和解决的原则》
《数学领域的发明心理学》阿达玛
《列奥纳多. 达芬奇传》
《棋与人生》
《思考的艺术》格雷厄姆. 沃拉斯

19
CHAPTER 2. 学习的认知心理学

20
第 3 章 数学学习方法论

本文也摘自一篇 (blog),因隐私原因不贴出原文地址。主要是写得关于数
学学习方法论,大多都是各大数学家和教授们的观点,有部分的改动原文的内
容。

数理乾坤自分明——如何科学的学习和理解数学

学习、理解和思考数学的精髓,探索之途,早已蕴藏于所有生命的本身之
中。
作为数学系毕业的学生,是时候写一篇关于专业学习的东西了,此文与考
研二三事 – 悟道学习存心法互为姊妹篇
如何学习科学的学习数学,或许是一个过于宏大的话题。对于我这种数学
基础本来也不怎么好的人而言,似乎有点吃不消了。即使对于大数学家而言,
这也不太像是一个简单的问题。也许答案本身并不重要,更重要的是,总得有
一些自己的观点和理解。这才像学数学的人应有的态度。
阅读本文之前请阅读上面提到的本文的姊妹篇,以便在讨论学习数学学习
之前,对学习方法有着一定的认识。
求木之长者,必固其根本。欲流之远者,必浚其泉源. ——唐. 魏征
我们必须知道,我们终将知道。——希尔伯特
数学天敌
个人觉得,如果是纯粹学数学,感到难是因为前提基础不够,而自己太希
望快速学好了,但是数学学习是最忌讳浮躁的。欲速则不达,见小利则大事不
成。
学而不思则罔,思而不学则殆。学习数学知识和做数学题之间一定要有一
个平衡。同样的,在理解数学思想和计算功底之间也要有一个平衡。
误区一:沉迷于某一个知识层次的解题技巧,而没有深入学习更高层次的
知识,技巧越玩越花,但是思考问题的角度、处理问题的方式却没有实质性的
进步。

21
CHAPTER 3. 数学学习方法论

误区一的对偶:某一个层次的知识还没有足够的训练,就去追求更高层次
的知识,看似知道了很多,但是什么问题都解决不了。
误区二:一方面要理解抽象的概念,另一方面还要会算、举例、应用。抽
象的概念是别人告诉你的,而计算才是你得花时间去“参与”的。
大家之言
摘录小平邦彦的一段话:我一般只看定理,努力去理解定理,然后自己独
立思考数学证明。不过,大多数情况下都百思不得其解,最终只能参考书上的
证明。也许有人不解。为何要如此左思右想?直接读到最后一页不就好了吗?
话虽如此,不过这样会存在一个问题。在数学书中,读完以后到底能否彻底解
决彻底理解,我对此持有怀疑态度。理解数学书是一种怎样的状态呢?只要一
步步验证已确认证明过程无误就行了吗?在阅读数学书籍的时候,我发现即使
确认了证明的求证过程,之前不理解的定理仍然不得其意。虽然证明过程正确,
但总感觉整体印象模糊不清。但我们为什么能够清晰的理解 2+2=4 呢?是因
为自己是从感觉上把我了这一数学事实,而不是通过论证。定理的理解同样如
此,应该从感觉上把握定理所要表述的数学事实,而不是通过论证。尝试摸索
定理的证明过程,是一种从感觉上把握定理的方法,而并非为了检验证明过程
的正确性。想要更好的理解定理,仅仅读一遍定理是不够的。将定理运用于各
种问题之中才是有效的方法。花时间详细分析定理所要表述的数学事实的结构
是正确的方向。
数学是一门具有高度技术性的学问,如果出现理解证明过程却无法理解定
理内容的情况,说明把握数学事实的感觉还不够发达。数学如同雕塑,普通的
木头里面没有埋藏着定理。不过,仅仅从外表观察,并看不出里面究竟埋着什
么,所以只好尝试雕刻看看,数学中的雕刻就是繁琐的计算和查阅文献,绝不
是什么简单的事情,往往会竹篮打水一场空。想要理解数学,只能一步一步遵
循证明过程,去体会它们表述的数学事实。我们之所以只有通过认真阅读、验
证证明过程才能理解定理,是因为证明过程不仅仅是验证手段,其背后隐藏着
高于验证的东西,也许我们能否清晰明了的理解数学在很大程度上取决于能否
把握这个东西。
伍鸿熙先生有一段话:一、你们会了解到书内的定义和定理既是人为的,
又同时是合理的。也许你们人为一本书要写得高深莫测,才能显出作者的学问
渊博。但是我却希望你们会觉得书中的一切,不但是理所当然,而且是容易得
到的,只要自己花一点功夫就可以自己做出来。要做到这一点,除了一般的”
定义、定理、证明” 以外,我设法多加一些术语来说清楚每一个概念的来龙去
脉和直观意义。另一方面我也要指出,书内的概念和结果之所以被认为是基本
性的,并不是因为某某权威说过是如此如此,而是因为经过时间的考验后,发

22
CHAPTER 3. 数学学习方法论

现的确如此。就是说,从经验的总结,我们现在知道这些概念和定理是有用的
和必须的。所以一个初学者应该致力于探求为什么所学的是有用的和必须的。
否则不能对所学的内容有一个全面的了解。这种治学的态度,其实不单适用于
数学上,而是适用于一切学问的领域上,包括社会科学在内。
其次,我希望你们能够把握每个定理、每个证明、每个概念的要点。一本
好的数学书应该不同于一本字典,在后者中每一个字都占有同样的地位。但是
如果说一本数学书中每一个定理、每个证明、每个概念的要点都相同的重要,
未免就太荒诞无稽了。比方说,弧长的二次变分公式只是一个一般性技巧性的
结果,要点在于弄清楚如何将他应用于具体的情况,而不再探讨这个公式本身
的深度和公式的推导,所以不应该只给出公式而不给出应用,更不应该把这个
公司当作主要定理之一。又比如,Synge 定理的证明看上去是相当累赘的。但
是如果从一个很直观的事实作出发点,就是” 任何一个非连通的紧致黎曼流形
上必存在一个非同伦于零的最短闭曲线”,则其他一切都是顺理成章的、所以
我希望你们能够培养一个习惯,总要问,这本书的要点何在?这一章的要点何
在?这个证明的要点何在?能找到这些所有问题的答案,才能说有真正的了解。
最后,我希望你们能够以完全直观的眼光去了解本书的内容,所有的数学
书都是充满了技术性的术语的,因为为了要表达清楚,作者毫无选择的余地。
但是一个数学工作者的思考,大部分时候都是靠直观 (甚至是过分简化的直观)
的想法来推进的。在几何学方向这一点尤其重要。所以书内这一类直观的讨论,
比其他的数学课本会多一点。也许你们还迷信所谓的数学严格性的证明,以为
数学上最重要的是每一步推论的正确性,这个论点,相当于说鲁迅文章的好处
是每一句都很通顺。希望你们不会犯这个” 见小不见大” 的毛病。
“搞数学不需要太聪明,中等天分就行,主要是毅力和专研。”– 张广厚
“我早上醒来,想得第一件事就是数学,我的生活就是数学,终身不倦的
追求就是数学。十年如一日,没有懈怠过,现在我每天至少花七八个小时思考
数学。”– 陈省身
“每天思考七八个小时的数学,而不是呆在办公室学习七八个小时。事实
上,没有个十年八年的时间,是很难成为一个合格的数学工作者。”– 陈省身
每日思考八小时,发奋研究十余载。
个人之见
当然,以上是大师们的看法。下面谈谈我自己的看法。
我主要讨论听课和看书这两个角度来谈数学。
首先坦然看书,大概是:直观理解化、合理启发化、要点条理化。心法是:
一题不做,何以做数学?一例不知,何以知学问?一字不疑,何以疑定理?一
句不思,何以思推广?一惑不解,何以解猜想?一理不悟,何以悟证明?一法

23
CHAPTER 3. 数学学习方法论

不创,何以创学科?
大体来讲,无外乎以下几点:要知道一个定义的 motivation,也就是为什
么要引进这个定义?最初引进这个定义的动机和目的是什么?为什么要这么定
义?为什么是这样定义而不是那样定义?可以从来下简单的例子的启发由此导
出这个定义?如果不这么定义有什么问题?一个定理到底是怎么来的?可以从
哪些显而易见的事实中归纳出来?我自己是否可以尝试自己去证明这个定理?
为什么要这么证明?有没有其他的证明思路和证明方法?要点在哪儿?这些方
法各自的优缺点在哪儿?这个定理有什么应用?剩下的,就是做一些习题练习
和尝试修改定理的条件看看能否得到其他的结果。
具体的操作手段 (请结合本文的姊妹篇的学习心法部分),学数学至少有三
遍。第一遍:看一遍书上的主要内容,过一遍书上的主要证明逻辑。第二遍:
做一遍书上的习题,并对书上的定义定理给出例子和直观理解,对证明写出要
点评注。第三遍:默写全书的主要内容。三者亦可以有机的结合在一起。
最核心的东西是:去思考定义和定理的来龙去脉,质疑它们为什么会是这
样子。学习数学有几个东西很重要:一、怀疑二、自信三、观察四、思考
在我看来,数学的本质在于,怀疑一切的力量 (可以参见本文姊妹篇中的
这部分内容,里面有着详细的介绍)。剩下的,需要的是深刻而空灵的洞察力。
看书的目的不是要你去相信,也不是要你去检验一遍书上的定理证明和定
义是否有着逻辑上的错误。你可以尝试站在上帝的眼光去看待一本数学教材和
参考书: 你是来找出该书有什么不合理的地方的,你是来找出书中隐藏的错误
的,你是来与作者共同修改和完善这本教材的,你是来检查书上的习题安排是
否合理以及是否有错误的习题的,你是来重新获得前人赠予的知识和探索前人
走过的路。而不是去“相信”这本教材,没有什么东西是在独立思考之前值得
“相信”的。
那些前人赠予我们的知识,如果我们需要,我们还得重新去“获取”他们。
但看书只是一方面,对于数学而言,是一定要做题的,你不做题是无法检
验自己的理解程度的。要有一定的习题量但也不能恋战。做题是为了增强理解
和记忆,促进独立思考。这也是丘成桐先生的观点。
当然,很多时候看书看第一遍是非常“一脸懵逼”的,这时候可以换一种
方式:听课/看视频代替看书。
相对而言,基础数学研究生课程稍微难一点,我的了解里面。本科的那些
(数学分析、高等代数、解析几何、常微分方程、复变函数、实变函数、泛函分
析、抽象代数、点集拓扑、偏微分方程、微分几何、初等数论) 都还是比较适
合自学的。但研究生的一些课程,比如微分流形、代数拓扑、黎曼曲面、代数
几何、同调代数、黎曼几何、解析数论、代数数论等课程,自学光靠看书的话

24
CHAPTER 3. 数学学习方法论

的确有些费力不讨好,这时候可以考虑先看看一些公开课的视频或者去听老师
讲课,或者先熟悉一下教材和参考书的主要结论而先放弃证明。
听课是一件很有意思的事情,但讲课比听课更有意思。至于如何听课,我
周围有两个特别典型的听课的同学。其一是阿昕 (喜欢玩十三阶魔方的数学天
才少年),这一位是有一些数学天赋的同学,可以说是我本科同学里面最为聪明
的一个。他听课很有意思,以葛优躺的方式瘫坐在凳子上,并且偶尔时不时的
点头。但老师叫他做笔记他也不愿意做笔记,但他是我们公认的数学鬼才,本
学院数学最强王者,没有之一,经常对老师讲过的一些东西保持怀疑态度。
其二是杨秀,这是一位非常喜欢记笔记的女生,几乎把老师在黑板上写得
东西一字不漏的给抄了下来。是一个非常勤奋的妹子,这一点上我是非常佩服
的。但问题在于,她对数学的理解并没有上面的那位同学深刻,并且有时候老
师写错了她也发现不了,仍然把错误的东西写到了笔记本上面。下来之后对笔
记的消化情况也只是一般,也就是笔记没有起到增强理解的效果。
事实上,听课最重要的不是记笔记,这也是我不太愿意做的事情,我更愿
意做的事跟着老师的思路走,最多记一下提纲就行了。至于笔记,肯定会有人
愿意把老师的板书全部抄在黑板上的。有的人是非常勤快的能一字不漏的把老
师的板书写了下来,并且,这种人在每个班上几乎都有。
做事情的时候有比勤奋更为重要的事情,特别是数学这种智力密集型学科
(与之相对的是劳力密集型学科, 只需要好好做事即可),观察和思考比勤奋更为
重要。因为听不懂就想着抄了自己下去看,但是下去也看不懂就不想看从而加
剧下次课听不懂,听不懂然后又想抄下老师的板书,如此恶性循环。在学习的
时候,思考是第一位的,甚至于说思考重于听课,重于看书。思考才是学习的
本质,其他的都是微不足道的。
自然,也有一些可以解决方案,应对这种抄笔记式学习的方法也有不少,
如果能上课不记笔记自然是最好。实在是无法改变的话,我推荐的方法是,第
二天 (早上) 在空白草稿纸上面” 默写” 昨天老师所学过的内容,如果你能把老
师讲得内容全部默写下来,自然是达到了某种理解程度。如果哪里卡住了,说
明还没有完全理解,需要加强这里的学习和思考。把问题弄明白。
常见问题
包括我在内很多同学都有这样的一些问题:数学教材/参考书看不太明白怎
么办?习题不会做怎么办?要看的内容太多了看不完怎么办?这两个问题困扰
着诸多学子。困扰着那些浮躁不安而又急功近利的心。
事实上,这也并非是一个不可解决的问题。仔细想来,为什么会看不懂
呢?要么你忽略了什么细节,要么你对某个概念和定理还理解得不够深刻,要
么你看书根本没有静下心来。你遇到了苦难,你遇到了挑战,遇到了难点。所

25
CHAPTER 3. 数学学习方法论

以你想逃跑,想逃避。你不愿意思考,总是想着求助于外界或者继续拖延时间,
这便是症结所在。
大部分时候,人们为了逃避真正的思考可以做任何事情。
作家蔡志忠对如何开发自己的学习能力、尝试各种开发都身体力行。
他喜欢讲一个故事,一只青蛙想跳上床,自以为会跳就可以。但是高度不
够,又没有方法的时候,他跳一整夜也是白跳。
后来看他整理的笔记,说得更清楚了:“人生不是 斜坡,只要持之以恒,
努力便可到达顶峰。”因此,他主张人生像走阶梯,每一阶有每一阶的难点。无
法克服难点,再怎么努力都只有在原地跳,毫无进展。
学数学也是如此,如果不能突破自己所看不懂的地方,如果不能突破自己
不会做的习题,如果不能抓住重点的学习。如果不能克服这些难点,那大概学
数学学得再久都是没有什么长进的。对数学而言,时间是微不足道的,唯有观
察、反思、思考、计算、想象才是更为重要的。
学数学,不要害怕看不懂,你应该感到高兴的是,小小的突破点终于来了,
你可以去尝试突破自己了。
任何问题,思考和尝试才是解决之道。
很多人都因为害怕看不懂而不愿意找一些数学书来看,但实际上大可不必
这样,应该有的心理准备是,读得懂、看得明白、想得清楚的书一定是从没看
明白而过来的。从另一个角度上看,看第一遍就看得懂的书。真的有很大去读
的必要吗?“……人于书有一见便晓者,天下之弃材也。读书从勤苦中得些许
滋味,自然不肯放下,往往见人家弟子,一见便晓,多无成就。”——清. 李光

“小疑则小悟,大疑则大悟,不疑则不悟。”——清. 李光地
事实上,也只有自己不懂之后,某一刻/某一天突然懂了,才是最开心的
事。所以,千万不要因为没有明白就放弃了思考,千万不要因为自己不懂而远
离一本书。
二十学问风与华,三千疑惑妙和趣。数理乾坤。
“读经有一 字要 ,一句不通,不看下句,今日不通,明日再读; 今年不
精,明年再读; 此所谓 也。困时切莫间断, 过此关,便可少进。再进再困,
再 再奋,自有亨通精进之日。不特写字,凡事都有极端困难之时,打得通的,
便是好汉。”——曾国藩 (这里的困解释为困难而不是犯困)
“读书如譬若掘井,掘数十井而不及一泉,不如掘一井而见泉。读书总以
背熟经书,常讲史鉴为要,每日有常,自有进境,万不可厌旧喜新,此书未完,
勿换彼书耳。”——曾国藩
“不深思,则不能造于道,不深思而得者,其得易失。”——曾国藩

26
CHAPTER 3. 数学学习方法论

纯粹数学
“午后,你在林荫小道散步时,或者夜深人静思绪迸发时,不经意地,你
想到那个定理/问题,并为它的精巧构思惊叹不已,这就是数学。”
“我的工作本身就是对我的奖励,除此之外都是微不足道的。”– 佩雷尔曼
分水岭
大抵学数学并有志于从事数学理论研究的人,都得经历一个痛苦的蜕变,
尤其是对于基础数学专业的学生。他们面临的是从学习数学知识到创造数学知
识,从做别人的习题到解决一些学术界尚未解决的问题。由于数学学科的特殊
性,他们在开始走上研究道路的时候会经历很多的挫折和苦难。如同一位即将
分娩的母亲,经历过绝望与挣扎过后,才会有光明和希望的出现。数学家阿蒂
亚也有类似的表述。
如果在这个分水岭放弃了,可能就不在想从事数学研究方面的工作了。这
其中的 Key 是,本科阶段学生们是在“学习”数学,而不是在“思考”数学,
是在向大脑中“输入数学“而不是从大脑中向外“输出数学”。
数学理论研究是中青年人的事业。
“什么时候开始做学问都没有关系,做学问也不用那么在意有没有才华。
要攀登山顶从哪里出发都无所谓。唯一必要的就是持续不断。只有能够这样,
总有攀上顶峰的一天。”——本居宣长《登山事始》
“我基本对做研究不感兴趣,而且我也没有做过什么研究。我对于理解事
物感兴趣,这对我来讲与做研究颇有不同。经常地,为了理解一些东西,你需
要完全依靠自己把它搞定,因为没有别的人曾经做过这一件事。”
参考文献
《黎曼几何初步》伍鸿熙
《惰者集》小平邦彦
《给年轻数学人的信》
《当科研成为一种职业》
《给青年知识追求者的十封信》
《越读者》郝明义

27
CHAPTER 3. 数学学习方法论

28
Part II

笔记

29
第 4 章 多重线性函数

本章从对偶空间开始讲起, 即研究线性函数的全体所构成的空间的性质, 从
单变量线性函数自然过渡到多变量的线性函数的概念, 称之为多重线性函数, 而
多重线性函数在一组基下的表示便是张量的分量表示, 类似于线性变换在一组
基下的表示就是一个矩阵, 自然我们会研究不同的基下的表示, 即张量的分量表
示在不同基下应该具有怎样的关系即是张量分量的坐标变换公式, 当张量的分
量表示退化为一个矩阵时, 上述坐标变换公式之间退化为矩阵的相似 (因为线性
变换在不同的基下的矩阵是相似的). 不同阶数的张量之间做一种新的乘积使乘
积的结果是具有非常良好的性质 (仍然是张量, 结合律等), 这便是张量积. 借助
于对称矩阵和实对称矩阵的启发, 自然引出了对称张量和反对称张量的概念. 张
量的反称化便是外形式 (外矢量), 而对张量积的反称化运算便得到外积. 我们把
所有的外形式放在一起便得到外形式空间, 借助于外积的基本性质研究了外形
式空间的基和维数, 证明了著名的 Cartan 引理. 并站在外积的角度说明了行列
式的本质. 布置了两道习题, 第一个习题与 Cartan 结构方程有密切关系, 第二
个习题是著名的分解定理. 两个习题均给出了详细证明过程.

4.1 对偶空间
2019-9-9
本节要点/问题提示:
1)、对偶空间的定义? 能不能举个具体的例子?
2)、定义一个线性结构使得对偶空间是一个线性空间?
3)、对偶空间的基和维数? 如何证明?
4)、对偶空间的对偶空间与原空间的关系是什么? 如何证明?
5)、什么是对偶基? 能不能举个具体的例子?
6)、在原空间作基变换得到了新的基, 给出新的基的对偶基与原来的对偶基

31
4.1. 对偶空间 CHAPTER 4. 多重线性函数

的关系和变化规律?
7)、什么是 Einstein 求和约定? 给出从高等代数矩阵的逆和原矩阵相乘等
于单位阵的例子, 并且最后用 Einstein 求和约定写出?
现代数学浅论 从本科的老三基 (数学分析、高等代数、解析几何) 新三基
(抽象代数、几何与拓扑、实变与泛函), 数学从具体的事物走向了抽象的高度,
这些基础学科也划分了数学的三大研究领域: 分析与方程, 代数与数论, 几何与
拓扑. 而本科的数学分析可以说是从求曲面梯形的面积而发展出来的一门学科,
其本质就是微积分, 但这种微积分是有局限的, 虽然大家学过曲面曲线积分, 可
是真正意义上在球面上做微积分时却很难把它说清楚, 并且对于多元微积分和
一元微积分的本质区别也不是很了解. 这门微分流形课程里面有这些问题的答
案, 粗略来说微分流形就是流形上的微积分 (或者称之为大范围分析), 所以自然
可以回答前面的问题, 如何在球面上做微积分. 而一元微积分和多元微积分的本
质区别在于外微分形式, 这也是陈省身先生的观点. 并且可以统一本科的三个曲
面曲线积分相互联系的公式 (Green 公式、Stokes 公式、Gauss 公式), 即流形
上的 Stokes 公式. 实变函数最初的出发点是人们对 Dirichlet 函数不可积性的
研究. 其中心问题是寻找一种更为普遍的积分与测度使得常见的 Dirichlet 函数
可积. 泛函分析的思想来源于把函数看成一个点, 对所有的点构成的空间整体性
质研究便成为了中心问题. 但对空间的整体研究是很难的, 更多的时候人们通过
空间上的映射来研究空间整体的性质. 从线性代数的观点看, 泛函分析更像是一
种无限维空间的线性代数学, 这是因为大部分函数空间等都是无限维的. 赋予线
性结构之后便可成为线性空间. 抽象代数起源于 Galois 对五次以上代数方程的
研究 (二次方程的代数方程的根人们早已熟知), 人们注意到了一些特殊的代数
系统 (群环域模格体) 并对此展开了系统性研究.
分析学粗略说是一切与极限有关的数学科学, 方程更多的时候与物理和生
物学有关. 数论可以说是关于整数的科学, 代数是关于代数系统及其图表的性质
研究. 而几何与拓扑是关于空间形式和图形的科学.
对于数学而言, 不仅仅是做几个习题, 看得懂书上的证明就 OK 了. 更最重
要的是正确的思考方式, 要学会提问为什么, 并且尽量去思考每一个问题. 读数
学书也不仅仅只是看得懂行了. 你得知道它为什么要这么做, 事实上, 当一本数
学书对你而言像你看中学数学书时那么自然地时候, 你才开始对数学产生自己
的理解. 学习在某种意义上讲是前人经验和成果的必要知晓, 那些前辈赠予我们
的知识, 如果我们需要, 我们还得重新自己” 获取” 它们.

设 V 是一个 R 上的 n 维线性空间,V ⋆ 为 V 上线性函数所构成的全体. 即


V ⋆ = {f | f 为V 上的线性函数} .

命题 4.1. V ⋆ 上可定义线性结构, 使得 V ⋆ 成为一线性空间.

32
CHAPTER 4. 多重线性函数 4.1. 对偶空间

证明. 定义线性结构如下:

(f + g)(v) := f (v) + g(v), ∀ f, g ∈ V ⋆ , v ∈ V.

(λf )(v) := λf (v), ∀ f ∈ V ⋆ , λ ∈ R.

我们断言 f + g ∈ V ⋆ , λf ∈ V ⋆ . 事实上, 对任意的 v1 , v2 ∈ V 有

(f + g)(v1 + v2 ) = f (v1 + v2 ) + g(v1 + v2 )


= f (v1 ) + f (v2 ) + g(v1 ) + g(v2 )
= f (v1 ) + g(v1 ) + f (v2 ) + g(v2 )
= (f + g)(v1 ) + (f + g)(v2 ).

对任意的 λ ∈ R, v ∈ V, 有

(f + g)(λv) = λ(f + g)(v)


= λ(f (v) + g(v))
= λ((f + g)(v))

这说明 f + g ∈ V ⋆ . 同理可证 λf ∈ V ⋆ . 于是断言成立. 这说明 V ⋆ 是一线性空


间.

注解 4.1. 上诉命题的意义在于, 为 V ⋆ 这个对象赋予了一种线性结构, 谈到结构


这个词, 不得不提到布尔巴基学派所提出的三大数学结构. 即拓扑结构 (邻域概
念的推广), 代数结构 (加法的推广), 序结构 (大于小于概念的推广). 在布尔巴基
学派看来, 数学对象无外乎集合加上结构. 事实上, 我们只有集合是不能用于做
一些具体的研究的, 之所以结构一直被很大一部分人所忽视是因为在常见的例
子和研究对象中已经赋予了其某种数学结构. 在这里, 所谓的线性结构是指的定
义的加法与数量乘法.

定义 4.1. 称 V ⋆ 为 V 的对偶空间 (共轭空间), 如果 V ⋆ 是上述赋予了线性结构


的线性空间.

问题 4.1. 设 dim V = n, 问 dim V ⋆ =?

证明. 设 e1 , e2 , · · · , en 为线性空间 V 的一组基, 于是对 ∀ v ∈ V ,v 可用 ei 的线


性表示, 即

n
v = a e1 + a e2 + · · · + a en =
1 2 n
a i ei .
i=1

33
4.1. 对偶空间 CHAPTER 4. 多重线性函数

设 αi 为第 i 个坐标分量函数, 即 αi (v) = ai . 特别地, 如果取 v = ej . 显然有



1, i = j,
δji = αi (ej ) =
0, i ̸= j

我们断言 αi ∈ V ⋆ , 事实上, 对 ∀ v1 , v2 , v ∈ V, λ ∈ R. 其中 v1 , v1 的坐标表示分


别为

n ∑
n ∑
n
v1 = ai1 ei , v2 = ai2 ei , a i ei
i=1 i=1 i=1

于是

n ∑n ∑
n
αi (v1 +v2 ) = αi ( (aj1 +aj2 )ej ) = ai1 +ai2 = αi ( aj1 ej )+αi ( aj2 ej ) = αi (v1 )+αi (v2 ).
j=1 j=1 j=1


n ∑n ∑n
αi (λv) = αi (λ a j ej ) = α i ( λaj ej ) = λai = λαi ( aj ej ) = λαi (v).
j=1 j=1 j=1

这说明断言成立. 下面说明 α1 , α2 , · · · , αn 为 V ⋆ 的一组基. 即证明任意元素


可用这一组元素表出和这组元素的线性无关性. 对 ∀ α ∈ V ⋆ , v ∈ V, 其中
∑n
v = i=1 ai ei . 考虑 α 作用于 v, 进一步有

∑n ∑
n ∑
n ∑
n ∑
n
α(v) = α( ai ei ) = ai α( ei ) = a i α(ei ) = αi (v) α(ei ) = α(ei )αi (v)
i=1 i=1 i=1 i=1 i=1

n
=( α(ei )αi )(v)
i=1
∑n
由 v 的任意性知 α = i=1 α(ei )αi , 即任意 V ⋆ 中的元素都能被 αi 的线性组合
表出. 不妨设存在 k1 , k2 , · · · , kn 使得

k1 α1 + k2 α2 + · · · + kn αn = 0.

上式两边同时作用于 ei

(k1 α1 + k2 α2 + · · · + kn αn )(ei ) = 0(ei ),

(k1 α1 + k2 α2 + · · · + kn αn )(ei ) = 0,

k1 δi1 + k2 δi2 + · · · + kn δin = 0.

利用 δji 的性质得到 ki = 0, ∀ i = 1, 2, · · · , n. 这就说明了 αi 是 V ⋆ 的一组基


(并且称 αi 为 ei 的对偶基). 进一步我们得到 dimV ⋆ = dimV = n.

34
CHAPTER 4. 多重线性函数 4.1. 对偶空间

注解 4.2. 我们通过一个简单的例子来” 看见” 这个定理, 不妨设 V = R, 显然


dimV = 1, 定义映射 f 如下:

fk : V = R 7→ R.

r 7→ f (r) = kr, k ∈ R.

显然 R⋆ 都可以表为上述形式. 并且 R⋆ 同构于全体 k 所构成的空间 (这是


自然的, 因为每一个 f 都有唯一的一个 k 与之对应, 每一个 k 也有一个 f 与
之对应). 于是 R⋆ ∼
= R. 这就得到了 dimR⋆ = dimR. 进一步我们可以猜测
dimV ⋆ = dimV.

注解 4.3. 上述证明的关键点在于任意元素可表出的证明. 注意到 ai = α(v) 和


括号所打的位置这两点即可.

∑n
注解 4.4. 求和约定为上下标相同的求和, 如 αi (v)ei 即为 i=1 αi (v)ei , 称之为
Einstein 求和约定, 求和指标称之为哑指标 (哑标). 但初学者往往看重求和的
形式而不注意这种求和的本质, 所以初学者对这种求和指标表现得非常不适应

注解 4.5. 仿造上述证明可得 v = αi (v)ei = ⟨αi , v⟩ei ,α = α(ei )αi = ⟨ei , α⟩αi . 通


过这个式子我们得到推论 (V ⋆ )⋆ ∼
= V , 因为上面的式子写成了内积的形式, 显然
可以得到 ei 是 V ⋆ 上的线性函数, 这种与基选取无关的同构称之为自然同构.

接下来的问题是, 在上述讨论中我们所给的基是随意给的, 如果换一组基,


其对偶基会怎么变化?

问题 4.2. 设 ei 为 V 中一组基,eei = aji ej , 其中 det(aji ) ̸= 0., 问 αei 的变换规律?

证明. 不妨设 = αei = bij ej , 考虑 δji = αei (eei ) = (bil al )(aki ek ) = bil aki al (ek ) =
bil aki δkl = bil ali . 这说明 aij 和 bij 互逆. 即 bij = (aij )−1

注解 4.6. 在对偶空间这里让一个对偶基作用于原来的基是常见的技巧. 但在带


入的时候一定要注意不能直接带进去计算, 不能多加入哑指标, 所以在带入的时
候一定要注意之前式子的非哑指标, 带入之后仍然必须是非哑指标. 之前的求和
指标, 带入新的式子之后仍然是求和指标. 另外需要说明的是这两个矩阵的确是
互逆的, 但对于初学者而言, 这并不显然, 所以在这里做一些计算以确定这个事

35
4.2. 张量代数 CHAPTER 4. 多重线性函数

实. 记号如上. 令 A = aij , B = bkl , 考虑 A 与 B 的乘积.


  
a11 a12 · · · a1n b11 b12 · · · b1n
 2  
 a1 a22 · · · a2n   b21 b22 · · · b2n 
  
AB =  . .. .. ..  . .. .. .. 
 .. . .  
.  . . . . .
 
an1 an2 · · · ann bn1 bn2 · · · bnn
 
a11 b11 + a12 b21 + · · · a1n bn1 · · · a11 b1n + a12 b2n + · · · a1n bnn
 .. .. .. 
= . . .


an1 b11 + an2 b21 + · · · ann bn1 ··· an1 b1n + an2 b2n + · · · ann bnn

利用 Einstein 求和约定, 可写为


 
a1l bl1 ··· a1l bln
 . .. .. 
AB = 
 .
. . . 

anl bl1 ··· anl bln

而 AB 的乘积中的每个元素即是 δji , 这说明 AB = E. 通过上面的符号计算, 我


们发现 A 和 B 的的确确是互逆的. 而不是想当然的认为 aij 和 bij 是互逆的. 对
于自己不太熟悉或者不太清楚的计算, 如果能自己去算一算, 自己花些时间思考,
也许能收获更多.

4.2 张量代数
2019-9-18
本节要点/问题提示:
1)、多重线性函数的定义? 能不能举几个具体的例子?
2)、定义一个线性结构使得 L(V1 , V2 , · · · , Vr ; F ) 是一个线性空间?
3)、如何对 αi ∈ V ⋆ 和 β j ∈ W ⋆ 作张量积使得做完张量积之后的元素属于
L(V, W ; F )?
4)、根据上述张量积猜测 L(V, W ; F ) 的基和维数? 如何证明?
5)、将上述两个元素的张量积推广到多个元素?
6)、张量积的本质是什么?
7)、根据上面的问题猜测 L(V1 , V2 , · · · , Vr ; F ) 的基和维数? 按照之前相同
的思路证明?
8)、张量在基下的表示形式如何写? 如果作基变换, 变换后的基下的表示形
式与原来的表示形式之间的变化关系是什么? 即张量分量表示的坐标变换公式
是什么

36
CHAPTER 4. 多重线性函数 4.2. 张量代数

9)、说明张量的坐标变换公式是矩阵相似的变换公式的推广?
回顾: 设 V 是一 n 维线性空间,ei 是 V 中的一组基,ei 是 V ⋆ 中的一组基
(也是 ei 的对偶基), 即 ei (ej ) = δji . 对任意的 α ∈ V ⋆ , 有 α = α(ei )ei , 对任
意的 v ∈ V, 有 v = ei (v)ei , 并且在基变换 eei = aji ej 下, 其对偶基变化情况为
eei = bi ej , 其中 ai bl = δ i . 即 (ai )−1 = bi .
j l j j j j

例 4.1. 二重线性函数 (双线性型)


{ }
记 Rn = x = (x1 , x2 , · · · , xn ) | xi ∈ R, 1 ≤ i ≤ n . 定义

f : R × R 7→ R.

f (x, y) = aij xi y j .

满足如下性质:
1) f (x1 + x2 , y) = f (x1 , y) + f (x2 , y).
2) f (λx, y) = λf (x, y) = f (x, λy).
3) f (x, y1 + y2 ) = f (x, y1 ) + f (x, y2 ).
上述性质的验证是显然的, 简单的计算即可. 我们只以 1) 为例验证, 事
实上, 可设 x1 = (x11 , x21 , · · · , xn1 ), x2 = (x12 , x22 , · · · , xn2 ). 于是 f (x1 + x2 , y) =
aij (xi1 + xi2 )y j = aij xi1 y j + aij xi2 y j = f (x1 , y) + f (x2 , y). 同理可验证另外两条
性质.
aij +aji
若记 F (x) = aij xi xj , 不妨设 aij 是对称的 (若不然, 令 bij = 2 , 显然
bij = bji , 即 bij 是对称的, 这种构造对称矩阵 (对称二阶张量) 的方式称之为对
称化). 我们断言 F (x) 和 f (x, y) 是等价的, 事实上,f (x, y) 推出 F (x) 是显然的,
只需令 F (x) = f (x, x) 即可. 另一方面,F (x + y) = f (x + y, x + y) = f (x, x) +
f (x, y) + f (y, x) + f (y, y) = f (x, x) + f (y, y) + 2f (x, y) = F (x) + F (y) + 2f (x, y),
F (x,y)−F (x)−F (y)
进一步 f (x, y) = 2 . 称 f (x, y) 是 F (x) 的完全极化多项式.

例 4.2. n 重线性函数 (行列式)


定 义 映 射 f : R × R × · · · × R 7→ R 对 任 意 的 x1 , x2 , · · · xn . 其 中 xi =
| {z }
n
x1 x1 · · · x1
1 2 n
2
x1 x22 · · · x2n

(xi1 , xi2 , · · · , xin ). 定义 f (x1 , x2 , · · · , xn ) = . .. .. .. .f 具有如下性质:
.. . . .

n
x1 x2 · · · xnn
n

1) f (x1 , x2 , · · · , xi +y i , · · · , xn ) = f (x1 , x2 , · · · , xi , · · · , xn )+f (x1 , x2 , · · · , y i , · · · , xn ).


2) f (x1 , x2 , · · · , λxi , · · · , xn ) = λf (x1 , x2 , · · · , xi , · · · , xn ).
上述两条性质在高等代数行列式的性质中早已学过.

37
4.2. 张量代数 CHAPTER 4. 多重线性函数

受到上面两个例子的启发, 我们有如下定义.

定义 4.2. 设 V1 , V2 , · · · , Vr 是 r 个线性空间,F 为域,dim Vk = nk , 如果函数


f : V1 × V2 × · · · × Vr 7→ F 如果 f 关于每一个变量都是线性的, 即满足:
1) f (x1 , · · · , λxk , · · · , xr ) = λf (x1 , · · · , xk , · · · , xr ), ∀ λ ∈ R
2) f (x1 , · · · , xk +y k , · · · , xr ) = f (x1 , · · · , xk , · · · , xr )+f (x1 , · · · , y k , · · · , xr ), ∀ y ∈
Vk .
则称 f 为 V1 × V2 × · · · × Vr 上的 r 重线性函数. 并且把 V1 × V2 × · · · × Vr
上所有线性函数的全体记为 L(V1 , V2 , · · · , Vr ; F ).

命题 4.2. L(V1 , V2 , · · · , Vr ; F ) 上可赋予一线性结构, 使得 L(V1 , V2 , · · · , Vr ; F )


为线性空间.

证明. 定义线性结构如下:

(f +g)(v1 , v2 , · · · , vr ) := f (v1 , v2 , · · · , vr )+g(v1 , v2 , · · · , vr ), ∀ f, g ∈ L(V1 , V2 , · · · , Vr ; F ).

(λf )(v1 , v2 , · · · , vr ) := λ(f (v1 , v2 , · · · , vr )), ∀ λ ∈ R.

容易验证 L(V1 , V2 , · · · , Vr ; F ) 按上述线性结构是一线性空间.

接下来的问题是,L(V1 , V2 , · · · , Vr ; F ) 的基和维数, 当然现在这还是一个比


较困难的问题. 我们通过几个例子来观察以尝试得到启发.

例 4.3. 设 V 是一 n 维向量空间,L(V ; F ) 是 V 7→ F 上全体线性函数, 即 V ⋆ . 这


说明 L(V ; F ) = V ⋆ , 于是 L(V ; F ) 的基即 V ⋆ 中的一组基 ei , 1 ≤ i ≤ n.

通过这个例子得到启发,L(V1 , V2 , · · · , Vr ; F ) 应该与 V1⋆ , V2⋆ , · · · , Vr⋆ 很有关


系, 但问题在于, 如何把这些空间” 放到一起去”, 在下面这个例子中, 我们将能
够更加具体的看到这个问题.

例 4.4. 设 V 是一 n 维向量空间,W 是一 m 维向量空间, 我们想求 L(V, W ; F )


的基和维数. 不妨设 αi 为 V 中的一组基, 其中 1 ≤ i ≤ n.,αi 为其对偶基.βj
为 W 中的一组基,1 ≤ j ≤ m,β j 为其对偶基. 显然 αi 是定义在 V 上的线性函
数,β j 是定义在 W 上的线性函数, 我们的基本想法是, 通过把 αi 和 β j ” 放在一
起”, 使得” 放在一起” 后得到的函数是一个双线性函数. 即” 放在一起” 后的函
数关于 (v, w) ∈ V × W 是双线性型函数. 下面我们就来尝试做这么一件事情, 记
这种放在一起的运算为 ⊗.
αi ⊗ β j 是关于 (v, w) 的双线性函数, 很自然的一个想法是, 直接把 αi 作用
于 v, 把 β i 作用于 w 然后再相乘, 这是因为乘法是我们见过最为自然的 (多重)
线性函数. 即 αi ⊗ β j (v, w) = αi (v)β j (w).

38
CHAPTER 4. 多重线性函数 4.2. 张量代数

下面我们断言 αi ⊗β j 是 L(V, W ; F ) 的一组基. 事实上, 对 ∀ f ∈ L(V, W ; F ),


显 然 αi ⊗ β j ∈ L(V, W ; F ). 进 一 步 对 任 意 的 (v, w) ∈ (V, W ), 其 中 v =
αk (v)αk , w = β l (w)βl . 注意到

f (v, w) = f (αk (v)αk , β l (w)βl )


= αk (v)β l (w)f (αk , βl )
= f (αk , βl )αk (v)β l (w)
= f (αk , βl )(αk ⊗ β l )(v, w)

即 f = f (αk , βl )(αk ⊗ β l ). 不妨设存在 kij 使得

k11 α1 ⊗ β 1 + · · · + knm αn ⊗ β m = 0.

上式两边同时作用于 (αk , βl ), 其中 1 ≤ k ≤ n,1 ≤ l ≤ m 有

(k11 α1 ⊗ β 1 + · · · + knm αn ⊗ β m )(αk , βl ) = 0.

k11 δk1 δl1 + · · · + knm δkn δlm = 0.

进一步 kkl = 0. 这就证明了断言.


正是因为 L(V, W ; F ) 可以用 V ⋆ 和 W ⋆ 的” 放在一起运算” 表出. 所以又
记 V ⋆ ⊗ W ⋆ = L(V, W ; F )

现在, 我们需要严格的来定义该” 放在一起” 的运算, 我们称之为张量积.

定义 4.3. 设 V1 , V2 , · · · , Vp , W1 , W2 , · · · , Wq 是域 F 上的线性空间, 定义 α ∈
L(V1 , V2 , · · · , Vp ; F ) 与 β ∈ L(W1 , W2 , · · · , Wq ; F ) 的张量积 α ⊗ β 如下: 对任
意的 (v1 , v2 , · · · , vp , w1 , w2 , · · · , wq ) ∈ V1 × V2 × · · · × Vp × W1 × W2 × · · · × Wq .

(α ⊗ β)(v1 , v2 , · · · , vp , w1 , w2 , · · · , wq ) = α(v1 , v2 , · · · , vp )β(w1 , w2 , · · · , wq ).

显然有

(α ⊗ β)(v1 , · · · , vi + vi , · · · , vp , w1 , w2 , · · · , wq ) = α(v1 , · · · , vi , · · · , vp )β(w1 , w2 , · · · , wq )+

α(v1 , · · · , vi , · · · , vp )β(w1 , w2 , · · · , wq ).

(α ⊗ β)(v1 , v2 , · · · , vp , w1 , · · · , wj + wj , · · · , wq ) = α(v1 , v2 , · · · , vp )β(w1 , · · · , wj , · · · , wq )+

α(v1 , v2 , · · · , vp )β(w1 , · · · , wj , · · · , wq ).

(α ⊗ β)(v1 , · · · , λvi , · · · , vp , w1 , w2 , · · · , wq ) = λα(v1 , · · · , vi , · · · , vp )β(w1 , w2 , · · · , wq ).

(α ⊗ β)(v1 , v2 , · · · , vp , w1 , · · · , λwj , · · · , wq ) = λα(v1 , v2 , · · · , vp )β(w1 , · · · , wj , · · · , wq ).


这说明 α ⊗ β ∈ L(V1 , V2 , · · · , Vp , W1 , W2 , · · · , Wq ; F )

39
4.2. 张量代数 CHAPTER 4. 多重线性函数

注解 4.7. 张量积的本质就是用 p 重线性函数和 q 重线性函数构造一个 p + q 重


线性函数的一种方法, 顾名思义, 可以理解为从低重线性函数扩张成更高重数的
线性函数的” 乘积”(低阶张量做乘积成为高阶张量). 但张量积与普通乘积是有
区别的, 比如说普通乘积是有交换律的, 而张量积是没有交换律的.

性质 4.1. 设 α1 , α2 , α ∈ L(V1 , V2 , · · · , Vp ; F ), β ∈ L(W1 , W2 , · · · , Wq ; F ), γ ∈


L(Z1 , Z2 , · · · , Zr ; F ), 有下列三条成立:
1) (α1 + α2 ) ⊗ β = α1 ⊗ β + α2 ⊗ β.
2) α ⊗ (β1 + β2 ) = α ⊗ β1 + α ⊗ β2 .
3) (α ⊗ β) ⊗ γ = α ⊗ (β ⊗ γ).

证明. 1) 根据定义,

(α1 + α2 ) ⊗ β(v1 , v2 , · · · , vp , w1 , w2 , · · · , wq ) = (α1 + α2 )(v1 , v2 , · · · , vp )β(w1 , w2 , · · · , wq )


= α1 (v1 , v2 , · · · , vp )β(w1 , w2 , · · · , wq )+
α2 (v1 , v2 , · · · , vp )β(w1 , w2 , · · · , wq )
= α1 ⊗ β(v1 , v2 , · · · , vp , w1 , w2 , · · · , wq )+
α2 ⊗ β(v1 , v2 , · · · , vp , w1 , w2 , · · · , wq )
= (α1 ⊗ β + α2 ⊗ β)(v1 , v2 , · · · , vp , w1 , w2 , · · · , wq )

这说明 (α1 + α2 ) ⊗ β = α1 ⊗ β + α2 ⊗ β.
2) 同理可证.
3) 根据定义,

(α ⊗ β) ⊗ γ(v1 , v2 , · · · , vp , w1 , w2 , · · · , wq , z1 , z2 , · · · , zr )
= (α ⊗ β)(v1 , v2 , · · · , vp , w1 , w2 , · · · , wq )γ(z1 , z2 , · · · , zr )
= α(v1 , v2 , · · · , vp )β(w1 , w2 , · · · , wq )γ(z1 , z2 , · · · , zr )
= α(v1 , v2 , · · · , vp )β ⊗ γ(w1 , w2 , · · · , wq , z1 , z2 , · · · , zr )
= α ⊗ (β ⊗ γ)(v1 , v2 , · · · , vp , w1 , w2 , · · · , wq , z1 , z2 , · · · , zr )

进一步有 (α ⊗ β) ⊗ γ = α ⊗ (β ⊗ γ).

注解 4.8. 这里给出的几个性质中, 第三个性质与老师在黑板上写的是不太一样


的, 老师写的是一个更为退化的性质, 在实际应用中退化的性质用得很少, 所以
这里改成了一个更为普遍的结论.

有了张量积的基础知识, 例 4.4 的讨论自然能推导到有限个线性空间上去.


这就回答了之前的问题.

40
CHAPTER 4. 多重线性函数 4.2. 张量代数

推论 4.1. 记 V1⋆ ⊗ Vr⋆ ⊗ · · · ⊗ Vr⋆ = L(V1 , V2 , · · · , Vr ; F ) 设 ai11 , ai22 , · · · , airr 分别


为 V1⋆ , V2⋆ , · · · , Vr⋆ 中的一组基, 其中 1 ≤ i1 ≤ dim V1⋆ , 1 ≤ i2 ≤ dim V2⋆ , · · · , 1 ≤
{ }
ir ≤ dim Vr⋆ . 则 ai11 ⊗ ai22 ⊗ · · · ⊗ airr 为 V1⋆ ⊗Vr⋆ ⊗· · ·⊗Vr⋆ 中的一组基, 特别地,
若 V1 = V2 = · · · = Vr = V, 则记 Vr0 = V ⋆ ⊗ V ⋆ ⊗ · · · ⊗ V ⋆ = L(V, V, · · · , V ; F )

显然上述 V 可以换成 V ⋆ ,V ⋆ 可以换成 V , 同样地有.

推论 4.2. 记 V1 ⊗Vr ⊗· · ·⊗Vr = L(V1⋆ , V2⋆ , · · · , Vr⋆ ; F ) 设 a1i1 , a2i2 , · · · , arir 分别为
V1 , V2 , · · · , Vr 中的一组基, 其中 1 ≤ i1 ≤ dim V1 , 1 ≤ i2 ≤ dim V2 , · · · , 1 ≤ ir ≤
{ }
dim Vr . 则 a1i1 ⊗ a2i2 ⊗ · · · ⊗ arir 为 V1⋆ ⊗ Vr⋆ ⊗ · · · ⊗ Vr⋆ 中的一组基, 特别地,
若 V1 = V2 = · · · = Vr = V, 则记 V0r = V ⊗ V ⊗ · · · ⊗ V = L(V ⋆ , V ⋆ , · · · , V ⋆ ; F )

定义 4.4. 设 V 是 n 维线性空间,V ⋆ 为 V 的对偶空间,(p, q) 是一对非负整


数,Vqp := V ⊗ V · · · ⊗ V ⊗ V ⋆ ⊗ V ⋆ · · · ⊗ V ⋆ = L(V ⋆ ⊗ V ⋆ · · · ⊗ V ⋆ ⊗ V ⊗ V · · · ⊗ V ; F )
| {z } | {z } | {z } | {z }
p q p q
称之为 (p, q) 型张量空间. 其中的元素称之为 (p, q) 型张量. 设 ai 为 V 的基,ai
{ }
为 其 对 偶 基, 则 ai1 ⊗ · · · ⊗ aip ⊗ aj1 · · · ⊗ ajq 为 Vqp 一 组 基. 即 对 任 意 的
i ···i
f ∈ Vqp , f = f (ai1 , · · · , aip , aj1 , · · · , ajq )ai1 ⊗ · · · ⊗ aip ⊗ aj1 · · · ⊗ ajq = fj11···jqp
ai1 ⊗ · · · ⊗ aip ⊗ aj1 · · · ⊗ ajq .

问题 4.3. 以 ai 为原始基底, 在基变换 ai = aji ej 下 (其对偶基变换为 ai = bij ej ,


其中 det(aji ) ̸= 0.), 问张量的坐标变换公式?
i ···i
证明. 在原来的基 {ai } 下, 有 f = fj11···jqp ai1 ⊗ · · · ⊗ aip ⊗ aj1 · · · ⊗ ajq . 设在新
基 {ei } 下, 有
^i ···i
f = fj11···jqp ei1 ⊗ · · · ⊗ eip ⊗ ej1 · · · ⊗ ejq . (1)

根据上述基变换公式, 有
i ···i
f = fj11···jqp ai1 ⊗ · · · ⊗ aip ⊗ aj1 · · · ⊗ ajq
i ···i k j
= fj11···jqp aki11 ek1 ⊗ · · · ⊗ aipp ekp ⊗ bjl11 el1 · · · ⊗ blqq elq
i ···i k j
= fj11···jqp aki11 · · · aipp bjl11 · · · blqq ek1 ⊗ · · · ⊗ ekp ⊗ el1 · · · ⊗ elq

因为上述指标的本质在于求和, 为了让上式与 {ei } 下 f 的表达式, 即 (1) 式相


同, 需要做对换 (i1 k1 ) · · · (ip kp )(j1 l1 ) · · · (jq lq ).(即所有的 i1 换成 k1 , 所有的 k1
换成 i1 , 其他以此类推) 进一步上式可写为
i ···i k j
f = fj11···jqp aki11 · · · aipp bjl11 · · · blqq ek1 ⊗ · · · ⊗ ekp ⊗ el1 · · · ⊗ elq
k ···k i l
= fl11···lq p aik11 · · · akpp blj11 · · · bjqq ei1 ⊗ · · · ⊗ ekp ⊗ ej1 · · · ⊗ ejq

41
4.2. 张量代数 CHAPTER 4. 多重线性函数

上式对比 (1) 式, 可得

^i ···i k ···k i l
fj11···jqp = fl11···lq p aik11 · · · akpp blj11 · · · bjqq . (2)

观察基坐标变换公式 ei = bji ej 下, 可以观察到上指标通过 bji 的逆矩阵 aji , 所以


上指标为逆变 (反变指标), 下指标为通过本身的基坐标矩阵, 可以说下指标和基
坐标变换的变化规律是协同的, 于是下指标称之为协变指标.

注解 4.9. 事实上, 老师在讲这里的时候, 是没有注意到一些细节的. 当然老师讲


得本身并没有任何错误, 但对于初学者而言, 这里逆变和协变以及老师最后讲
的新的张量分量表示的坐标变换公式实在是足以绕晕了. 因为老师两次形式看
上去讲得是不一样的, 本质却是一样的, 而一个初学者要去询问为什么这里不
一样以及为什么本质却是一样的本身就不是一个简单的问题. 这里解释一下为
什么这里写的和后面即将写的东西不太一样. 再次回到问题本身, 注意到题目
中所给的坐标变换公式, 基变换 ai = aji ej . 仔细看, 这里是用得 ai 是原来的基,
而 ei 是新的基, 也就是说, 它这里的坐标变换公式是用新的基来表示旧的基. 这
本来就是一个不太常用的表示方法, 通常而言, 我们都是用旧的基来表示新的
基, 而在这里老师却在不知道新的基的情况下用新的基来表示旧的基. 当然不是
说这么做不行, 而是说通常不会这么表示. 所以实际上在常见的情况下应该是
(aji )−1 ai = bij ai = ej . 这时候便可以理解为什么老师讲逆变和协变的时候要观
察 bij 作为变换公式. 因为所有的逆变和协变都是相对于用旧的基表示新的基的
那个矩阵 (坐标变换) 而言的. 而在这里这个变换矩阵是 bji 而不是 aji .

下面的结论是上面讨论的总结, 虽然形式看上去有些区别但本质是一样的.

定理 4.1. 设 V 是 n 维向量空间,δi 为 V 中的一组基.δ i 为 V ⋆ 的一组 (对偶)


基, 并且设 δ¯i 也是 V 中的一组基,δ¯i 也是 V ⋆ 一组 (对偶) 基. 设 δ¯i = aji δj , 其中
det(aji ) ̸= 0, 相应有 δ¯i = bij δ j , 对任意的 f ∈ V ⊗ · · · ⊗ V ⊗ V ⋆ ⊗ · · · ⊗ V ⋆ , 在原
| {z } | {z }
p q
来的基 {δi } 下有
i ···i
f = fj11···jqp δi1 ⊗ · · · ⊗ δip ⊗ δ j1 · · · ⊗ δ jq .
{ }
在新基 δ¯i 下有有

^i ···i
f = fj11···jqp δ¯i1 ⊗ · · · ⊗ δ¯ip ⊗ δ¯j1 · · · ⊗ δ¯jq .

则张量的坐标变换公式为
^i ···i k ···k i l
fj11···jqp = fl11···lq p bik11 · · · bkpp alj11 · · · ajqq . (3)

42
CHAPTER 4. 多重线性函数 4.2. 张量代数

注解 4.10. 对比该定理和上面的问题, 即对比 (2) 式和 (3) 式, 我们可以发现, 两


个式子恰好是翻过来的, 一个式子的 aij 是另外一个式子的 bji 这是因为这里两
个式子 aij 的意义是不一样的, 一个是用新的基表示旧的基, 另外一个是用旧的
基来表示新的基. 在更多的时候, 我们采用后者.

注解 4.11. 之前的例 4.1 中我们提到过二重线性函数 (线性变换), 在基下的表示


也就是二阶张量的分量表示 (矩阵), 现在我们来说明, 一个线性变换在不同基下
的表示是满足 (3) 式的. 在高等代数中我们知道, 一个线性变换在不同的基下的
矩阵是相似的. 我们将会看见, 利用 Einstein 求和约定写出来的相似关系就是
上面张量分量的坐标变换关系的退化. 换言之, 张量分量的坐标变换公式是矩阵
相似的高维推广. 记号如上. 令 Fe = (fei ),F = (f i ),A = (ai ),B = (A)−1 = (bi ).
j j j j
其中 Fe 是新基下的矩阵.F 是原来的基下的矩阵,A 是基坐标变换矩阵,B 是 A
的逆. 显然它们是相似的, 即 Fe = AF A−1 = AF B. 具体写出来便是 (写的时候
已经采用用 Einstein 求和约定):

Fe = AF B
   
a1 a12 ··· a1n f11 f21 ··· fn1 b11 b12 ··· b1n
 21  2  2 
 a1 a22 ··· a2n   f22 ··· fn2   b22 ··· b2n 
   f1   b1 
= . .. .. 
..   .. .. .. 
..   .. .. .. .. 
 .. . . .  .  .
  . . . . . . 
an1 an2 ··· ann f1n f2n · · · fn n
b1 b2 · · · bnn
n n
 
  b1 b12 ··· b1n
1 n  1 
a11 f11 + a12 f12 + · · · a1n f1n ··· a 1 fn + a 2 fn + · · · a n fn
1 1 1 2
  b21 b22 ··· b2n 
=
.. .. ..  
 . . . 
 ... .. .. .. 
n n 
. . .
an1 f11 + an2 f12 + · · · ann f1n ··· a 1 fn + a 2 fn + · · · a n fn
n 1 n 2
bn1 bn2 ··· bnn
 
  b11 b12 · · · b1n
a1l f1l ··· a1l fnl  2 
 .  b1 · · · b2n 
.. .. 
b22 
=
 .
. .  .. 
.  ... ..
.
..
. .
n l  
anl f1l ··· a l fn
bn1 bn2 · · · bn n

43
4.2. 张量代数 CHAPTER 4. 多重线性函数

进一步:

Fe = AF B
 
  b11 b12 ··· b1n
a1l f1l ··· a1l fnl  2 
 . b ··· b2n 
.. ..    .1
b22 
=
 .
. . .  .. .. .. 
 .. . . .
 
anl f1l ··· anl fnl
bn1 bn2 ··· bnn
 
a1l f1l b11 + · · · + a1l fnl bn1 ··· a1l f1l b1n + · · · + a1l fnl bnn
 .. .. .. 
=
 . . .


anl f1l b11 + · · · + anl fnl bn1 · · · anl f1l b1n + · · · + anl fnl bnn
 
a1l fkl bk1 · · · a1l fkl bkn
 . .. .. 
=
 .
. . . 

anl fkl bk1 ··· anl fkl bkn

我们得到等式 feji = ail fkl bkj . 这恰恰是 (3) 式中的张量分量坐标变换公式的


q = 1, q = 1 的情形.
这说明矩阵是 (1, 1) 型张量的分量表示. 在这个意义上讲, 张量的分量表示
无外乎是矩阵的推广. 多重线性函数 (即张量) 无外乎是线性变换的推广. 张量
分量的坐标变换公式无外乎是矩阵相似关系的推广.
除了从相似的角度说明张量的分量表示是矩阵的推广 (张量是线性变换的
推广) 以外, 还有一种有意思的角度, 不知大家是否记得在李锋老师讲运筹学的
第一课的时候, 提到过多元函数的 T aylor 展开, 即
1
f (x) = f (x0 ) + ∇f (x0 )T △ X + (△ X)T H △ X + o((△ X)2 )
2!
老师讲到这里的时候提到了, 如果要写第四项而不是写成无穷小量的形式, 那
么就需要用到张量, 也就是说, 上述展开的本质是张量, 但由于高阶的张量很难
表示就没有用到, 这就说明前面几项应该也是张量 (因为一般项都可以用张量
来表示,说明前面几项也可以用张量表示), 即 0 阶张量的分量表示是一个数
(f (x0 )), 一阶张量的分量表示是一个向量 (∇f (x0 )T ), 二阶张量的分量表示是一
个矩阵 (Hessian 矩阵 H), 三阶张量的分量表示是一个长方体矩阵.
当然上述只是从我们学过的一些基础知识里面去体会这些概念, 从高阶知
识回到基础知识也许能提升我们对知识的理解与体会.

注解 4.12. 关于本质与表现形式. 这一点是高等代数的精华所在, 高等代数的精


华在于, 所有的研究重点都在线性变换上而不是在矩阵上, 因为线性变换比矩阵

44
CHAPTER 4. 多重线性函数 4.3. 外代数

更为本质. 矩阵不过是一个线性变换在一组基下的表示, 而线性变换才是更为本


质的东西, 无论你是否给定一组基, 线性变换就在那里, 它不会随基的变化而变
化, 但矩阵在不同的基下的表现形式是不同的. 如同高中立体几何, 无论你是否
给了一个坐标系, 那个图形就在那里, 而图形的坐标是随坐标系的变化而变化的.
但图形本身是与坐标系无关. 简单比喻来说, 这就像, 你看见了某些漂亮 MM 之
后情不自禁的拍了几张照片, 重要的是这个 MM 本身, 重点不在于照片, 但照
片是我们了解到的一种表现形式. 不同基下的矩阵就像你在不同的角度所拍的
MM 照片, 而线性变换本身是 MM 这个人. 一个向量也是, 无论你是否给定坐标
系, 向量本身是不会随着坐标系改变而改变了, 但向量的坐标却会随着坐标的改
变而改变 (为何有一种马克思主义哲学现象与本质的既视感?).

注解 4.13. 老师在结束这节课之前, 提了一个很有意思的问题, 满足上述的张量


的分量表示的坐标变换公式的” 一堆数” 的全体所构成的空间 (结构) 有什么样
的性质? 实际上, 这便是过去张量的定义: 在基坐标变换下满足坐标变换公式 (3)
的” 一堆数” 定义了一种特殊的” 结构” 就称之为一个张量, 退化到高代的语言
来说, 在基变换下是相似的矩阵的全体定义了一个” 线性变换”.

4.3 外代数
2019-9-25
本节要点/问题提示:
1)、对称张量和反对称张量的定义? 能不能举几个具体的例子?
2)、置换群作用于一个张量的定义??
3)、一个张量 f 是对称张量和反对称张量的充分必要条件 (用置换群的语
言)?
4)、证明反对称张量的全体构成以线性空间 (最重要的是如何定义线性结
构)? 由此说明它是张量空间的一个子空间?
5)、如何从一般的张量出发定义一个新的反对称张量? 给出二阶张量和三
阶张量的例子?
6)、利用置换群定义一个反称化算子, 使得对于任意的张量, 经过反称化算
子后都变成了反对称张量?
7)、是否所有的反对称张量都是某个张量经过反称化算子作用后得到的?
如何证明?
8)、能否对张量积反称化?(因为张量积是构造张量的一种方式, 既然张量有
反称化, 那么张量积无外乎是一个新的张量, 能否借此定义张量积的反称化)

45
4.3. 外代数 CHAPTER 4. 多重线性函数

9)、外积的定义? 外积的常见性质有哪些? 能否因这些性质猜测外形式空间


的基和维数?

例 4.5. 二重线性函数 (对称化与反称化)


设 V 是 n 维线性空间, 定义函数 f : V × V 7→ R 的二重线性函数. 我
们想要构造一种对称的二重线性函数, 受到例 4.1 的启发, 一个自然地想法
1
是考虑 α(x, y) = 2 (f (x, y) + f (y, x)), 显然有 α(x, y) = α(y, x), 即 α(x, y) 是
对称的二阶张量, 同理, 为了构造反对称二重线性函数, 我们考虑 β(x, y) =
1
2 (f (x, y) − f (y, x)).

如何利用两个一阶张量构造一个二阶对称张量和二阶反对称张量? 请看下
面的例子.

例 4.6. 设 η, ξ ∈ V ⋆ , 则 η⊗ξ ∈ V ⋆ ⊗V ⋆ . 令 h = 12 (ξ⊗η+η⊗ξ), g = 21 (ξ⊗η−η⊗ξ).


显然 h 是对称的二阶张量,g 是反对称的二阶张量. 并且 g 还可以写成行列式的
形式. 详细写便是
1
g(x, y) = (ξ ⊗ η − η ⊗ ξ)(x, y)
2
1
= (ξ(x)η(y) − η(x)ξ(y))
2
1 ξ(x) η(x)
=
2 ξ(y) η(y)

二重线性函数 (二阶张量) 是否一定存在某种相对简单形式的表示呢? 请看


下面例子.

例 4.7. 设 V 是 n 维线性空间,δi 是 V 中的一组基,V ⋆ 是其对偶空间,δ i 是其对


偶基. 对任意的 f ∈ V ⋆ ⊗ V ⋆ , f = f (δi , δj )δ i ⊗ δ j , 为方便表示, 令 aij = f (δi , δj ).
若 f 是 对 称 的, 则 aij = aji , 若 f 是 反 对 称 的, 则 aij = −aji . 对 任 意 的
x = xk δk ∈ V, y = y l δl , 有 f (x, y) = f (xk δk , y l δl ) = aij δ i ⊗ δ j (xk δk , y l δl ) =
aij xk y l δ i (δk )δj (δl ) = aij xk y l δki δlj = aij xi y j . 这说明对 V ⋆ ⊗ V ⋆ 中的元素都可
以表为高等代数中的双线性型 (双线性型的一种特例便是二次型), 回答是肯定
的.

从二阶对称张量和二阶反对称张量得到启发, 我们定义更为一般阶数的对
称张量和反对称张量.

定义 4.5. 设 f ∈ V ⋆ ⊗ V ⋆ · · · ⊗ V ⋆ , 对任意的 k, l ∈ 若 f (v1 , · · · , vk , · · · , vl , · · · , vp )


| {z }
p
= f (v1 , · · · , vl , · · · , vk , · · · , vp ), 则称 f 为对称张量. 若 f (v1 , · · · , vk , · · · , vl , · · · , vp )
= −f (v1 , · · · , vl , · · · , vk , · · · , vp ), 则称 f 为反对称张量.

46
CHAPTER 4. 多重线性函数 4.3. 外代数

从上面的定义中, 容易看出对称张量和反对称张量的重要区别就在于做了
一个对换过后的符号改变情况. 注意到更多的时候可能不止一次对换, 也许是好
几次对换的复合. 自然我们会考虑置换群的作用与对称张量和反对称张量的关
系. 但置换群中的元素的写法在不同抽象代数的书中是不一样的. 这里我们规定
( )
σ(1) σ(2) · · · σ(q)
σ= .
1 2 ··· q

进一步, 我们得规定置换群里面的元素作用于张量之后到底发生了哪些改
变, 一个自然而然的想法是把自变量中的序号做一次置换 (因为我们希望一个对
换作用于 f 之后恰好是与对称张量和反对称张量有关的东西.)

定义 4.6. 记 φ(q) 为 q 阶置换群, 对任意的 σ ∈ φ(q), 定义 (σf )(v1 , v2 , · · · , vq ) :=


f (vσ(1) , vσ(2) , · · · , vσ(q) ).

命题 4.3. f 是 q 阶对称张量 ⇐⇒ σ(f ) = f, ∀ σ ∈ φ(q).


f 是 q 反对称张量  ⇐⇒ σ(f ) = sign(σ)f, ∀ σ ∈ φ(q).
−1, σ为奇置换,
其中 sign(σ) :=
1, σ为偶置换.

证明. 利用抽象代数中的关于置换群的结论: 任意一个置换可以表为若干对换的


乘积即可证明.

由上述命题, 我们立即得到如下结论.

命题 4.4. 设 V 是 n 维向量空间,δi 是 V 中的一组基,δ i 是 V ⋆ 是中的一组对偶基.


对任意 q 阶协变张量 f , 令 fi1 ···iq = f (δi1 , δi2 , · · · , δiq ), 其中 1 ≤ i1 , i2 , · · · , iq ≤
n,f = fi1 ···iq δ i1 ⊗ · · · ⊗ δ iq , 则
f 是对称的 ⇐⇒ fσ(i1 )···σ(iq ) = fi1 ···iq , ∀ σ ∈ φ(q).
f 是反对称的 ⇐⇒ fσ(i1 )···σ(iq ) = sign(σ)fi1 ···iq , ∀ σ ∈ φ(q).

当定义了一个新的概念时, 按照惯例, 我们会问这些新的概念的全体所构成


的空间是否具有某种比较好的性质, 这便是下面的引理.

引理 4.1. 设 Λq (V ⋆ ) 为 q 阶反对称张量所构成的全体, 则 Λq (V ⋆ ) 是一线性空


间, 并且是 Vq0 的线性子空间.

证明. 对任意的 f1 , f2 , f ∈ Λq (V ⋆ ), λ ∈ R. 因 σ(fi ) = sign(σ)fi , 于是定义线性


结构如下:

σ(f1 + f2 )(v1 , v2 , · · · , vq ) := (f1 + f2 )(vσ(1) , vσ(2) , · · · , vσ(q) ).

47
4.3. 外代数 CHAPTER 4. 多重线性函数

σ(λf ) := λσ(f ).

进一步

σ(f1 + f2 )(v1 , v2 , · · · , vq ) = (f1 + f2 )(vσ(1) , vσ(2) , · · · , vσ(q) )


= f1 (vσ(1) , vσ(2) , · · · , vσ(q) ) + f2 (vσ(1) , vσ(2) , · · · , vσ(q) )
= σ(f1 )(v1 , v2 , · · · , vq ) + σ(f2 )(v1 , v2 , · · · , vq )
= sign(σ)(f1 + f2 )(v1 , v2 , · · · , vq ).

这说明 σ(f1 + f2 ) = sign(σ)(f1 + f2 ), 即 f1 + f2 ∈ Λq (V ⋆ ). 同理可证 λf ∈


Λq (V ⋆ ). 这说明 Λq (V ⋆ ) 是一线性空间. 显然 Λq (V ⋆ ) ⊆ Vq0 , 并且具有相同的线
性结构, 于是 Λq (V ⋆ ) 是 Vq0 的线性子空间.

定义 4.7. 称 Λq (V ⋆ ) 为 q 次外矢量 (形式) 空间, 称 Λq (V ⋆ ) 的元素为 q 次外矢


量 (形式).

既然 Λq (V ⋆ ) 是一线性空间, 一个很自然的问题是, 该线性空间的基和维数?


我们知道外形式本质上是张量的反称化, 自然我们会考虑如何从任意的 q 重线
性函数出发构造 q 次外形式. 也就是如何从一般的 q 重线性函数出发构造一个
反称化的 q 重线性函数, 问题归结于: 如何构造一个普遍的反称化算子, 使得每
一个多重线性函数都变成一个反称的多重线性函数, 先观察 q = 2, 3 的时候的情
况.

例 4.8. 二重/三重线性函数的反称化
q = 2 时, 显 然 α(x, y) = 1
2 (f (x, y) − f (y, x)) 是 f (x, y) 的 反 称 化, 即
α(x, y) = −α(y, x). q = 3 时, 注意到 β(x, y, z) = 1
3! (f (x, y, z) + f (y, z, x) +
f (z, x, y) − f (y, x, z) − f (x, z, y) − f (z, y, x)) 是一反对称函数. 例如 β(y, x, z) =
1
3! (f (y, x, z)+f (x, z, y)+f (z, y, x)−f (x, y, z)−f (y, z, x)−f (z, x, y)) = − 3!
1
(f (x, y, z)+
f (y, z, x) + f (z, x, y) − f (y, x, z) − f (x, z, y) − f (z, y, x)) = −β(x, y, z).

注意到上述反称化运算的结果都与 φ(q) 和 sign(σ) 有关. 这启发我们引入


下面的定义.

定义 4.8. 定义 q 阶反称化算子
1 ∑
Aq := sign(σ)σ.
q!
σ∈φ(q)

于是 Aq (f ) = 1
q! σ∈φ(q) sign(σ)σ(f ), ∀ f ∈ Vq0 .

引入了上面的定义后, 接下来的问题是, 是否所有的 q 次外形式都是某个 q


重线性函数的反称化? 这便是下面的命题.

48
CHAPTER 4. 多重线性函数 4.3. 外代数

命题 4.5. Λq (V ⋆ ) = Aq (Vq0 ).

证明. 对任意的 f ∈ Λq (V ⋆ ), 注意到

1 ∑
Aq (f ) = sign(σ)σ(f )
q!
σ∈φ(q)
1 ∑
= sign(σ)sign(σ)(f )
q!
σ∈φ(q)
1 ∑
= (f )
q!
σ∈φ(q)

= f.

即存在 f ∈ Vq0 , 使得 f = Aq (f ). 这说明 Λq (V ⋆ ) ⊆ Aq (Vq0 ).


另一方面, 对任意的 f ∈ Aq (Vq0 ), 则存在 g ∈ Vq0 , 使得 f = Aq (g). 对于
τ ∈ φ(q), 进一步有

1 ∑
τ (f ) = τ ( sign(σ)σ(g))
q!
σ∈φ(q)
1 ∑
= sign(σ)τ σ(g)
q!
σ∈φ(q)
1 ∑
= sign(τ )sign(τ )sign(σ)τ σ(g)
q!
σ∈φ(q)
1 ∑
= sign(τ ) sign(τ )sign(σ)τ σ(g)
q!
σ∈φ(q)
1 ∑
= sign(τ ) sign(τ σ)τ σ(g)
q!
σ∈φ(q)

= sign(τ )Aq (g)


= sign(τ )f

(上述第三个等号是因为 sign2 (τ ) = 1(这是因为 sgin(τ ) = ±1),上述倒数第二


个等号值得思考一下, 求和的本质是让每一项都相加而不是要其形式是一样的,
比喻来讲, 如同两个人在操场跑步一样, 判断两个人跑的里程是否相等不需要同
时出发同时到达终点, 只需要你跑过的每一步我都跑过, 我走过的每一步你也走
过. 那么我们两个的里程数必然是相等的, 这里的求和也是, 一个求和里面的每
一项另外一个里面都有, 另外一个求和里面的每一项也在前一个之中, 那么它们
求和必然是相等的.). 这说明 f ∈ Λq (V ⋆ ). 即 Aq (Vq0 ) ⊆ Λq (V ⋆ ).
综上有 Λq (V ⋆ ) = Aq (Vq0 ).

49
4.3. 外代数 CHAPTER 4. 多重线性函数

定 义 4.9. 设 α ∈ Λr (V ⋆ ), β ∈ Λs (V ⋆ ). 定 义 α 与 β 的 外 积 为 α ∧ β =
(r+s)!
r!s! Ar+s (α ⊗ β). 容 易 看 出, 外 积 ∧ 是 Λr (V ⋆ ) × Λs (V ⋆ ) 7→ Λr+s (V ⋆ ) 的
映射.

外积可以看作是张量积的反称化. 自然会问它具有一些什么样的性质, 哪些
是与张量积相同的, 哪些是自身所独有的.

性质 4.2. 设 α, α1 , α2 ∈ Λr (V ⋆ ), β, β1 , β2 ∈ Λs (V ⋆ ), γ ∈ Λt (V ⋆ ). 有下面三条性
质成立.
1) (α1 + α2 ) ∧ β = α1 ∧ β + α2 ∧ β.

α ∧ (β1 + β2 ) = α ∧ β1 + α + ∧β2 .
2) α ∧ β = (−1)rs β ∧ α.
3) α ∧ (β ∧ γ) = (α ∧ β) ∧ γ.

证明. 1)

(r + s)!
(α1 + α2 ) ∧ β = Ar+s ((α1 + α2 ) ⊗ β)
r!s!
(r + s)!
= Ar+s (α1 ⊗ β + α2 ⊗ β)
r!s!
(r + s)! (r + s)!
= Ar+s (α1 ⊗ β) + Ar+s (α2 ⊗ β)
r!s! r!s!
= α1 ∧ β + α2 ∧ β.

同理可证 α ∧ (β1 + β2 ) = α ∧ β1 + α + ∧β2 .


2) 令

( )
s+1 r+2 ··· s+r 1 2 ··· s
σ= .
1 2 ··· r r+1 r+2 ··· r+s

注意到 sign(σ) = (−1)rs (这是因为把第一排的 1 挪动到 s + 1 这个位置需要 s


次对换, 一直这么下去 2 挪到 s + 2,· · · , 直到 s 挪到 s + r, 一共要 r 次, 每次需
要 s 个对换.) 因为 α ∧ β 是反称的, 根据命题 4.3 有

σ(α ∧ β) = sign(σ)(α ∧ β) = (−1)rs (α ∧ β).

50
CHAPTER 4. 多重线性函数 4.3. 外代数

上式两边同时乘以 (−1)rs , 有

(α ∧ β) = (−1)rs σ(α ∧ β)
(s + t)!
= (−1)rs σ( Ar+s (α ⊗ β))
s!t!
(s + t)!
= (−1)rs σ(Ar+s (α ⊗ β))
s!t! (4)
(s + t)! ∑
= (−1)rs σ( (sign(τ )τ (α ⊗ β)))
s!t!
τ ∈φ(s+t)

(s + t)! ∑
= (−1)rs ( (sign(τ )στ (α ⊗ β))).
s!t!
τ ∈φ(s+t)

注意到

στ (α ⊗ β)(v1 , · · · , vr , vr+1 , · · · , vr+s ) = τ (α ⊗ β)(vσ(1) , · · · , vσ(r) , vσ(r+1) , · · · , vσ(r+s) )


= τ (α ⊗ β)(vs+1 , · · · , vs+r , v1 , · · · , vs )
= (α ⊗ β)(vτ (s+1) , · · · , vτ (s+r) , vτ (1) , · · · , vτ (s) )
= α(vτ (s+1) , · · · , vτ (s+r) )β(vτ (1) , · · · , vτ (s) )
= β(vτ (1) , · · · , vτ (s) )α(vτ (s+1) , · · · , vτ (s+r) )
= (β ⊗ α)(vτ (1) , · · · , vτ (s) , vτ (s+1) , · · · , vτ (s+r) )
= τ (β ⊗ α)(v1 , · · · , vs , vs+1 , · · · , vs+r ).

这表明 στ (α ⊗ β) = τ (β ⊗ α). 带入 (4) 式有


(s + t)! ∑
(α ∧ β) = (−1)rs ( (sign(τ )στ (α ⊗ β)))
s!t!
τ ∈φ(s+t)

(s + t)! ∑
= (−1)rs ( (sign(τ )τ (β ⊗ α)
s!t!
τ ∈φ(s+t)

(s + t)!
= (−1)rs Ar+s (β ⊗ α)
s!t!
= (−1)rs (β ∧ α).

3) 令
1 ∑ 1 ∑
ar = sign(σ)σ, Aq = sign(τ )τ.
r! q!
σ∈φ(r) τ ∈φ(q)

其中 r ≤ r < q, 令
( )
σ(1) σ(2) ··· σ(r) r+1 r+2 ··· r+s
σ= .
1 2 ··· r r+1 r+2 ··· r+s

51
4.3. 外代数 CHAPTER 4. 多重线性函数

显然 φ(r) ⊆ φ(q), 进一步我们断言 Aq ◦ ar = Aq . 事实上,

1 ∑ 1 ∑
Aq ◦ ar = sign(τ )τ ◦ sign(σ)σ
q! r!
τ ∈φ(q) σ∈φ(r)
1 ∑ ∑
= sign(σ)sign(τ )τ σ
q!r!
σ∈φ(r) τ ∈φ(q)
1 ∑ ∑
= sign(τ σ)τ σ
q!r!
σ∈φ(r) τ ∈φ(q)
1 ∑ ∑
= sign(γ)γ
q!r!
σ∈φ(r) γ∈φ(q)
1 ∑
= sign(γ)γ
q!
γ∈φ(q)

= Aq .

(这里的几个等号与命题 4.5 同理) 这就证明了断言. 进一步

(r + s + t)!
(α ∧ β) ∧ γ = Ar+s+t ((α ∧ β) ⊗ γ)
(r + s)!t!
(r + s + t)! (r + s)!
= Ar+s+t (( )(Ar+s (α ⊗ β)) ⊗ γ)
(r + s)!t! r!s!
(r + s + t)!
= (Ar+s+t Ar+s (α ⊗ β)) ⊗ γ))
r!s!t!
(r + s + t)!
= Ar+s+t ((α ⊗ β) ⊗ γ)
r!s!t!
(r + s + t)!
= Ar+s+t (α ⊗ (β ⊗ γ))
r!s!t!
(r + s + t)!
= Ar+s+t As+t (α ⊗ (β ⊗ γ))
r!s!t!
(r + s + t)! (s + t)!
= Ar+s+t As+t (α ⊗ (β ⊗ γ))
r!(s + t)! s!t!
(r + s + t)! (s + t)!
= Ar+s+t (α ⊗ ( )As+t (β ⊗ γ))
r!(s + t)! s!t!
(r + s + t)!
= Ar+s+t (α ⊗ (β ∧ γ))
r!(s + t)!
= α ∧ (β ∧ γ)

有了以上的这些性质, 我们就可以试着对之前的那个问题进行一定的分析
了, 即 Λq (V ⋆ ) 的基和维数问题, 因为 Λq (V ⋆ ) 是 Vq0 的子空间, 设 ei 是 V ⋆ 的

52
CHAPTER 4. 多重线性函数 4.4. 外形式空间

{ }
一组基, 则 Vq0 的基是 ei1 ⊗ ei2 ⊗ · · · ⊗ eiq . 注意到反称化的张量有下面这两
个性质, 即 α ∧ β = (−1)rs β ∧ α, 这说明选取 Λq (V ⋆ ) 的基的时候,∧ 的顺序并
不重要 (如果顺序不同, 但它们仍然在相差一个正负号的时候可以相互表出,
仍然是线性相关的). 这启发我们该空间的基与组合数有关. 并且 α ∧ α = 0,
同时我们知道在反对称二重线性函数构成的全体空间的维数等于反对称矩阵 ( )
n(n−1) n
的全体的维数等于 2 . 显然该维数等于 Cn2 = . 于是我们可以猜测
2
( )
n { }
Λq (V ⋆ ) 的维数为 Cnq = . 进一步 Λq (V ⋆ ) 的基为 ei1 ⊗ ei2 ⊗ · · · ⊗ eiq ,
q
其中 1 ≤ i1 < i2 < · · · < iq ≤ n.

4.4 外形式空间
2019-10-09
本节要点/问题提示:
1)、利用反称化算子写出 r 个一阶张量作外积得到的结果?
2)、求值公式? 求值公式的证明要点是什么?
3)、外形式空间的基和维数? 如何证明? 证明的要点是什么?
4)、能否写出常见外形式空间的基?
5)、行列式的本质是什么 (利用外积的语言)?
6)、低阶张量为零的充分必要条件? 以二阶和三阶为例, 由此推广到高阶?
7)、利用外积的语言重新描述向量的线性相关和线性无关?
8)、Cartan 引理的内容? 如何证明? 证明的要点是什么?
9)、能否独立的证明老师布置的那两个习题? 如果不能, 尽量看完之后独立
做几遍?

例 4.9. 行列式的新形式
a1 a21 ··· an1
1
1
a2 a22 ··· an2

在高等代数中, 一个方阵的行列式记为 det(aj ) = .
i
.. .. .. . 不妨
.. . . .

1
an a2n · · · ann
记 det = f : V × V · · · × V →7 R. 显然 f 是满足如下两条性质的:
| {z }
n
1) f (α1 , α2 , · · · , αi +β i , · · · , αn ) = f (α1 , α2 , · · · , αi , · · · , αn )+f (α1 , α2 , · · · , β i , · · · , αn ).
2) f (α1 , α2 , · · · , λαi , · · · , αn ) = λf (α1 , α2 , · · · , αi , · · · , αn ). 并且进一步,

53
4.4. 外形式空间 CHAPTER 4. 多重线性函数

可用置换的形式表示行列式, 由行列式的定义有
∑ σ(1)
det(aij ) = (−1)sign(σ) a1 · · · aσ(n)
n
σ∈φ(n)
1 ∑ ∑ i1 ···in i1
= δ a · · · aijnn
n! j ···j i ···i j1 ···jn j1
1 n 1 n



−1, i1 · · · in 为排列,j1 · · · jn 为i1 · · · in 的奇排列,


其中 δji11 ···i
···jn
n
= 1, i1 · · · in 为排列,j1 · · · jn 为i1 · · · in 的偶排列,



0, 其他.

有了外积, 我们自然会思考外积作用在一组向量上面得到的是什么. 这便是


下面的求值公式, 在证明这个命题之前, 我们先证明一个引理.

引 理 4.2. 设 δ 1 , δ 2 , · · · , δ r 为 V ⋆ 中 的 元 素 中 的 元 素, 则 δ 1 ∧ δ 2 · · · ∧ δ r =
r!Ar (δ 1 ⊗ δ 2 · · · ⊗ δ r ).

证明. 用数学归纳法. 当 r = 2 的时候, 注意到 δ i ∈ V ⋆ = L (V, R), 这说明 δ i


(1+1)!
为一阶张量 (一重线性代数), 由外积的定义,δ 1 ∧ δ 2 = 1!1! A1+1 (δ
1
⊗ δ2 ) =
2!A2 (δ 1 ⊗ δ 2 ). 假设 r − 1 的时候, 引理成立. 注意到 δ 1 ∧ δ · · · ∧ δ 2 r−1
是 r−1
阶张量, 于是

(r − 1 + 1)!
δ 1 ∧ δ 2 · · · ∧ δ r−1 ∧ δ r = Ar−1+1 ((δ 1 ∧ δ 2 · · · ∧ δ r−1 ) ⊗ δ r )
(r − 1)!1!
(r − 1 + 1)!
= Ar−1+1 (((r − 1)!Ar−1 (δ 1 ⊗ δ 2 · · · ⊗ δ r−1 )) ⊗ δ r )
(r − 1)!1!
(r − 1 + 1)!
= (r − 1)!Ar−1+1 (Ar−1 (δ 1 ⊗ δ 2 · · · ⊗ δ r−1 )) ⊗ δ r )
(r − 1)!1!
= r!Ar−1+1 (Ar−1 (δ 1 ⊗ δ 2 · · · ⊗ δ r−1 )) ⊗ δ r )
= r!Ar (Ar−1 (δ 1 ⊗ δ 2 · · · ⊗ δ r−1 )) ⊗ δ r )
= r!Ar (Ar−1 (δ 1 ⊗ δ 2 · · · ⊗ δ r−1 )) ⊗ δ r )
= r!Ar (δ 1 ⊗ δ 2 · · · ⊗ δ r−1 ⊗ δ r ).

这就证明了引理.

注解 4.14. 当时老师出错的第一步就在这里的系数问题上 (老师所著的《拓扑


与流形》也在这里出了问题), 当然这里引理的出发点在于证明外积具有结合律
(r+s+t)!
的时候得到的表达式 (α ∧ β) ∧ γ = r!s!t! Ar+s+t (α ⊗ (β ⊗ γ)). 另外一个关键
点是 δ i ∈ V ⋆ , 根据 V ⋆ 的定义,V ⋆ 是 V 上的所有 1 重线性函数的全体所构成的

54
CHAPTER 4. 多重线性函数 4.4. 外形式空间

空间, 即 V ⋆ = L(V, R), 即 δ 1 是一阶张量, 注意到这个细节之后其他的计算都


是显然的. 很多时候我们会忽略的往往是这种细节 (δ i 是一阶张量), 本质上说是
我们对基本的定义和概念不熟悉. 值得一提的是, 上课的时候老师往往会写错一
些东西, 选择去思考老师的板书而不是去“相信”, 带着一种质疑和批判的精神
去听课.

注解 4.15. 上述引理老师上课的时候并没有写到, 这里写出来是为了更好的去证


明下面的求值公式. 把细节梳理清楚.

命题 4.6. (求值公式) 设 δ 1 , δ 2 , · · · , δ n 为 V ⋆ 中的一组基, 对任意的 v1 , v2 , · · · , vq , 1 ≤


δ i1 (v ) δ i1 (v ) · · · δ i1 (v )
1 2 q
i
δ 2 (v1 ) δ i2 (v2 ) · · · δ i2 (vq )

i ≤ n, 有 (δ ∧ δ · · · ∧ δ )(v1 , v2 , · · · , vq ) = .
i1 i2 iq
.. .. .. .
.. . . .

iq
δ (v1 ) δ iq (v2 ) · · · δ iq (vq )

证明. 由上述引理 (引理 4.2),

(δ i1 ∧ δ i2 · · · ∧ δ iq )(v1 , v2 , · · · , vq ) = q!Aq (δ i1 ⊗ δ i2 · · · ⊗ δ iq )(v1 , v2 , · · · , vq )


1 ∑
= q! sign(σ)σ(δ i1 ⊗ δ i2 · · · ⊗ δ iq )(v1 , v2 , · · · , vq )
q!
σ∈φ(q)

= sign(σ)σ(δ i1 ⊗ δ i2 · · · ⊗ δ iq )(v1 , v2 , · · · , vq )
σ∈φ(q)

= sign(σ)(δ i1 ⊗ δ i2 · · · ⊗ δ iq )(vσ1 , vσ2 , · · · , vσq )
σ∈φ(q)

= sign(σ)δ i1 (vσ1 )δ i2 (vσ2 ) · · · δ iq (vσq )
σ∈φ(q)

δ i1 (v ) δ i1 (v2 ) ··· δ i1 (vq )
1
i
δ 2 (v1 ) δ i2 (v2 ) ··· δ i2 (vq )

= . .. .. .. .
.. . . .

iq
δ (v1 ) δ iq (v2 ) ··· δ (vq )
iq

55
4.4. 外形式空间 CHAPTER 4. 多重线性函数

注解 4.16. 在上述证明中, 特别地, 取 δ 1 ∧ δ 2 · · · ∧ δ n (δ1 , δ2 , · · · , δn ) = 1, 考虑



δ i1 (δ ) δ i1 (δ ) · · · δ i1 (δ )
j1 j2 jq
i
δ 2 (δj1 ) δ i2 (δj2 ) · · · δ i2 (δjq )

δ ∧ δ · · · δ (δj1 , δj2 , · · · , δjq ) = .
i1 i2 iq
.. .. ..
.. . . .

iq
δ (δj1 ) δ iq (δj2 ) · · · δ iq (δiq )

δ i1 δ i1 · · · δ i1
j1 j2 jq
i2
δj δji2 · · · δji2
1 q
= . ..
2

.. .. ..
. . .
iq
δj δj · · · δiiq
iq
1 2 q

i i ···i
= δj11 j22 ···jqq

定理 4.2. 设 V 是一个线性空间,δi 为 V 的一组基,V ⋆ 为其对偶空间,δ i 为其对


偶基,dimΛq (V ⋆ ) = Cnq .

证明. 对任意的 α ∈ Λq , 由于 α 是 q 阶张量, 于是 (使用了爱因斯坦求和约定)

α = α(δi1 , δi2 , · · · , δiq )δ i1 ⊗ δ i2 ⊗ · · · ⊗ δ iq = αi1 i2 ···iq δ i1 ⊗ δ i2 ⊗ · · · ⊗ δ iq .

因 α 是反称的, 于是

α = Aq (α)
= Aq (αi1 i2 ···iq δ i1 ⊗ δ i2 ⊗ · · · ⊗ δ iq )
1
= αi i ···i δ i1 ∧ δ i2 ∧ · · · ∧ δ iq
q! 1 2 q

= αi1 i2 ···iq δ i1 ∧ δ i2 ∧ · · · ∧ δ iq .
1≤i1 <i2 <···<iq ≤n

这里需要说明的是, 最后一个等式的成立性, 先看一个简单的例子.n = 2, q = 2



1 1
ai i δ i1 ∧ δ i2 = (a11 δ 1 ∧ δ 2 + a12 δ 1 ∧ δ 2 + a22 δ 2 ∧ δ 2 )
2! 1 2 2!
1
= (a12 δ 1 ∧ δ 2 + a21 δ 2 ∧ δ 1 )
2!
1
= (a12 δ 1 ∧ δ 2 − a21 δ 1 ∧ δ 2 )
2!
= a12 δ 1 ∧ δ 2 .
从这里我们可以看出当 i1 i2 · · · iq 不是一个排列的时候,δ i1 ∧ δ i2 ∧ · · · ∧ δ iq 必然
为 0,ai1 i2 ···iq = 0. 当 i1 i2 · · · iq 是一个排列的时候, 我们知道一共有 q! 种排列方

56
CHAPTER 4. 多重线性函数 4.4. 外形式空间

式, 如果我们相对于一种固定的大小关系 i1 < i2 < · · · < iq , 那么无论一个排列,


每一次对换导致了两个符号, 一个是 ai1 i2 ···iq 这里出现的负号, 一个是外积运算
δ i1 ∧ δ i2 ∧ · · · ∧ δ iq 这里出现的负号. 因而任何一次对换所得到的结果都等于按
照上面大小顺序所得到的结果 αi1 i2 ···iq δ i1 ∧ δ i2 ∧ · · · ∧ δ iq . 而我们最终需要的和
式是所有的满足 i1 < i2 < · · · < iq 的全部相加, 这便是之前的最后一个等式成
立性的解释 (一句话讲因为它的反称性).
下面说明 δ i1 ∧ δ i2 ∧ · · · ∧ δ iq 是线性无关的. 不妨设存在 ki1 i2 ···iq 使得

ki1 i2 ···iq δ i1 ∧ δ i2 ∧ · · · ∧ δ iq = 0.

它作为零函数作用于元素 (ej1 , ej2 , · · · , ejq ) 上的结果必然为零, 其中 1 ≤ j1 <


j2 < · · · < jq ≤ n.

ki1 i2 ···iq δ i1 ∧ δ i2 ∧ · · · ∧ δ iq (ej1 , ej2 , · · · , ejq ) = 0.

在根据注解 4.16 的结果, 我们有

i i ···i
ki1 i2 ···iq δj11 j22 ···jqq = 0.

i i ···i
因 i, j 都是按照一定大小关系的限制, 加之 δj11 j22 ···jqq 的性质, 我们有

kj1 j2 ···jq = 0.

这就证明了 δ i1 ∧ δ i2 ∧ · · · ∧ δ iq 是一组基, 对应的,Λq (V ⋆ ) 的维数为 Cnq .

例 4.10. 常见外形式空间的基
Λ0 (V ⋆ ) 的基:0.
Λ1 (V ⋆ ) 的基:δ 1 , δ 2 , · · · , δ n .
Λ2 (V ⋆ ) 的基:δ 1 ∧ δ 2 , · · · , δ 1 ∧ δ n , · · · , δ 2 ∧ δ n , · · · , δ n−1 ∧ δ n .
Λ3 (V ⋆ ) 的基:δ 1 ∧ δ 2 ∧ δ 3 , · · · , δ 1 ∧ δ 2 ∧ δ n , · · · , δ 1 ∧ δ n−1 ∧ δ n , · · · , δ n−2 ∧
δ n−1 ∧ δ n .
..
.
Λn−1 (V ⋆ ) 的基:δ 1 ∧ δ 2 ∧ · · · ∧ δ n−1 , · · · , δ 2 ∧ δ 3 ∧ · · · ∧ δ n .
Λn (V ⋆ ) 的基:δ 1 ∧ δ 2 ∧ · · · ∧ δ n .

例 4.11. 行列式的本质
设 V 是一 n 维线性空间,δi 为 V 的一组基,V ⋆ 为其对偶空间, 对偶基为 δ i ,

57
4.4. 外形式空间 CHAPTER 4. 多重线性函数

对任意的 A1 , A2 , · · · , An ∈ V, 设 Ai = aji δj = δ j (Ai )δj . 定义函数



a1 a2 · · · an
1 1 1
1
a2 a22 · · · an2

f (A1 , A2 , · · · , An ) = . .. .. ..
.. . . .

1
an a2n · · · ann

δ 1 (A ) δ 2 (A ) · · · δ n (A )
1 1 1
1
δ (A2 ) δ 2 (A2 ) · · · δ n (A2 )

= . .. .. ..
.. . . .

1
δ (An ) δ (An ) · · · δ (An )
2 n


= sign(σ)δ 1 (Aσ(1) )δ 2 (Aσ(2) ) · · · δ n (Aσ(n) )
σ∈φ(n)

= sign(σ)(δ 1 ⊗ δ 2 · · · ⊗ δ n )(Aσ(1) , Aσ(2) , · · · , Aσ(n) )
σ∈φ(n)

= sign(σ)σ(δ 1 ⊗ δ 2 · · · ⊗ δ n )(A1 , A2 , · · · , An )
σ∈φ(n)
1 ∑
= n! sign(σ)σ(δ 1 ⊗ δ 2 · · · ⊗ δ n )(A1 , A2 , · · · , An )
n!
σ∈φ(n)

= n!An (δ 1 ⊗ δ 2 · · · ⊗ δ n )(A1 , A2 , · · · , An ).
由引理 4.2 知,δ 1 ∧δ 2 · · ·∧δ n (A1 , A2 , · · · , An ) = n!An (δ 1 ⊗δ 2 · · ·⊗δ n )(A1 , A2 , · · · , An ).
于是 f (A1 , A2 , · · · , An ) = δ 1 ∧δ 2 · · ·∧δ n (A1 , A2 , · · · , An ), 即 f = δ 1 ∧δ 2 · · ·∧δ n .

问题 4.4. 在基变换 δi⋆ = aij δi , δ i⋆ = bij δ j , 问 δ 1⋆ ∧ δ 2⋆ ∧ · · · ∧ δ n⋆ = Bδ 1 ∧ δ 2 ∧


· · · ∧ δ n . 中 B =?

证明.
δ 1⋆ ∧ δ 2⋆ ∧ · · · ∧ δ n⋆ = (b1j1 δ j1 ) ∧ (b2j2 δ j2 ) ∧ · · · (bnjn δ jn )
= b1j1 b2j2 · · · bnjn δ j1 ∧ δ j2 · · · ∧ δ jn

= b1j1 b2j2 · · · bnjn δ j1 ∧ δ j2 · · · ∧ δ jn
j1 ,j2,··· ,jn 为1,2,··· ,n的一个排列

= (−1)sign(σ) b1j1 b2j2 · · · bnjn δ 1 ∧ δ 2 · · · ∧ δ n
σ∈φ(n)

= (−1)sign(σ) b1σ(1) b2σ(2) · · · bnσ(n) δ 1 ∧ δ 2 · · · ∧ δ n
σ∈φ(n)

= det(bij )δ 1 ∧ δ 2 · · · ∧ δ n

58
CHAPTER 4. 多重线性函数 4.4. 外形式空间

于是有 det(bij ) = B.

例 4.12. (二阶反对称张量为零的条件) 已知 X ∈ Λ2 (V ⋆ ), 其中 X = aij δ i ∧ δ j =


0, 试考虑 aij 的性质.

证明. 显然

X= aij δ i ∧ δ j
i,j
∑ ∑
= aij δ i ∧ δ j + aij δ i ∧ δ j
i̸=j i=j

= aij δ i ∧ δ j
i̸=j
∑ ∑
= aij δ i ∧ δ j + aij δ i ∧ δ j
i<j i>j
∑ ∑
= aij δ i ∧ δ j + aji δ j ∧ δ i
i<j j>i
∑ ∑
= aij δ i ∧ δ j − aji δ i ∧ δ j
i<j j>i

= (aij − aji )δ i ∧ δ j
i<j

而 δ ∧ δ (i < j) 为一组基. 于是 aij = aji .


i j

例 4.13. (三阶反对称张量为零的条件) 已知 X ∈ Λ3 (V ⋆ ), 其中 X = aijk δ i ∧


δ j ∧ δ k = 0, 试考虑 aijk 的性质.

证明. 取任意的 (δl , δm , δn ),X 作用之, 有

0 = aijk δ i ∧ δj ∧ δj (δl , δk , δm )
ijk
= aijk δlmn

i j
δl δl δlk

= aijk δmi j
δm k
δm
i
δn δnj δnk

= aijk (δli (δm δn − δnj δm


j k k
) − δlj (δm δn − δni δm
i k k
) + δlk (δm δn − δni δm
i j j
))
= almn − alnm − amln + anlm + amnl − anml

定理 4.3. 设 ξ 1 , ξ 2 , · · · , ξ r 为 V 中的 r 个向量, 则 ξ 1 , ξ 2 , · · · , ξ r 线性无关的充


分必要条件是 ξ 1 ∧ ξ 2 ∧ · · · ∧ ξ r = 0.

59
4.4. 外形式空间 CHAPTER 4. 多重线性函数

证明. 必要性 =⇒,(反证) 若 ξ 1 , ξ 2 , · · · , ξ r 线性相关, 不妨设

ξ r = k1 ξ 1 + · · · + kr−1 ξ r−1 .

进一步有

ξ 1 ∧ ξ 2 ∧ · · · ∧ ξ r = ξ 1 ∧ ξ 2 ∧ · · · ∧ (k1 ξ 1 + · · · + kr−1 ξ r−1 )


= (−1)l1 k1 ξ 1 ∧ ξ 1 · · · ∧ ξ r−1 + · · · + (−1)lk−1 kr−1 ξ 1 ∧ · · · ξ r−1 ∧ ξ r−1
= 0.

充 分 性 ⇐=,(反 证) 设 ξ 1 ∧ · · · ∧ ξ r = 0, 若 ξ 1 , · · · , ξ r 线 性 无 关. 将
ξ 1 , · · · , ξ r 扩 充 为 V 中 的 一 组 基 ξ 1 , · · · , ξ r , ξ r+1 , · · · , ξ n . 进 一 步 取 对 偶 基
ξ1 , · · · , ξr , ξr+1 , · · · , ξn . 则

0 = 0 ∧ ξ r+1 ∧ · · · ∧ ξ n
= ξ 1 ∧ · · · ∧ ξ r ∧ ξ r+1 ∧ · · · ∧ ξ n (ξ1 , · · · , ξr , ξr+1 , · · · , ξn )
= 1.

矛盾. 这就证明了定理.

注解 4.17. 事实上, 在我们知道行列式的本质是外积之后, 就可以得出这个定理,


因为行列式不等于零等价于线性无关的的, 而行列式不等于零又等价于外积不
等于零

引理 4.3. (Cartan 引理) 设 α1 , α2 , · · · , αr , β1 , β2 , · · · , βr 是 n 为线性空间 V 上


∑r
的 2r 个 1− 形式, 其中 α1 , α2 , · · · , αr 是线性无关的. 若恒等式 i=1 αi ∧βi = 0,
∑r
则 βi , 1 ≤ i ≤ r 可表为 α1 , α2 , · · · , αr 的线性组合的形式, 即 βi = j=1 cij αj ,
其中 cij = cji .

证明. 注意到 α1 , α2 , · · · , αr 是线性无关的, 将其扩充为 V 中的一组基

α1 , α2 , · · · , αr , αr+1 , · · · , αn .

于是 βi 可被上述基线性表出. 设
∑ ∑
βi = cij αj + cij αj .
j≤r r+1≤j≤n

60
CHAPTER 4. 多重线性函数 4.4. 外形式空间

将上式带入引理给定的恒等式中有


r
0= α i ∧ βi
i=1

r ∑ ∑
= αi ∧ ( cij αj + cij αj )
i=1 j≤r r+1≤j≤n

r ∑ ∑
r ∑
= αi ∧ cij αj + αi ∧ cij αj
i=1 j≤r i=1 r+1≤j≤n

r ∑
r ∑
r ∑
= cij αi ∧ αj + cij αi ∧ αj
i=1 j=1 i=1 r+1≤j≤n
∑ ∑r ∑
= (cij − cji )αi ∧ α + j
cij αi ∧ αj
i<j≤r i=1 r+1≤j≤n

另一方面, 我们知道 αi ∧ αj (i < j ≤ r),αi ∧ αj (i ≤ r < j) 为 Λ2 (V ⋆ ) 中基的一


部分. 因而是线性无关的. 进一步有 cij − cji = 0(i < j ≤ r), cij = 0(i ≤ r < j).

即 cij = cji , βi = j≤r cij αj . 这就完成了引理的证明.

习题 4.1. 给定 n 个外形式 η i ∈ Λ2 (V ⋆ ), 1 ≤ i ≤ n. 设 δ 1 , δ 2 , · · · , δ n 为 V ⋆ 的一
组基, 则方程组
{ ∑n
ηi = j=1 δ ∧ wj
j i

j
wji + wi = 0

证明:wji 存在且唯一 (第二个式子不求和, 没有采用求和).

证明. 令 wji = xijk δ k , η i = aijk δ j ∧ δ k , 因 η i ∈ Λ2 (V ⋆ ), 即二阶反对称张量, 进一


步有 aijk + aijk = 0. 另一方面,η i 可由基表出, 于是


ηi = (bijk δ j ∧ δ k ).
j<k

61
4.4. 外形式空间 CHAPTER 4. 多重线性函数

显然有

ηi = aijk δ j ∧ δ k
j,k
∑ ∑
= aijk δ j ∧ δ k + aijk δ j ∧ δ k
j̸=k j=k

= aijk δ j ∧ δ k
j̸=k
∑ ∑
= aijk δ j ∧ δ k + aijk δ j ∧ δ k
j<k j>k
∑ ∑
= aijk δ j ∧ δ k + aikj δ k ∧ δ j
j<k k>j

= (aijk − aikj )δ j ∧ δ k
j<k

这说明有 bijk = aijk − aikj . 带入第一个方程



ηi = bijk δ j ∧ δ k
j<k

= (xijk δ j ∧ δ k )
j,k
∑ ∑
= (xijk δ j ∧ δ k ) + (xijk δ j ∧ δ k )
j̸=k j=k

= (xijk δ j ∧ δ k )
j̸=k
∑ ∑
= (xijk δ j ∧ δ k ) + (xijk δ j ∧ δ k )
j<k j>k

= ((xijk − xikj )δ j ∧ δ k ))
j<k

于是有
bijk = xijk − xikj (5)

显然 i, j, k 的指标地位是对称的, 轮换一次有 (即 i 变成 j,j 变成 k,k 变成 i)

bjki = xjki − xjik (6)

再轮换一次 (即对 (6) 式在做一起相同的操作)

bkij = xkij − xkji (7)

62
CHAPTER 4. 多重线性函数 4.4. 外形式空间

注意由第二个方程有 xijk + xjik = 0, 考虑 (7) + (6) − (5) 有

xjik + xkij + xikj − xjik − xkji − xijk = xkij + xikj − xjik − xijk + xjik − xkji
= 0 + 0 + 2xjki
= bjki + bkij − bijk

于是 xjki = 21 (bjki + bkij − bijk ). 这说明 xijk 完全由 bijk 决定, 而 bijk 由 η i 完全决
定. 并且是唯一的. 这就说明了方程所确定的 wji 是存在唯一的.

习题 4.2. (分解定理) 设 ξ 1 , ξ 2 , · · · , ξ r ∈ Λ1 (V ⋆ ) 线性无关,Ω ∈ Λp (V ⋆ ). 则存在


p − 1 次外形式 φ1 , φ2 , · · · , φr ∈ Λp−1 (V ⋆ ) 使得 Ω = φ1 ∧ ξ 1 + φ2 ∧ ξ 2 + · · · +
∑r
φr ∧ ξ r = α=1 φα ∧ ξ α 的充分必要条件是 ξ 1 ∧ ξ 2 ∧ · · · ∧ ξ r ∧ Ω = 0.

证明. 因 ξ 1 , ξ 2 , · · · , ξ r 线性无关, 将其扩充为 V ⋆ 中的一组基 ξ 1 , ξ 2 , · · · , ξ r , ξ r+1 , · · · , ξ n


若 p + r > n, 则 ξ 1 ∧ ξ 2 ∧ · · · ∧ ξ r ∧ Ω 是 p + r > n 次外形式, 但我们知道任意
一个大于 n 外形式都为零, 所以 ξ 1 ∧ ξ 2 ∧ · · · ∧ ξ r ∧ Ω = 0 自动成立, 显然 Ω 可
以被 Λp (V ⋆ ) 中的一组基表出.


Ω= Ωi1 i2 ···ip ξ i1 ∧ ξ i2 · · · ∧ ξ ip
1≤i1 <i2 <···<ip ≤n

进一步有

∑ ∑
Ω= Ωi1 i2 ···ip ξ i1 ∧ ξ i2 · · · ∧ ξ ip + Ωi1 i2 ···ip ξ i1 ∧ ξ i2 · · · ∧ ξ ip
1≤i1 ≤n−p+1<i2 <···<ip ≤n n−p+2≤i1 <i2 <···<ip ≤n

注意到第二个和式需要 p 个不同的数, 但从 n − p + 2 到 n 只有 n − 1 个不同的


数, 这说明第二个的和式为零, 于是


Ω= Ωi1 i2 ···ip ξ i1 ∧ ξ i2 · · · ∧ ξ ip
1≤i1 ≤n−p+1<i2 <···<ip ≤n

注意到 p + r > n, 于是 r > p − n, 因为 r 为整数, 于是 r ≥ n − p + 1, 我们定义


∑
i1 <i2 <···<ip ≤n Ωi1 i2 ···ip ξ · · · ∧ ξ , i1 ≤ n − p + 1, 对 i1 不求和.
i2 ip
φ i1 = 则
0, n − p + 1 < i1 ≤ r.

63
4.4. 外形式空间 CHAPTER 4. 多重线性函数

Ω 的表达式可化简为


Ω= Ωi1 i2 ···ip ξ i1 ∧ ξ i2 · · · ∧ ξ ip
1≤i1 ≤n−p+1<i2 <···<ip ≤n

= ξ i1 ∧ Ωi1 i2 ···ip ξ i2 · · · ∧ ξ ip
1≤i1 ≤n−p+1<i2 <···<ip ≤n

= ξ i1 ∧ φi1
1≤i1 ≤n−p+1

= ξ i1 ∧ φi1
1≤i1 ≤r

= (−1)r φi1 ∧ ξ i1 .
1≤i1 ≤r

这说明在 p + r > n 条件和结论都自动成立.

若 p + r ≤ n, 显然有 r ≤ n − p, 于是 r < n − p + 1.

必要性是显然的, 因为


r
ξ1 ∧ ξ2 ∧ · · · ∧ ξr ∧ Ω = ξ1 ∧ ξ2 ∧ · · · ∧ ξr ∧ ( φα ∧ ξ α )
α=1

r
= ξ1 ∧ ξ2 ∧ · · · ∧ ξr ∧ ξα
α=1
∑r
= (−1)lαi ξ 1 ∧ ξ 2 ∧ · · · ∧ ξ α ∧ ξ α ∧ · · · ∧ ξ r
α=1

= 0.


下证明充分性. 同理 Ω 可被一组基表出, 定义 φi1 = i1 <i2 <···<ip ≤n Ωi1 i2 ···ip ξ i2 · · ·∧
ξ ip . 进一步


Ω= Ωi1 i2 ···ip ξ i1 ∧ ξ i2 · · · ∧ ξ ip
1≤i1 <i2 <···<ip ≤n
∑ ∑
= Ωi1 i2 ···ip ξ i1 ∧ ξ i2 · · · ∧ ξ ip + Ωi1 i2 ···ip ξ i1 ∧ ξ i2 · · · ∧ ξ ip
1≤i1 ≤r<i2 <···<ip ≤n r+1≤i1 <i2 <···<ip ≤n
∑ ∑
= ξ i1 ∧ φ i1 + Ωi1 i2 ···ip ξ i1 ∧ ξ i2 · · · ∧ ξ ip
1≤i1 ≤r r+1≤i1 <i2 <···<ip ≤n

64
CHAPTER 4. 多重线性函数 4.4. 外形式空间

将上式带入 ξ 1 ∧ ξ 2 ∧ · · · ∧ ξ r ∧ Ω = 0. 有
∑ ∑
0 = ξ1 ∧ ξ2 ∧ · · · ∧ ξr ∧ ( ξ i1 ∧ φi1 + Ωi1 i2 ···ip ξ i1 ∧ ξ i2 · · · ∧ ξ ip )
1≤i1 ≤r r+1≤i1 <i2 <···<ip ≤n

= 0 + ξ1 ∧ ξ2 ∧ · · · ∧ ξr ∧ Ωi1 i2 ···ip ξ i1 ∧ ξ i2 · · · ∧ ξ ip
r+1≤i1 <i2 <···<ip ≤n

= Ωi1 i2 ···ip ξ 1 ∧ ξ 2 ∧ · · · ∧ ξ r ∧ ξ i1 ∧ ξ i2 · · · ∧ ξ ip .
r+1≤i1 <i2 <···<ip ≤n

显然当 r + 1 ≤ i1 < i2 < · · · < ip ≤ n 时,ξ 1 ∧ ξ 2 ∧ · · · ∧ ξ r ∧ ξ i1 ∧ ξ i2 · · · ∧ ξ ip 是


线性无关的, 自然有 Ωi1 i2 ···ip = 0. 带入到 Ω 的表达式即有

Ω= ξ i1 ∧ φi1 .
1≤i1 ≤r

这就证明了分解定理.

65
4.4. 外形式空间 CHAPTER 4. 多重线性函数

66
第 5 章 微分流形初步

本章从流形的定义开始讲起, 先复习了拓扑空间的基本概念和基本性质, 研
究了单位圆周上的可微函数与闭区间上的可微函数的不同, 由此引出了拓扑流
形的定义, 但我们在拓扑流形上只能讨论与连续有关的性质. 我们想要把欧式空
间中可微函数的概念搬到拓扑流形上面去 (把欧式空间中开集/邻域的概念一般
化便是拓扑 (拓扑结构)), 这就引出了微分结构. 有了微分的定义就自然有了微
分流形的概念. 简单来说微分流形就是一片一片欧式空间粘接起来得到的, 换言
之就是局部上看和欧式空间是一样的. 在这个意义上讲, 微分流形是欧式空间的
推广. 黎曼的最初想法, 流形是三维空间中曲面曲线的高维推广. 至于流形定义
的三条 (开覆盖、相容性、极大性), 比喻来讲, 流形就像一条鱼. 鱼鳞就是开覆
盖, 鱼鳞与鱼鳞之间的相交的部分看起来很光滑就是相容性, 极大性就是任意一
条鱼鳞如果与其他相交鱼鳞相容, 那么这片鱼鳞本身就在这条鱼上. 接下来介绍
了一些常见的流形的例子及其流形结构. 简单来说流形作为集合就像一个承载
的对象, 流形结构就是如何承载这个对象. 如同一个杯子, 流形本身就是杯子的
背景空间, 而流形结构就是杯子的装水的那个结构. 回顾了切向量和古典微分几
何的内蕴思想, 从内蕴和外围空间的角度解释了为什么我们需要用泛函的观点
去看切向量, 进一步说明了用割线的极限定义的切向量的局限性, 由此用公理化
的方式利用泛函定义了一个切向量. 对于流形上的映射而言, 用分析的最基本的
思想就是线性化 (微分), 我们加以推广就得到切映射, 即一个映射在一点处的线
性化, 这是因为线性的映射是简单的, 性质是比较好的, 在分析学中我们总希望
拿好的映射去逼近一般的映射.

5.1 微分流形
2019-10-16
本节要点/问题提示:

67
5.1. 微分流形 CHAPTER 5. 微分流形初步

1)、什么是外形式? 外形式空间的基和维数? 外形式空间的元素表示形式?


2)、圆上的可微函数与区间上的光滑函数有何不同?
3)、为什么圆 S 1 不能同胚于 R1 及其子区间?
4)、微分几何的基本思想和方法?
5)、从数学分析中的连续函数的定义过渡到拓扑中连续函数的定义?
6)、从邻域/开集的性质过渡到拓扑结构的定义? 什么是拓扑空间?
7)、 什 么 是 拓 扑 空 间 的 第 二 可 数 性? 什 么 是 拓 扑 空 间 的 T2 性 质 (即
Hausdorf f ) 空间的定义?
8)、什么是拓扑流形? 能不能举个拓扑流形的例子?
9)、什么是微分结构? 微分流形的定义? 为什么要这么定义? 好处和目的何
在?
10)、流形定义的数学思想和哲学思想? 结构一词的字面意思与布尔巴基学
派?
11)、为什么说流形是个整体的概念? 你所知道的数学上整体的概念还有哪
些? 体会一下数学中整体与局部的对立关系?
回顾: 外形式空间.
设 V 为 n 维线性空间,δ1 , δ2 , · · · , δn 为 V 中的一组基,V ⋆ 为其对偶空间,
相应的对偶基为 δ 1 , δ 2 , · · · , δ n . 考虑 V 上的 r 次外形式空间 Λr (V ⋆ ), 则对任意
的 α ∈ Λr (V ⋆ ), 有规范式
1
α= αi i ···i δ i1 ∧ δ i2 ∧ · · · ∧ δ ir .
r! 1 2 r
进一步化为标准式

α= αi1 i2 ···ir δ i1 ∧ δ i2 ∧ · · · ∧ δ ir .
i1 <i2 <···<ip

对偶地, 对任意的 ξ ∈ Λr (V ), 有规范式


1 1
ξ= δ ∧ δ 2 ∧ · · · ∧ δ r (ξ)δ1 ∧ δ2 ∧ · · · ∧ δr .
r!
进一步化为标准式

ξ= δ 1 ∧ δ 2 ∧ · · · ∧ δ r (ξ)δ1 ∧ δ2 ∧ · · · ∧ δr .
i1 <i2 <···<ip

特别地, 取 x1 , x2 , · · · , xr ∈ V, 则 x1 ∧ x2 ∧ · · · ∧ xr ∈ Λr (V ), 于是有
1 1
ξ= δ ∧ δ 2 ∧ · · · ∧ δ r (x1 , x2 , · · · , xr )δ1 ∧ δ2 ∧ · · · ∧ δr
r!

i1 i2 ···ir
= ξ12···r δ1 ∧ δ2 ∧ · · · ∧ δr
i1 <i2 <···<ip

68
CHAPTER 5. 微分流形初步 5.1. 微分流形

∑n ∑n
设上述 xi = j=1 aji δj = δ j (xi )δj , 利用求值公式, 我们得到
j=1

δ i1 (x ) δ i2 (x ) · · · δ ir (x )
1 1 1
i
δ 1 (x2 ) δ i2 (x2 ) · · · δ ir (x2 )

δ ∧ δ ∧ · · · ∧ δ (x1 , x2 , · · · , xr ) = .
i1 i2 ir
.. .. ..
.. . . .

i1
δ (xr ) δ (xr ) · · · δ (xr )
i 2 i r


ai1 ai2 · · · air
1 1 1
i1
a2 ai22 · · · ai2r

= . .. .. .. .
.. . . .

i1
ar air2 · · · airr

进一步

ai1 ai12 ··· ai1r
1
i1
∑ a2 ai22 ··· ai2r

x1 ∧ x2 ∧ · · · ∧ xr = . .. .. ..
.
1≤i1 <i2 <···<ir ≤n . . . .
i1
ar air2 ··· airr

于是

x1 ∧ · · · ∧ xr = 0 ⇐⇒ x1 , x2 , · · · , xr 线性相关

ai1 ai2 · · · ai1r
1 1
i1
a2 ai22 · · · ai2r

⇐⇒ . .. .. .. = 0, 对任意的1 ≤ i1 < i2 < · · · < ir ≤ n.
.. . . .

i1
ar air2 · · · airr
 
a1 a21 · · · ar1 · · · an1
 11 
a2 a22 · · · ar2 · · · an2 
 
⇐⇒ 矩阵  . .. .. ..  的秩 < r
 .. . . . 
 
a1r ar · · · ar · · · ar
2 r n

⇐⇒ 秩的定义: 最大阶非零子式

这便从外积的角度给出了高等代数中关于矩阵的秩的一种新的理解方式.

注解 5.1. 关于单位圆定义域的说明, 我们在本科和中学阶段, 学过的用三角函


数表示单位圆 (cos(x), sin(x)), 0 ≤ x ≤ 2π, 这样写的定义域会导致一个问题, 即
x = 0 和 x = 2π 所对应的两点是重合的. 都是表示起点的位置 (因为圆是首
尾相接的, 起点就是终点). 这就导致了一个问题, 定义在整个圆上面的函数在

69
5.1. 微分流形 CHAPTER 5. 微分流形初步

x = 0 和 x = 2π 上的两点的函数值应该是一样的. 这直接导致了下面例子中现
象的出现.

例 5.1. 设 f : S 7→ R,0 ≤ x ≤ 2π. 若 f 是定义在 S 上面的可微函数, 且


f ′′ (x) = 0, 试问 f (x) 的性质与定义闭区间上的可微函数性质有何不同?

证明. 若仅仅是定义在 [0, 2π], 由于 f ′′ (x) = 0, 进一步 f (x) = ax + b, 考虑到


该函数是定义在圆上的函数, 在圆上 x = 0, x = 2π 是表示的同一点, 必然有
f (0) = f (2π), 于是 b = 2aπ + b, 则 a = 0, 进而 f (x) = b = const. 而定义在闭
区间上面的可微函数形如 f (x) = ax + b.

注解 5.2. 上面的例子相对于数学分析而言是不可思议的, 定义在圆上的可微函


数与闭区间上的可微函数竟然有如此大的差异, 同样的, 可以考虑定义在球面上
的可微函数与定义在二维闭区域上的可微函数有何不同?之所以这么考虑, 因
为圆是我们最常见的封闭曲线, 自然会考虑在封闭曲面的时候会是怎样的结果.
推广到高维的时候又会是怎样的结果?为了弄清楚这些看上去再也基本不过的
问题, 我们还得再来研究研究单位圆.

注解 5.3. 为了更好的说明拓扑流形和微分流形的概念, 我们不得不在此多说两


句, 先谈谈流形这个词的由来, 在现存的资料中, 流形这个词取自于老一辈数学
家的惊为天人的翻译能力, 也许是老一辈数学家的文学素养比较强, 这个词取自
于文天祥的《正气歌》:
天地有正气, 杂然赋流形.
上则为星辰, 下则为河岳.
语文徘徊在及格边缘的我尝试解释一下这个词, 流形这个词在《正气歌》中
的大概指天地万物, 各种形状的自然造物 (这个词与哲学中的形而上学的来源是
相似的, 形而上学来源于形而上者之谓道, 形而下则之谓器). 在日语翻译中, 他
们把流形称之为多面形. 这个本身的意思指的就是各种形状的事物.
下面谈谈几何的研究方法与基本工具, 首先几何研究的是空间形式与性质
的科学. 几何学的兴起与笛卡尔有莫大的关系, 笛卡尔横空出世用坐标系的方法
将几何问题解析化和代数化, 建立了本科阶段学过的解析几何. 主要的思想是建
立坐标系, 将几何问题转化为代数问题, 如果将几何问题转化为分析/方程问题,
这便是微分几何. 古典微分几何的主要思想就是在坐标系下用映射的值域 (图
像) 表示曲面和曲线, 即我们的研究问题是曲面和曲线的基本性质, 而曲面和曲
线可以用一个映射的图像表示. 所以问题转化为研究这些映射的基本性质, 我们
在数学分析中研究映射的基本思想是微积分, 自然我们会想到用微积分的思想
去研究映射的基本性质, 从而得到曲面与曲线的基本性质, 这便是古典微分几何

70
CHAPTER 5. 微分流形初步 5.1. 微分流形

的出发点:一条曲线的切线和微分是同一个概念, 同样. 一条封闭曲线所围成的


面积就是积分论, 微积分在几何上的应用便演变成曲线论和曲面论.
但这么做有一个重大的前提, 即是否所有的曲面和曲线都可以用一个映射
的图像表示出来. 也就是我们想要的第一步转化是否能够实现? 对于每一条曲线
而言, 是否一定存在一个映射, 使得该映射的图像恰好是我们给定的曲线? 如果
存在, 那么这种映射唯一吗? 如果不存在, 我们又应该如何去解决这个问题? 对
于曲面而言同样有这个问题.
这时候先考虑我们生活中最常见的曲线: 圆. 上面的问题转化为, 是否存在
一个定义在 R 映射, 使得该映射的图像刚好是一个圆. 在这里大家可能有疑惑
的是, 这种映射不应该就是中学阶段的 (r cos θ, r sin θ)? 但是这里所谓的映射还
有一些其他条件的限制, 这是来源于拓扑学的一些条件. 在拓扑学中, 我们认为
两个拓扑空间是等价的当且仅当它们是同胚的. 我们知道拓扑流形的维数 n 是
因为在拓扑流形上每一点的邻域都与 Rn (或者 Rn 为开集) 同胚. 回到圆上, 我
们知道圆上任意一个点的邻域都与 R1 中的开区间同胚. 于是最终的问题转化
为, 是否存在一个同胚映射, 使得该映射的图像是个圆, 并且该映射定义在 R1
上, 之所谓会这么问, 是因为在局部上看圆 (即看圆上每一个点的邻域), 它总是
与开区间同胚的. 自然就会问在整体的情况下它是怎样的. 进一步解释整体与局
部的对偶观点, 局部意味着从曲线/曲面等的邻域去看, 整体意味着一整段曲线
和一整块曲面的角度上去看, 我们自然希望能够整体的去看曲线和曲面.
该问题进一步可以转化, 因为问题是是否存在同胚映射, 等价于圆是否与
R 本身或者其开区间同胚. 从拓扑性质上讲,S 1 是紧致 (连通) 的,R1 和 R1 中
1

的非闭区间是非紧的, 它们自然不能与 S 1 同胚, 而且由于同胚保持连通性, 如果


有与 R1 上与 S 1 同胚的区间, 则只能是闭区间. 如果是闭区间, 我们知道连通区
域个数是一个拓扑不变量. 圆周去掉任何一个点后剩下一个连通区域, 但闭区间
去掉一个点 (不去掉端点) 剩下两个连通区域. 这说明 S 1 可能不与任何区间同
胚. 但我们知道 S 1 是一维的, 却不能整体的找到一个 R1 上的区间与之同胚, 同
样的, 考虑二维曲面 S 2 , 也可以证明不存在平面上的一个区域与其同胚. 自然我
们会问, 一个整体坐标是不是要求过于苛刻了, 于是我们只需要一个局部的开覆
盖, 并在这种局部开覆盖上面定义坐标映射 (与 Rn 同胚的那个同胚映射), 我们
想对这种高维的” 曲线曲面” 有着清楚的了解. 但同时我们知道, 坐标本身是没
有什么意义的, 对于流形而言更重要的是它的形状及与形状伴随而来的性质. 第
一个问题得到了解决, 但一个问题的解决往往是另外一个问题的开始, 因为局部
的开覆盖会带来一个问题, 如果两个开邻域相交, 那么相交区域对应的坐标函数
是否要满足某种性质呢? 第一反应我们会想到坐标要相同或者” 相似”, 前面刚
说坐标是没有什么实际的意义的, 之所以要利用坐标是为了把这种东西给表达

71
5.1. 微分流形 CHAPTER 5. 微分流形初步

清楚, 但我们本身又不能被坐标所限制, 我们需要接纳坐标但又要超越坐标. 如


果是相同, 直观感觉上未免也太苛刻了, 我们希望能够整体的去看曲面和曲线,
但在这么做遇到问题问题的时候, 自然希望能够先用局部的观点去看待这个东
西, 希望能够得到一些好的结果. 并且似乎隐含着坐标 (同胚映射) 本身是有意
义的这种背后的观点. 类似地, 还是考虑 S 1 上两个不同的局部坐标覆盖, 显然可
以看出这里不需要相交区域的坐标映射是完全相同的, 但需要满足它们之间可
以相互转化 (直观上我们可以说坐标映射” 相似”). 另外, 我们知道曲线如果有”
结点” 的时候, 其可微性是被破坏了的, 在” 结点” 处是不可微的, 微分几何微分
几何, 顾名思义, 微分是必须的技术手段, 上面坐标映射” 相似” 的要求也是为了
防止曲线曲面自交的情况. 按照这种思路下去, 流形的定义就很自然了.
流形本身大数学家 Riemann 在推广 R3 中的微分几何到 Rn 中的微分几何
时候引出的一种自然地概念, 直到 W eil 在他的黎曼面的著作中才提出了现在的
流形的概念. 简单的说流形就是曲线曲面在 n 维欧式空间的推广, 因为流形 (曲
面/曲线) 不能找到一个整体的坐标函数 (同胚映射), 于是用局部的开覆盖去覆
盖整体的流形 (曲面曲线), 并且伴随每一个开覆盖有一个坐标函数 (同胚映射),
因为这些开覆盖是会相交的, 所以在相交邻域上的坐标函数 (同胚映射) 应该会
满足一些性质, 又因为坐标本身是没有意义的, 只需要其坐标能够相互转化即可.
流形的定义是一个很有意思的东西, 其精髓在于用任意的东西去定义一个
唯一的东西, 用变化的观点去看不变的东西. 如同我们可以给高等代数中的线
性变换下一个定义: 设 Ai 是一族矩阵, 并且对任意的 i ̸= j, 存在非退化矩阵
Ai , Aj 使得 Aj = P Ai P −1 . 这一族矩阵定义了一个线性变换. 这就是用变化的
观点去定义不变的东西, 因为线性变化本身是与基的选取无关的, 无论你选不选
取基, 线性变换本身就在那里. 但线性变换在基下的矩阵却随着基的变化而变
化, 并且在不同基下的矩阵是相似的. 如同高中的立体几何, 一个立体几何体本
身与坐标的选取没有任何关系, 无论你选不选取坐标系, 立体几何体本身就在那
里, 不会发生任何改变. 但立体几何体上每一个点的坐标却随着你坐标系的选取
的改变而改变. 在这种不变与变化中, 利用变化的东西去定义不变的东西, 本来
就是一种非常精妙的思维, 体现了一种相当的哲学高度. 流形的开覆盖是任意的,
该覆盖上的同胚映射需要满足在相交邻域上转化映射光滑即可. 开覆盖和同胚
映射本身都是变化的, 但是这些变化的东西却定义了一个不变的几何体, 这便是
流形.

在引入流形的定义之前, 我们再来研究单位圆作为流形的例子. 该例子是一


个非常好的产生直观理解的例子. 因为流形本身并不是一个特别神秘的东西, 简
单说它就是曲面曲线在高维空间的推广, 在老一辈数学眼中的天地万物, 数学家
眼中局部上与欧式空间同胚的东西. 自然圆作为曲线, 肯定也能体现流形的一些

72
CHAPTER 5. 微分流形初步 5.1. 微分流形

特性.

例 5.2. 如图所示 , 记 U = S\(1, 0),V = S\(−1, 0), 令 φ : U 7→ (0, 2π) ⊆ R1 ,

图 5.1: 单位圆作为流形示意图

ψ : V 7→ (π, 3π) ⊆ R1 . 其 中 φ(p) = θ, ψ(p) = α. 于 是 在 U ∩ V =


θ + 2π, θ ∈ (0, π),
S\(1, 0) ∪ (−1, 0), α = 再考 虑 S 1 上 的 可 微 函 数 f ,
θ, θ ∈ (π, 2π).
通过之前的分析我们知道 S 1 不与任何 R1 上的区间同胚, 但 S 1 \(1, 0) 却与
(0, 2π) 同胚, 同样的,S 1 \(−1, 0) 也与 (π, 3π) 同胚. 而一条曲线上面的可微函数
我们却没有过清晰的定义. 所以, 我们的想法是, 把一条曲线局部的同胚到 R1
上, 转化为 R1 , 我们所熟悉的东西上面. 进一步说, 因为 f 是定义在 S 1 上的, 而
且有同胚映射 φ, 考虑到同胚映射及其逆映射均连续, 所以 f ◦ φ−1 就转化成了
定义在 (0, 2π) = φ(U ) 的可微函数, 这就完成了我们的转化, 我们无法直接在曲
线上面定义可微函数, 但我们通过同胚映射就可以转化为定义在 R1 上的可微函
数.
令 g = f ◦ φ−1 , h = f ◦ ψ −1 , 若 g 在 (0, 2π) = φ(U ) 上可微,h 在 (π, 3π) =
ψ(V ) 可微, 我们就称 f 在 S 1 上可微, 之所以这么说, 其中一个原因是 U ∪ V =
S1.
按照这种想法考虑例 5.1 的变形, 设 g ′′ (θ) = 0, θ ∈ (0, 2π), h′′ (α) = 0, 于是
g(θ) = aθ + b, h(α) = cα + d, 若 φ−1 (θ) = ψ −1 (α) = p ∈ U ∩ V, α ∈ (π, 3π), 则
g = f ◦ φ−1 (θ) = h = f ◦ ψ −1 (α), 进而有
{
aθ + b = cα + d = cθ + d, θ ∈ (π, 2π), α ∈ (π, 2π)
aθ + b = cα + d = c(θ + 2π) + d, θ ∈ (0, π), α ∈ (2π.3π)

这说明 a = c = 0, b = d.

注解 5.4. 上述例子的数学思想是非常重要且常见的, 把没有精确定义的概念转


化到我们已经熟悉的概念上, 这也是流形这个概念的思想, 从某种角度上讲, 流

73
5.1. 微分流形 CHAPTER 5. 微分流形初步

形不过是一片片同胚于欧式空间的东西按照一定规律粘接而成的东西. 因为我
们无法精确刻画流形, 所以我们就用欧式空间为模型构造出了流形的概念.

微分流形是拓扑流形附加了一个可微结构, 在定义微分流形之前, 我们补充


一点点拓扑的基础知识.
拓扑是一门很有意思的学问, 一门类似几何研究图形的科学, 数学中看上去
一些很简单很直观的一些问题, 刚刚提到拓扑是研究图形的科学, 对此, 数学家
们不得不提一个非常简单的问题: 什么是图形? 正是由于对这种问题的研究而促
使数学家们去思考, 首先我们研究的大部分图形都是连续的, 也就是说图形一般
是不会跳跃的, 但是连续在数学分析里面的确有定义, 我们又应该如何在一个集
合上定义连续呢? 一个集合上附加一个什么东西我们就可以认为它是一个图形
呢? 进而研究他的一些基本性质?
仔细分析一下这个问题, 我们首先回顾连续的概念 (一元函数连续的概念):
对任意的 ϵ > 0, 存在 δ > 0, 使得当 |x − x0 | < δ 时, 有 |f (x) − f (x0 )| < ϵ.
即对任意的 ϵ > 0, 存在 δ > 0, 使得当 x ∈ (x0 − δ, x0 + δ) 时, 有 f (x) ∈
(f (x0 ) − ϵ, f (x0 ) + ϵ). 这样我们便可以从邻域的角度刻画连续, 即对任意的一个
包含 f (x0 ) 的邻域 V , 存在一个包含 x0 的邻域 U , 使得 f (U ) ⊆ V. 这说明如果
要讨论连续在集合上的定义, 首先要讨论邻域在集合上面的定义.
再次回顾邻域的性质: 因为邻域本质上一种开集, 所以开集具有的性质它应
该也具有. 即任意多个开集的并是开集, 有限多个开集的交是开集. 规定空集和
全集是开集也是闭集, 于是空集和全集是开集. 这三条性质抽象出来便是拓扑结
构的定义. 有了拓扑结构的基础之上我们可以讨论连续和图形相关的性质.

定义 5.1. 设 X 是一集合,τ 是一子集族, 如果 τ 是满足如下性质:


1)∅, X ∈ τ.
2) 任意多个 τ 中元素的并属于 τ .
3) 有限多个 τ 中元素的交属于 τ .
称 τ 为 X 的一个拓扑 (拓扑结构),τ 中的元素称为开集. 偶对 (X, τ ) 称为
拓扑空间.

为了引出拓扑流形的概念, 我们还需要做一点点拓扑的铺垫. 分别是分离性


公理和可数性公理. 关于第二可数性公理, 说明它是什么之前我们先谈谈拓扑空
间的基, 拓扑空间的基和线性空间的基定义是不一样的. 但更为深刻的本质是一
样的.
首先我们知道泛函分析中的度量空间是拓扑空间的一种, 而在度量空间中
球形邻域起着基本建筑块的作用. 球形邻域的全体构成的子集族就可作为度量
空间的拓扑. 并且我们知道若 U 是度量空间中的开集, 则对任意的 x ∈ U, 存在

74
CHAPTER 5. 微分流形初步 5.1. 微分流形

一个球形邻域 B(x, ϵ) ⊆ U, 因此 U = ∪x∈U B(x, ϵ), 通过这个例子我们可以看到


一个集合是度量空间的开集当且仅当它是一个度量空间的若干球形邻域之并.
因此我们可以说度量空间的拓扑是由它所有球形邻域关于求并运算所生成的.
这启发我们定义如下拓扑空间基的概念.

定义 5.2. 设 (X, τ ) 是一拓扑空间,σ 是 τ 的一个子族, 如果 τ 中的每一个元素


(即拓扑空间的每一个开集) 是 σ 中的某些元素的并. 即对于每一个 U ∈ τ, 存在
σ1 ∈ τ, 使得 U = ∪B∈σ1 B. 则称 σ 是拓扑 τ 的一个基. 或者称 σ 为拓扑空间 X
的一组基.

定义 5.3. 如果一个拓扑空间 X 存在一可数拓扑基. 则称 X 为满足第二可数性


公理的拓扑空间.

注解 5.5. 简单对之前那句话做个解释. 为什么说拓扑空间的基和线性空间的基


本质是一样的. 我们知道在线性空间它有线性结构, 线性结构简单来说就是定义
了加法和数乘. 所以线性空间的基只需要一组通过加法和数乘可以表出任何元
素的元素即可. 但我们又知道在拓扑空间里面只有拓扑结构, 还没有其他结构.
所以它只能做集合的基本运算. 于是拓扑基的定义为通过集合的并运算可以表
出任何元素即可. 所谓的基, 就是按照其空间结构定义的运算能够表出任意元素
的最小组合者.

下面介绍分离性公理, 最常见的 Hausdorf f 空间. 它旨在说明点和邻域的


关系.

定义 5.4. 设 X 是一拓扑空间, 如果对任意的 x, y ∈ X,x ̸= y. 存在 x 的邻域 U ,


以及 y 的邻域 V, 使得 U ∩ V = ∅, 则称 X 为 Hausdorf f 空间.

有了上述拓扑的铺垫和之前关于流形的一些直观介绍, 现在我们可以给出
拓扑流形的定义了.

定义 5.5. 如果一个拓扑空间 X 满足一下条件:


1)X 是 Hausdorf f 空间.
2)X 满足第二可数性公理.
3) 对 X 中的任意点 x, 都存在 x 的一个邻域 U ⊆ X, 使得 U 同胚 (设同胚
映射为 φU ) 于 Rn 中的开集 (或者 Rn 本身). 则称 X 是一个 n 维拓扑流形.

注解 5.6. 在上述定义中, 我们知道 φU (U ) 是 Rn 中的开集. 则对任意的 y ∈ U,


可以把 φU (y) 的坐标看作是 y 的局部坐标. 即令 y i = (φU (y))i ,y ∈ U, i =
1, 2, · · · , n. 通常我们称 (U, φU ) 为坐标卡 (坐标图/坐标卡图). 顾名思义, 坐标
卡像地图册一样, 地图册把每一个地方用图册上的一些点的位置来表示, 坐标卡

75
5.1. 微分流形 CHAPTER 5. 微分流形初步

把流形上的每一个点用坐标域 φU (U ) 中的坐标表示. 比喻来讲, 我们有成语字


如其人, 每个人写字可以看作是一个同胚映射, 把每一个人映射为他的笔迹/字.
看到这个字就可以代表这个人. 坐标卡便是字和人对应放在一起的集合.
当然, 这样定义的拓扑流形自然就有当 U ∩ V ̸= ∅ 时, 有 φV ◦ φ−1
U 在
φU (U ∩ V ) 上连续,φU ◦ φ−1
V 在 φV (U ∩ V ) 上连续. 这就是老师在黑板上写的拓
扑流形的定义. 但事实上, 我们国内外大部分教材都是采用上面的定义而不是采
用老师在黑板上的定义 (因为老师写的三个条件, 实际上可以由第一个和第二个
条件推出第三个条件).

图 5.2: 拓扑流形之间的转化映射示意图

之前已经说过, 在拓扑空间上可以讨论连续有关的性质, 有了连续, 下一个


自然的概念便是微分/可微, 如果要在拓扑流形上定义可微函数, 又需要附加怎
样的条件?
上面的注记还存在那么一些问题, 如果两个邻域 U ∩ V ̸= ∅, 即那么相交
区域的点便有了两个坐标, 这两个坐标需要满足某些转化关系吗? 回答了这个
问题, 我们便可以更进一步的解决如何在拓扑流形定义可微函数的问题. 为了解
决两个坐标之前是否需要满足某种转化关系这个问题. 即考虑 φU ◦ φ−1
V 在集合
φV (U ∩ U ) 是否需要满足某些性质. 显然 φU ◦ φ−1
V 是 φV (U ∩ U ) 到 φU (U ∩ U )
的同胚. 但同胚的要求仅为映射及其逆映射均连续, 一个很自然的想法, 如果把
条件加强为映射和逆映射均为 C r 映射, 那么我们是否能在流形上定义可微函数
了呢? 从这个角度去思考, 启发我们如下定义.

定 义 5.6. 设 M n 为 为 一 n 维 拓 扑 流 形, 其 坐 标 卡 集 (流 形 结 构) 为 S =
−1
{(Uα , φα ) | α ∈ A} , 若 φβ ◦ φ−1
α : φα (Uα ∩ Uβ ) 7→ φβ (Uα ∩ Uβ ) 和 φα ◦ φβ :
φβ (Uα ∩ Uβ ) 7→ φα (Uα ∩ Uβ ) 均为 C r 映射. 则称 (M n , S) 为一个 C r 的微分结
构. 称 (M n , S) 为 C k 微分流形. 上述性质称为 (Uα , φα ) 和 (Vβ , φβ ) 为 C r 相
容.

76
CHAPTER 5. 微分流形初步 5.1. 微分流形

下面的微分流形的定义是更为标准的定义. 可以证明, 他们两个证明是等价


的.
{ }
定义 5.7. 设 M 是一个 n 维拓扑流形, 令 Σ0 为坐标卡集 Σ0 = (Uα , φα ) : (Uα , φα )是M 的坐标卡, α ∈ I ,
如果 Σ0 满足以下条件 (即称 Σ0 是一微分结构), 则称 M 是一 C r 微分流形:
1)(开覆盖性)Uα , α ∈ I 是 M 的一开覆盖, 即 M = ∪α∈I Uα .
2)(C r 相容性)Σ0 中任意两个元素都是 C r 相容的.
3)(极大性) 对任意 M 的坐标卡 (Uβ , φβ ), 如果 (Uβ , φβ ) 与任意的 Σ0 中的
元素都是 C r 相关, 则 (Uβ , φβ ) ∈ Σ0 . 换言之,Σ0 是极大的.

值得一提的是, 可以证明, 上述的 (1)、(2) 可以推出 (3). 所以在考虑一个拓


扑流形是否为微分流形 (光滑流形) 的时候, 只需要找到那些坐标卡和判断转化
映射是否 C r (光滑) 即可.

注解 5.7. 谈微分结构之前, 先谈谈三大数学结构与布尔巴基学派, 数学从最开始


大地测量和等价交换发展至今, 首先是数字的发明, 我们都很熟悉一二三四五六
七, 但有时候却说不清楚什么是一, 各位想想, 抽象的”1” 到底在哪儿?首先”1”
是一个具体的数, 但也是一个抽象的概念. 除了我们的数字”1” 和汉字“一”以
外我们似乎也不能说清楚什么是”1”, 我们是从 1 个苹果,1 个梨子,1 头牛,1 只
鸟. 从这些具体的东西, 抽出那个共同的数字特征, 就是”1” 表征着这种数量的大
小, 用久了就习惯了这种数字了, 但是要真正说清楚什么是”1”, 我想这并不是一
件简单的事情.
虽然我们说不清楚”1” 到底是个什么东西, 但是我们已经把数字用着” 出神
入化” 了. 我们忽略了一个伟大的发现:一个抽象的概念在人们的日常生活中诞
生了: 数字. 这是数学史上一个伟大的进步. 随着人们生产生活的发展, 数学也相
应随之发展, 当然也包括大地测量学的发展, 当时人们已经认知到了这么一件事
情, 任何一条过圆心的直线段把圆的面积平均分成两部分, 但在当时的数学资料
当中却不是这么写的, 它们都是以例子的形式出现, 而不是一个定理. 当时都是
说以 1 为半径的圆, 过它的直径把这个圆的面积平均分成两部分 (或者把半径
换成其他数字). 但这些都是例子, 人们还处于观察状态, 还没有进入反思和总结
的状态. 这时候有一位数学家在数学资料上写下了数学意义下的第一个定理: 任
何一条过圆心的直径把圆的面积平均分成两部分. 这也是伟大的突破. 数学家们
终于有了定理了. 上面这些分析, 旨在说明从具体到抽象这一条路就是数学的发
展之路. 这也是我写这本小册子的主要原因: 我想和大家一起在从具体走到抽象,
又从抽象走会到具体来.
之所以想走这一条路, 也是因为自己在刚开始学习本课数学专业基础知识
的时候的一点点感悟, 具体和抽象之间来来回回好几次也许才是理解数学的正

77
5.1. 微分流形 CHAPTER 5. 微分流形初步

确道路. 另一方面, 所谓的布尔巴基学派, 也就是结构数学的代表学派. 由于第三


次数学危机的影响, 他们提倡使用公理化定义, 也就是铺面而来的公理、定义、
定理、推论等. 没有任何启发性, 不带有任何废话的逻辑严密绝对正确的教材.
他们的观点中, 数学有三大基本结构, 分别是代数结构、拓扑结构、序结构. 简
单来说代数结构是” 加法” 的推广, 拓扑结构是” 邻域” 的推广, 序结构是” 大于
小于” 的推广. 这里的微分结构, 实质上是在拓扑结构基础之上的一种东西, 有
了拓扑结构之后, 我们便可以研究连续的概念, 连续在分析中是 C 0 的, 自然而
然我们会向如果是 C r 的, 此时需要对拓扑结构上附加一些怎样的条件, 同样的,
在分析中可微是一个比连续强很多的一个条件, 通过泛函分析中的纲定理等手
段可以说明 C 1 在 C 0 中是第一纲的, 通俗的说就是 C 1 在 C 0 是很少很少的. 从
这个结论我们可以想象到在连续的基础之上应该要加相当的限制, 并且这种限
制可以使得我们在流形上的函数能够微分, 从前面讨论流形的定义可以猜测, 这
种限制就是转化映射” 相似”, 进一步微分结构就是微分流形的开覆盖及其同胚
族, 所谓微分流形就是拓扑流形 + 微分结构. 而拓扑流形, 数学上的定义是指满
足空间 A2 性质 (任意两个不同的点有不相交的开邻域) 且任意的一个点局部上
都同胚于某一维数的欧式空间的空间. 微分结构, 简单的说是开覆盖和具有 C r
相容和极大性同胚族所构成的坐标卡集. 如同抽象代数中的群结构一样. 所谓结
构, 我们最开始接触这个概念应该是在中学化学中, 有机物的化学结构, 结构式
等. 直观来说, 结构就是存在于宏观物质内部的某种微观连接性质. 其本身存在
于我们通常认知的事物之中, 人们从这种事物中抽象出来一种” 结构”. 于研究而
言, 结构是为了让某个集合的对象拥有一些良好的性质方便人们方便研究. 而微
分结构就是为了能够在流形上进行微分等基本分析学的操作而引入的一种结构.
关于微分结构, 值得说得还不止这一点.M ilnor 的球的微分结构一文引起了
人们对流形上微分结构的关注, 微分拓扑中对微分结构的关注比微分流形要多.

比喻来讲, 流形就像一条鱼. 鱼鳞就是开覆盖, 鱼鳞与鱼鳞之间的相交的部


分看起来很光滑就是相容性, 极大性就是任意一条鱼鳞如果与其他相交鱼鳞相
容, 那么这片鱼鳞本身就在这条鱼上. 流形作为集合就像一个承载的对象, 流形
的微分结构就是如何承载这个对象. 如同一个杯子, 流形本身就是杯子的背景空
间, 而流形的微分的结构就是杯子的装水的那个结构. 但比喻只是一种扭曲, 虽
然这种扭曲对理解而言是有帮助的, 因为人们都是通过类比进行联想和理解的,
在比喻过后, 我们仍然需要细细去品味这些定义定理究竟是想表达什么. 之所以
需要这些奇奇怪怪的东西. 是因为流形只是一个几何体而已. 几何体本身不具有
任何坐标, 一旦没有了坐标, 我们的微分就没办法做了, 没有了微分, 微分几何这
门课最为强有力的工具微分便失去了意义. 我们为了使微分仍然有意义, 不得不
想办法去把流形与有坐标系的几何体联系在一起, 当然我们自然会想到有坐标

78
CHAPTER 5. 微分流形初步 5.2. 流形的例子

系的几何体就是欧式空间, 自然地. 我们得继续想办法如何将流形和欧式空间联


系在一起. 想要整体把流形和欧式空间联系在一起, 圆周和球面都告诉我们这是
做不到的 (因为圆不与实数轴及其子区间同胚, 球面也不与平面和平面的区域同
胚), 所以我们放弃整体的联系的想法, 局部的把流形和欧式空间联系起来, 这种
联系便是拓扑里面最为常用的等价关系: 同胚. 局部与局部相交的时候, 其同胚
映射需要满足一定的转化关系. 这样子我们就把流形和欧式空间联系在一起了,
虽然流形上没有可微的定义. 但是欧式空间有, 所以我们考虑将流形上的函数通
过同胚映射转化到欧式空间的函数, 这样我们就把欧式空间中可微函数的概念
搬到了流形上面. 这样我们强大的工具微分在此起了作用. 但这里值得一提的是,
如何在流形上正确的求高阶导数这个问题直接推动了黎曼几何中联络这个概念
的产生.

5.2 流形的例子
2019-10-23
本节要点/问题提示:
1)、用三种方式给出 S 1 的微分流形结构, 以此说明 S 1 为光滑流形?
2)、给出圆柱面的微分流形结构?
3)、什么是实射影空间? 给出 RP n 的微分流形结构?
4)、两个光滑流形的乘积一定光滑流形吗? 如果是证明的关键点在哪儿? 以
此给出环面的微分流形结构?
回顾: 上节课讲了微分流形的定义以及这个定义的来龙去脉, 直观而简单的
来说微分流形就是局部上看上去和欧式空间差不多并且可以在这种东西上面定
义可微函数的拓扑空间. 它是三维空间中曲线和曲面的推广. 从这个角度上说,
光滑的曲面和曲线都是光滑流形. 流形的原始定义有三点: 开覆盖性、C r 相容、
极大性. 事实上, 我们在验证一个拓扑空间是流形的时候只需要验证开覆盖性和
C r 相容性.
设 M n 是一微分流形,{(Uα , φα ) | α ∈ I} 是其微分结构. 对于 Uα ∩ Uβ ̸= ∅,
设对应的坐标域为 (Uα , xi ) 和 (Uβ , y i ). 在相交区域上, 设 x ∈ φα (Uα ). 其中
x = (x1 , x2 , · · · , xn ) ∈ Rn . 对于 y ∈ φβ (Uβ ). 其中 y = (y 1 , y 2 , · · · , y n ) ∈ Rn . 则

∂(y 1 , y 2 , · · · , y n )
= φβ ◦ φ−1
α .
∂(x1 , x2 , · · · , xn )
因此, 设 y i = y i (x1 , x2 , · · · , xn ), 即

y = φβ ◦ φ−1
α (x).

79
5.2. 流形的例子 CHAPTER 5. 微分流形初步

φα (Uα ∩ Uβ ) 7→ φβ (Uα ∩ Uβ ).

上节课就举了一个单位圆的例子. 这节课我们来看看更多的例子, 数学有时候需


要在抽象而一般的理论与具体而特殊的例子之间需要一个平衡. 既要通过具体
的例子去理解抽象的理论, 也要能从抽象的理论回到具体的例子上面.
前面说过流形不过是常见曲面曲线的推广, 这节课我们严格的说明, 我们常
见的曲面的的确确可以赋予一个微分结构使得曲面成为一微分流形.

例 5.3. 圆柱面
如图所示 设该圆柱面为 S, 令 U = S\m, V = S\l. 显然 U, V 是 S 的一

图 5.3: 圆柱

个开覆盖. 分别沿着 m 和 l 剪开, 即考虑映射 φ : U 7→ [(π, 3π) × R] ⊆ R2 . 即


φ(x, y, z) = (θ, z). 类似地, 令 ψ : V 7→ [(0, 2π) × R] ⊆ R2 . 即 ψ(x, y, z) = (α, s).
注意到 U ∩ V = S\ {m, l} . 考虑对应的转换映射

ψ ◦ φ−1 : φ(U ∩ V ) 7→ ψ(U ∩ V ).


ψ ◦ φ−1 (θ, z) = (α, s).

θ + 2π, θ ∈ (0, π),
其中 α = 相应的 z = s. 计算相应的 Jacobian 知
θ, θ ∈ (π, 2π).

∂(α, s)
= 1.
∂(θ, z)

即 {{U, φ} , {V, ψ}} 为相应的光滑结构.

在上一节中我们讲了 S 1 , 我们在下面的例子中先回顾 S 1 , 然后再来给出球


面的微分结构. 并给出 S n 的光滑结构.

例 5.4. n 维球面 S n

80
CHAPTER 5. 微分流形初步 5.2. 流形的例子

图 5.4: 单位圆周 S 1

虽然我们在前面提及过 S 1 , 如图所示 但我们现在想对这个微分结构 (转换


映射) 做一些比较好的处理, 使得后面我们对 S n 有着更好的理解. 设 θ 为直线
m 和横坐标轴的夹角. 自然, 由平面几何基本知识知直线 n 和纵坐标轴的夹角
1
为 θ, 注意到在上述的两个三角形中有 tan θ = u = v1 , 这说明 v = 1
u, u = v1 . 受
上述启发, 考虑高维球面 S n 如图所示 令 N 为 S n 的北极点,S 为 S 的南极点. n

图 5.5: 高维球面 S n

自然 v = (v 1 , v 2 , · · · , v n ),u = (u1 , u2 , · · · , un ). 令 U = S n \N, V = S n \S. 显然


u 和 v 是共线的. 作为向量而然自然有 v = λu, 另一方面, 受上述启发计算相
|v|
应的 tan, 有 tanα = 1
|u| = 1 , 带入向量共线的式子有 λ | u2 |= 1, 综上可解得
j
λ= ∑n 1 i 2 , 这就得到 v = ∑n u (ui )2 .
j
i=1 (u ) i=1

事实上 S n 还可以用其他方式给出微分结构. 为简单起见只对 S 1 进行说明.

例 5.5. S 1 的另外一种给出同胚映射和开覆盖的方式.
idea: 把圆周分解成四个半圆, 然后让这四个半圆拼接起来.
− −
令 U1+ = {(x1 , x2 ) | x2 > 0} , φ+
1 ((x1 , x2 )) = x1 .U1 = {(x1 , x2 ) | x2 < 0} , φ1 ((x1 , x2 )) =
− −
2 ((x1 , x2 )) = x2 .U2 = {(x1 , x2 ) | x1 < 0} , φ2 ((x1 , x2 )) =
x1 . 相应的,U2+ = {(x1 , x2 ) | x1 > 0} , φ+
x2 . 当 上 述 四 个 开 集 相 交 非 空 时, 不 是 一 般 性, 即 考 虑 U1+ ∩ U2+ , 注 意 到
√ + −1

φ+ +
1 (p) = x1 , φ2 (p) = x2 = 1 − x11 . 于是 φ+
2 ◦ (φ1 ) = 1 − x11 显然是
可微的.

除了球面以外, 实射影空间 RP n 在微分流形中也是重要的, 顾名思义, 这个


空间与射影有关, 也就是和一条直线上的等价类有关.

81
5.2. 流形的例子 CHAPTER 5. 微分流形初步

例 5.6. 实射影空间 RP n
为 简 单 起 见 先 考 虑 RP 2 , 即 考 虑 R3 中 过 原 点 的 直 线 (在 同 一 条 直
线 上 的 点 看 做 是 等 价 的). 换 言 之, 定 义 R3 \O 上 的 等 价 关 系:x 与 y 等 价
(x ∼ y) 当 且 仅 当 存 在 λ ̸= 0, 使 得 x = λy, 记 x 的 等 价 类 为 [x] , 实
射 影 平 面 RP 2 定 义 为 RP 2 = R3 \O/ ∼. 其 中 [x] 的 拓 扑 为 商 拓 扑. 这 样
定 义 的 拓 扑 空 间 是 满 足 A2 性 质 和 T 2 性 质 的 (即 满 足 第 二 可 数 性 公 理 的
Hausdorf f 空间). 取如下开集为其开覆盖 U1 = {[x1 , x2 , x3 ] | x1 ̸= 0} , U2 =
{[x1 , x2 , x3 ] | x2 ̸= 0} , U3 =( {[x1 , x )2 , x3 ] | x3 ̸= 0} . 相 应 的 同 胚 映(射 (坐 标 )函
x2 x3 x1 x3
数) 为 φ1 ([x1 , x2 , x3 ]) = x1 , x1 = (ξ1 , ξ2 ) .φ2 ([x1 , x2 , x3 ]) = x2 , x2 =
( )
(η1 , η2 ) .φ3 ([x1 , x2 , x3 ]) = xx13 , xx32 = (ζ1 , ζ2 ) . 当各个开集相交非空的时候,
不失一般性, 考虑 U1 ∩ U2 ̸= ∅, 转换映射的表达式为
1 ξ2
(η1 , η2 ) = φ2 ◦ φ−1
1 (ξ1 , ξ2 ) = ( , ).
ξ1 ξ1
显然是光滑的, 这就说明了 RP 2 为光滑流形, 同理可说明 RP n 为光滑流形.

为了能够构造相当一大类的流形, 我们给出如下定理, 它说明了任何光滑流


形的乘积仍然为光滑流形.

定理 5.1. 设 M 和 n 分别为 m 和 n 维光滑流形, 则 M × N 为 m + n 维光滑


流形.

证明. 由 M 和 N 是 m 和 n 维流形知, 可设

A1 = {(Uα , φα ) | α ∈ I1 } 和 A2 = {(Vβ , ψβ ) | β ∈ I2 } .

分别为 M 和 N 的开覆盖 (光滑结构), 从而

{Uα × Vβ | (α, β) ∈ I1 × I2 } .

必为 M × N 的开覆盖. 对每一个指标 (α, β) ∈ I1 × I2 , 定义映射 η(α,β) 如下:

Uα × Vβ 7→ Rm+n .

η(α,β) (x, y) = (φα (x), ψβ (y)), ∀ (x, y) ∈ Uα × Vβ .

显然 η(α,β) 是 Uα × Vβ 到 η(α,β) (Uα , Vβ ) ⊂ Rm+n 的同胚映射. 考虑其转换映


射, 当
Uα1 ∩ Uα2 ̸= ∅ 且 Vβ1 ∩ Vβ2 ̸= ∅.

时, 有
−1
η(α2 ,β2 ) η(α 1 ,β1 )
: η(α1 ,β1 ) ((Uα1 ×Vβ1 )∩(Uα2 ×Vβ2 )) 7→ η(α2 ,β2 ) ((Uα1 ×Vβ1 )∩(Uα2 ×Vβ2 )),

82
CHAPTER 5. 微分流形初步 5.3. 切向量

进一步考虑转换映射的表达式:

−1 −1 −1
η(α2 ,β2 ) η(α 1 ,β1 )
= η(α2 ,β2 ) (φ−1 −1
α1 , ψβ1 ) = (φα2 ◦ φα1 , ψβ2 ◦ ψβ1 ).

这说明是转化映射光滑的. 极大性显然. 进而就说明了 M × N 是 m + n 为光滑


流形.

注解 5.8. 证明的要点在于转换映射这里, 首先是相交邻域非空, 因为是两个流


形的乘积, 自然会问是要求两个流形的邻域都相交非空, 还是只要求一个流形的
邻域相交非空. 再联想到邻域的维数要求 (同胚于相同维数的欧式空间), 进一步
确定是前者. 其次是转换映射的表达式和定义式的计算, 粗略的说就是把两个流
形“放”在一起. 当然如果熟悉拓扑的话, 自然会问有限个流形的乘积是否还有
这样的结论, 凭感觉应该是对的 (似乎利用数学归纳法是一件显然的事情). 至于
无限个流形的乘积, 也许可以利用泛函的一些基本想法去处理. 另外,2018 年丘
赛初赛的几何与拓扑个人赛第一题就是这个题目 (因为当时啥也不会考了零分,
所以印象十分深刻).

根据上述定理, 我们给出如下例子.

例 5.7. n 维环面 T n
设 T n = S 1 × S 1 × · · · × S 1 (一共 n 个流形做乘积), 因为 S 1 为光滑流
形, 有上述定理知 T n 为光滑流形, 并且可以写出其坐标表示 p ∈ T n ,p =
(eiθ1 , eiθ2 , · · · , eiθn ), 其中 (θ1 , θ2 , · · · , θn ) ∈ Rn .

5.3 切向量
2019-10-30
本节要点/问题提示:
1)、什么是内蕴?
2)、为什么需要站在泛函的角度看切向量?
3)、切向量的公理化定义?
4)、定义一个线性结构使得切空间是一个线性空间?
5)、切空间的基和维数? 证明的要点在哪儿? 证明所需要的关键引理是什
么?
回顾: 切线是我们在中学就接触过的概念, 但这个概念在学完本科的古典微
分几何的时候就产生了一些新的问题. 什么意思, 考虑平面上的一条光滑曲线.
对曲线上固定的一个点, 用割线的极限去定义切线, 也就是切向量. 这么做看上
去是完美而直观的. 但我们在古典微分几何的时候就已经在 Gauss − Bonnet 定

83
5.3. 切向量 CHAPTER 5. 微分流形初步

理之后产生了一个新的概念, 内蕴. 什么意思, 就是说一个几何体是一个单独的


几何体, 除了这个几何体本身之外没有任何外围空间. 比如平面上的单位圆周.
平面上除了单位圆周以外的所有点都是它的外围. 平面就它的外围空间. 但内蕴
的意思就是, 只有流形 (几何体本身), 没有任何外围空间. 也就是只有圆周本身,
圆周外的点都不存在. 在这种情况下如何去定义有外围空间的时候的那些几何
概念便成了数学家们研究的问题. 这一节我们主要讨论切向量, 同样的, 我们要
问, 如果只有单位圆周没有任何外围的点, 没有平面, 没有任何其他点, 我们要
问, 这时候圆上是否还存在切向量, 也就是说, 切向量是流形本身所具有的还是
说切向量是流形依赖于外围空间才有的概念? 换言之, 切向量是否一定需要有平
面才有定义. 因为流形之外的点都不存在了, 曲线外面的点都不存在了, 所以割
线也不存在了, 自然割线的极限也就无从谈起. 所以我们想办法去不用割线的极
限去定义切向量. 自然, 缺少了几何直观直观. 我们只能通过一些所谓的公理化
的方式去推导切向量. 当然在曲面的情况下就变成了切平面, 高维的流形就变成
了切空间. 当然无论是切平面还是切空间, 最基本的建筑块就是就是切向量. 既
然是切空间本身是一个线性空间, 我们自然而然想到线性空间是有同构. 从而我
们想办法去找一个线性空间, 这个线性空间是与外围空间的存在与否没有什么
关系的. 对于线性空间而言, 我们可以认为同构的线性空间都是一样的. 下面我
们就开始来做这件事情.

具体来说, 考虑 Rn 的切向量,p ∈ Rn . 先给定过 p 的曲线 γ : (a, b) 7→


Rn , 0 ∈ (a, b), γ(0) = p, 在 γ 在 p 的切向量 v,v = γ ′ (0). 设 γ(t) = (x1 (t), x2 (t), · · · , xn (t)).
1 2 n 1 2 n
则对 γ 求导,γ(0) = dγ
dt |0 = ( dx
dt , dt , · · · , dt ) |0 = ( dt |0 , dt |0 , · · · , dt |0 ).
dx dx dx dx dx

对于给定的 v ∈ Rn , 存在曲线 γ : (a, b) 7→ Rn 使得 γ(0) = p, γ ′ (0) = v. 事实上,


令 γ(t) = p + tv. 显然 γ(0) = p, γ ′ (0) = v. 一般而言, 满足这种条件的曲线有无
穷多条.

我们想要通过这个向量构造一个与这个外围空间无关的东西, 自然, 我们会


想到利用这个向量去定义一个东西, 显然会想方向导数这个东西, 我们想要建立
这个向量与所谓方向导数之间的一一对应, 即同构. 我们用泛函的观点去看向量,
也就是向量与方向导数之间的一一对应. 因为方向导数 Dv : f ∈ Cp∞ 7→ R. 即在
某个点 p 处 Dv (f ) 是一个实数. 把函数映射为数的映射我们称之为泛函, 这便
是用泛函分析的观点看待方向导数在某一点的取值这件事情. 再重新理一遍逻
辑, 把某一点处的切向量看作是某一点的方向导数, 再把某一点处沿着切向量的
方向导数看作是一个泛函. 我们知道方向导数具有线性性和导子性质. 自然把他
看成一个泛函的时候应该也会具有线性性和导子性质. 这就是我们在流形上用
公理化的方式定义切向量的基本思路:n 为光滑流形 M 在点 p 的切向量是指一
个如下定义的泛函 v : Cp∞ 7→ R. 该映射具有线性性和导子性质. 在给出精确的

84
CHAPTER 5. 微分流形初步 5.3. 切向量

数学严格定义之前, 我们还是先看一个例子.

例 5.8. 设 p ∈ U ⊆ Rn , f ∈ Cp1 , 即 f 在 p 处一阶可导. 设 v 是 p 处的一个切向


量. 对任意的光滑曲线 γ 满足条件 γ(0) = p, γ ′ (0) = v. 求复合映射 f ◦ γ 在 0 处
的切向量和在 p 处沿着向量 v 的方向导数 Dv .

证明. 考虑映射 f ◦ γ 的具体表达式, 按照上面讨论的相同记号, 有

f ◦ γ(t) = f (x1 (t), x2 (t), · · · , xn (t)).

根据求导的链式法则, 我们有

df ◦ γ(t) f (x1 (t), x2 (t), · · · , xn (t))


=
dt dt
∂f dxi
= |p |0
∂xi dt
dxi ∂f
= |0 |p
dt ∂xi
dxi ∂
= |0 |p (f ).
dt ∂xi
1 2 n

dt |0 , dt |0 , · · · , dt |0 ). 于是我们记
根据之前的说明, 我们知道 v = ( dx dx dx

dxi ∂
Dv = |0 |p .
dt ∂xi
我们证明 v 与 Dv 之间存在着一一对应, 即 v 的全体所构成的线性空间与 Dv
的全体所构成的线性空间是一一对应的. 记映射 D : v 7→ Dv 显然 D 是满射, 也
是单射. 于是对任意的 v = (ξ1 , ξ2 , · · · , ξn ) ∈ Rn , 按照上述记号有
∂ ∂ ∂
Dv = ξ1 |p +ξ2 2 |p + · · · + ξn n |p
∂x1 ∂x ∂x
因为上 述 Dv 组成 的全体为线性空间, 所以我们 定义 全体 Dv 组成 的集 合
中 的 线 性 结 构 使 之 自 然 的 成 为 一 线 性 空 间. 对 任 意 的 f ∈ Cp∞ , 对 任 意 的
i |p , Dv2 = ξ2 ∂xi |p . 定义元素之间的加法
∂ i ∂
Dv1 = ξ1i ∂x


(Dv1 + Dv2 )(f ) = (ξ1i + ξ2i ) |p (f ).
∂xi
定义元素的数乘, 对任意的 λ ∈ R. 有

λDv = (λξ i ) |p (f ).
∂xi
容易验证, 这样定义的线性结构使得 Dv 的全体组成的集合成为一线性空间, 并
且这个线性空间与 v 组成的线性空间 Rn 同构.

85
5.3. 切向量 CHAPTER 5. 微分流形初步

注 解 5.9. 首 先 说 明 流 形 上 p 点 定 义 的 可 微 函 数 的 全 体 记 为 Cp∞ . 说 到 可
微 函 数 这 个 问 题, 其 实 在 微 分 流 形 那 一 节 已 经 提 过 了, 在 这 里 在 做 一 个 声
∂f ◦φ−1
i |p := |φ(p) 左边是形式记号, 右边是实质意义.
∂f
明. ∂x ∂xi
−1
d(f ◦γ) dui ∂f ◦φ dui dui
注解 5.10. 记 v(f ) := dt |p = dt ∂ui |φ(p) = dt |0 ∂f
∂ui |p = |p
dt |0 ∂
∂ui
i
du
(f ). 即 v = | ∂
dt 0 ∂ui p| . 特别地, 令 γ i (t) 使 φ◦γ i (t) = (u1
0 , · · · , u i
0 +t, · · · , u n
0 ).
j duj j duj j ∂
记 uj (t) = u0 + δi t, 显然有 dt = δi . 于是有 dt |0 ∂uj |p = δi ∂uj |p = ∂ui |p .
i ∂ ∂

注解 5.11. 上述例子就完成了我们的第一步, 找到了一个与外围空间无关的线性


空间和同构映射, 使得切空间能够与这个线性空间同构, 为此我们可以认为切向
量和 Dv 是同一个东西. 有了这个线性空间, 接下来, 我们自然会问, 这个线性空
间中的元素具有哪些性质. 即 Dv 有哪些性质.

性质 5.1. 对任意的 α, β ∈ R, f, g ∈ Cp∞


1)、(线性性质)Dv (αf + βg) = αDv (f ) + βDv (g).
2)、(导子性质)Dv (f g) = f (p)Dv (g) + g(p)Dv (f ).

注解 5.12. 事实上, 上述性质就是微积分中方向导数本身的性质, 后面我们将会


从这两个性质出发, 公理化的去定义切向量. 并且在把 Dv 和 v 视作同一种东西
的情况下, 直接用 v 替代 Dv 于是上述性质可改写为.

性质 5.2. 对任意的 Rn 中的一个切向量 v, 对任意的 α, β ∈ R, f, g ∈ Cp∞


1)、(线性性质)v(αf + βg) = αv(f ) + βv(g).
2)、(导子性质)v(f g) = f (p)v(g) + g(p)v(f ).

接下来如果我们想用上述性质去定义一个切向量, 自然要问, 满足上述性质


的映射是否一定是方向导数 Dv , 也就是说上述映射是否一定对应一个切向量 v?

问题 5.1. 满足上述性质 5.2 的映射 v 是否一定对应一个切向量?

答案是肯定的, 至于为什么, 暂时不在笔记中给出, 有的时候学数学还是需


要一定的思考空间, 如果全部都把答案全盘托出, 扼杀了你们的想象力, 记得我
本科那个数学很好的同学跟我说过一句话:书籍限制了我的想象力, 所以不要老
是看书, 关上书你也可以尝试写书, 不要让自己生活在某种限制之下, 而逃离限
制最好的方式就是想象力和思考力.
有了上述的铺垫, 我们就能定义内蕴的一个切向量了, 这就达到了之前的目
的.

定义 5.8. 设 M 是一个 n 维光滑流形,p ∈ M, 流形上的点 p 处的一个切向量是


指一个泛函 (映射)v : Cp∞ 7→ R. :

86
CHAPTER 5. 微分流形初步 5.3. 切向量

1)、(线性性质)v(αf + βg) = αv(f ) + βv(g).


2)、(导子性质)v(f g) = f (p)v(g) + g(p)v(f ).

有了一个定义, 自然我们研究这个概念本身所具有的性质是一方面. 另一方


面, 我们还得研究这个元素/概念的全体构成的空间是否具有某种性质/结构? 基
于这种想法, 我们引入切空间及其相应的线性结构.
将 n 维光滑流形 M 在 p 的所有切向量的集合记为 Tp M, 有了集合, 因曲线
的切向量有线性结构, 曲面的切平面也有自然的线性结构. 我们自然要问 Tp M
是否可以定义线性结构? 使之成为一线性空间. 猜测的话肯定是可以的. 我们下
面着手来坐这一件事情.

命题 5.1. 集合 Tp M 可赋予线性结构使之成为线性空间, 称之为 M 在 p 点处的


切空间.

证明. Tp M 上可赋予线性结构. 事实上, 我们定义如下的加法和代数乘法运算.


对任意的切向量 ν, µ ∈ Tp M , 对任意的 f , 定义

(ν + µ)(f ) := ν(f ) + µ(f ).

对任意的 λ ∈ R, 定义
(λµ)(f ) := λ · µ(f ).

我们要证明 ν + µ ∈ Tp M, λµ ∈ Tp M. 即证明 ν + µ, λµ 仍然是 p 处的切向量.


即证明 ν + µ, λµ 满足线性性质和导子性质. 对任意的 f, g ∈ Cp∞ , 和任意的
α, β ∈ R. 考虑

(ν + µ)(αf + βg) = ν(αf + βg) + µ(αf + βg)


= αν(f ) + βµ(g) + αµ(f ) + βµ(g)
= α(ν(f ) + µ(f )) + β(ν(g) + µ(g))
= α(ν + µ)(f ) + β(ν + µ)(g)

进一步验证导子性质. 考虑

(ν + µ)(f g) = ν(f g) + µ(f g)


= f (p)ν(g) + g(p)ν(f ) + f (p)µ(g) + g(p)µ(f )
= f (p)(ν(g) + µ(g)) + g(p)(ν(f ) + µ(f ))
= f (p)(ν + µ)(g) + g(p)(ν + µ)(f )

87
5.3. 切向量 CHAPTER 5. 微分流形初步

这说明 ν + µ 也是 p 处的切向量. 同理, 我们验证, 对任意的 λ ∈ R, 我们验证


λµ 也是切向量. 记号同上. 考虑

λµ(αf + βg) = λ · µ(αf + βg)


= λ · (αµ(f ) + βµ(g))
= αλ · µ(f ) + βλ · µ(g)
= αλµ(f ) + βλµ(g)

同理可验证导子性质. 这就说明了 λµ ∈ Tp M. 于是 Tp M 成一线性空间.

有了线性空间, 我们自然会问我们对这个线性空间了解了多少, 对于一


个线性空间的大致了解, 无外乎说是线性空间的基和维数这两个问题, 当然.
我们至少知道 ∂
∂ui |p ∈ Tp M. 自然, 我们希望从这里出发, 去找出这个线性
空间的基和维数. 但在做这件事情之前, 我们先来看看熟悉的曲线的切向量
和曲面的切平面的基和维数. 显然曲线在一点处的切向量的维数是 1, 并且
{ ∂ }
1 |p ∈ Tp M = span ∂u1 |p . 对 于 曲 面 而 言, 显 然 在 曲 面

可 以 认 为 就 是 ∂u
上的任意 一点的切平面的维数是 2, 在适当 的 坐标 系 下我 们可 以 认为 就 是
{ ∂ }
∂u1 |p , ∂u2 |p ∈ Tp M = span ∂u1 |p , ∂u2 |p . 因为流形不过是曲面曲线的推广,
∂ ∂ ∂

自然我们有理由猜测 ∂
∂ui |p 就是 Tp M 的一组基. 从低维先想明白了, 理解了最
基本的想法的来源. 接下来的逻辑推导就是数学里面最常见的东西了. 有时候学
数学的一个误区就是所有的逻辑推导都明白, 但对于这么做的目的以及这些想
法的来源一无所知, 这并不是一个好现象.
为了证明我们的猜测, 我们首先得花点功夫去证明一个引理, 这是数学分析
(欧式空间) 中的一个引理, 我们只需要花点功夫把这个欧式空间上的函数变成
流形上的函数即可.

引理 5.1. 设 M 是一个 n 为光滑流形,(U, φ) 是点 p ∈ M 的一个局部相容坐标


卡,(U, xi ) 是相应的局部坐标系, 对任意的 q ∈ U , 记 p 的坐标 ui0 = ui (p), 则对
任意的 f ∈ Cp∞ , 有

n
f (q) = f (p) + (ui − ui0 )gi .
i=1

其中 gi ∈ Cp∞ , 且 gi (p) = ∂
∂ui |p (f ).

证明. 对任意的 f ∈ Cp∞ , f ◦ φ−1 ∈ Cφ(p)



. 则 f ◦ φ−1 为欧式空间中的光滑函数,

88
CHAPTER 5. 微分流形初步 5.3. 切向量

令 u = φ(q), 利用微积分基本定理有

f ◦ φ−1 (u) − f ◦ φ−1 (u0 ) = f ◦ φ−1 (u1 , u2 , · · · , un ) − f ◦ φ−1 (u10 , u20 , · · · , un0 )
∫ 1
d(f ◦ φ) 1
= (u0 + t(u1 − u10 ), u20 + t(u2 − u20 ), · · · , un0 + t(un − un0 ))
0 dt
∫ 1∑ n
∂f ◦ φ−1
= i
(u0 + t(u − u0 ))(ui − ui0 )dt
0 i=1 ∂u
∑n ∫ 1
∂f ◦ φ−1
= i
(u0 + t(u − u0 ))(ui − ui0 )dt.
i=1 0 ∂u
(5.1)
定义 gi ∈ Cp∞ 使得
∫ 1
∂f ◦ φ−1
gi ◦ φ−1 (u) = (u0 + t(u − u0 ))(ui − ui0 )dt.
0 ∂ui

于是 (5.1) 式可化为

n
f ◦ φ−1 (u) − f ◦ φ−1 (u0 ) = (ui − ui0 )gi ◦ φ−1 (u). (5.2)
i=1

注意到 u0 = φ(p), u = φ(q), 并且 φ 是同胚, 进一步我们有 p = φ−1 (u0 ), q =


φ−1 (u). 则 (5.2) 式可化为

n
f (q) − f (p) = (ui − ui0 )gi (u).
i=1



n
f (q) = (ui − ui0 )gi (u) + f (p).
i=1

考虑

gi (p) = gi (φ−1 (u0 )) = gi ◦ φ−1 (u0 )


∫ 1
∂f ◦ φ−1
= (u0 + t(u − u0 ))(ui − ui0 )dt |u=u0
0 ∂ui
∫ 1
∂f ◦ φ−1
= (u0 + t(u − u0 ))(ui − ui0 ) |u=u0 dt
0 ∂ui
∂f ◦ φ−1
= |φ(p)
∂ui

= |p (f ).
∂ui
这就完成了引理的证明.

89
5.3. 切向量 CHAPTER 5. 微分流形初步

注解 5.13. 上述引理的结果无外乎是把数学分析中欧式空间中的函数变成了流
形上的函数, 我们证明的要点就是把流形上的函数映射到到欧式空间去做微积
分, 再利用局部坐标映射把把欧式空间重新拉回到流形上去. 这种数学思想贯穿
于整个微分几何中, 流形本身就是我们不太熟悉的东西, 把不熟悉的东西转化为
欧式空间中的东西. 在把欧式空间中的东西通过局部坐标映射的逆重新拉回到
流形上面去.

注解 5.14. 上述的 (5.1) 式的第三个等号需要解释一下, 不止有一个同学和我探


讨过这个问题. 先把问题描述一下: 在 (5.1) 式的第三个等号实际上抽离出来便

∫ 1
d(f ◦ φ−1 ) 1
(u0 + t(u1 − u1 − u10 ), · · · , un0 + t(un − un0 ))
0 dt
∫ 1∑ n (5.3)
∂f ◦ φ−1
= (u0 + t(u − u0 ))(ui − ui0 )
0 i=1 ∂ui

为什么这个式子的偏导可以写成这种样子? 如果 f ◦ φ−1 对 ui , 那么下面那个和


式不应该还有一个 t 吗? 事实上, 在解释上面第三个等式之前, 我们还得重新回
顾二元函数的一点点东西.
考虑平面区域 D 上的二元可微函数 f (x, y), 对任意的 (x0 , y0 ) ∈ D, 考虑

f (x0 + ∆x, y0 + ∆y) − f (x0 , y0 ) = f (x0 + ∆x, y0 + ∆y) − f (x0 + ∆x, y0 )


+ f (x0 + ∆x, y0 ) − f (x0 , y0 )
= fy (x0 + ∆x, y0 + θ1 ∆y)∆y + fx (x0 + θ2 ∆x, y0 )∆x
∂f ∂f
= (x0 + ∆x, y0 + θ2 ∆y)∆y + (x0 + θ2 ∆x, y0 )∆x
∂y ∂x
(5.4)
上述式子便能够解释为什么 (5.3) 等式成立, 观察 (5.4) 式. 我们在考虑 f (x0 +
∂f
∆x, y0 +∆y)−f (x0 +∆x, y0 ) = fy (x0 +∆x, y0 +θ1 ∆y) = ∂y (x0 +∆x, y0 +θ1 ∆y)
这个式子在我们看来是在正常不过的, 但其中的道理是一样的. 刚刚这个式子,
为什么我们求偏导的时候不写成 fy+∆y 而写成 fy 为什么偏导符号不写成
∂f ∂f
∂y+∆y 而写成 ∂y ? 把这个问题想明白了也就可以解释之前的那个问题. 仔细想
想也大概能猜到这个问题的答案, 这里的 y 只是一个记号而已, 我们默认 f (x, y)
默认其中的变量就是这种形式 y, 方便统一的计算. 计算的时候只需要把自变量
的形式重新带入即可. 这里也是一样的, 实际上, 在式子 (5.3) 中也是这个道理,
用数学符号来形式的说明的话, 便是

∂f ∂f
(u0 + t(u − u0 )) =
∂ui ∂(ui0 + t(ui − ui0 )

90
CHAPTER 5. 微分流形初步 5.3. 切向量

这便是带入的具体含义, 这样子我们就能够准确的理解为什么 (5.1) 的第三个等


号成立了.

定理 5.2. 设 M 是 n 维光滑流形,(U, φ) 是点 p ∈ M 的局部容许坐标卡.(U ; ui )


是相应的局部坐标系. 则 ∂
∂ui |p , 1 ≤ i ≤ n 是切空间 Tp M 的一组基底. 因此
Tp M 的维数是 n.


证明. 要证明 ∂ui 是一组基, 只需要证明可表性和线性无关性. 首先证明线性无
关性. 不妨设存在 ξi 使得

ξi |p = 0. (5.5)
∂ui

注意到 uj = uj ◦ φ. 将 (5.5) 式同时作用于 uj , 有


ξi |p (uj ) = 0(uj ) = 0.
∂ui

∂uj
ξi |p = 0.
∂ui

∂uj ◦ φ
ξi |p = 0.
∂ui

∂uj ◦ φ ◦ φ−1
ξi |φ(p) = 0.
∂ui

∂uj
ξi |φ(p) = 0.
∂ui

这就得到了 ξ = 0. 这就证明其线性无关性. 下面证明任意的切向量都可以用



∂ui 的线性组合表出. 注意到

v(1) = v(1 · 1) = 1 · v(1) + v(1) · 1 = 2v(1)

这说明 v(1) = 0, 进一步 v(c) = v(c · 1) = c · v(1) = c · 0 = 0. 对任意的 f ∈ Cp∞ ,

91
5.4. 切映射 CHAPTER 5. 微分流形初步

由引理 5.1, 有


n
v(f ) = v(f (p) + (ui − ui0 ) · gi )
i=1

n
= v(f (p)) + v( (ui − ui0 ) · gi )
i=1

n
= v( (ui − ui0 ) · gi )
i=1

n
= v((ui − ui0 ) · gi )
i=1

n
= (ui − ui0 ) |p v(gi ) + v(ui − ui0 ) · gi |p
i=1

n
= v(ui − ui0 ) · gi |p
i=1

n
= v(ui ) · gi |p
i=1

n

= v(ui ) · |p (f )
i=1
∂ui

其中 f (p), ui0 看作常值函数. 这就得到了


n

v= v(ui ) · |p
i=1
∂ui


于是 ∂ui 是 Tp M 中的一组基.

注解 5.15. 现在我们已经基本了解了 Tp M 的基和维数. 因为 Tp M 是线性空间,


而线性空间有对偶空间, 我们自然要问, 切空间的对偶空间是什么, 它与我们通
常的微分符号 dui 有怎样的关系, 这就引出了余切空间, 切空间与余切空间之间
有着微妙而美丽的联系.

5.4 切映射
2019-11-6
本节要点/问题提示:
1)、一句话说明什么是余切空间?
2)、通过等价关系如何定义余切空间和微分?

92
CHAPTER 5. 微分流形初步 5.4. 切映射

3)、光滑映射的局部表示?
4)、什么是一般映射的秩? 什么是临界点
5)、如何通过微积分引出切映射? 切映射的一般定义是怎样的?
6)、计算切映射在局部坐标系的表达式? 切映射的几何意义是什么?
7)、如何引出余切映射? 余切映射的一般定义是什么?
8)、计算余切映射在局部坐标系的表达式?
回顾: 上节课讲了切向量, 设 R3 中的一条光滑曲线 γ(t),γ(t) = (x(t), y(t), z(t)).
于是 γ ′ (t0 ) = dγ
dt |t0 = ( dx dy dz
dx
dt |t0 , dt |t0 , dt |t0 ) =
dy
dt (1, 0, 0) + dt |t0 (0, 1, 0) |t0
dy
(0, 0, 1) + dz dx dz
dt |t0 (0, 0, 1) = dt |t0 E1 + dt |t0 E2 |t0 E2 + dt |t0 E3 . 这是最初利
用割线的极限所定义的切向量. 如果我们用泛函的观点定义切向量 v : Cp∞ 7→ R.
dy
根据上一节的定理,v 可以表示为 v = dx ∂ ∂ dz
dt |t0 ∂x |p + dt |t0 ∂y |p + dt |t0 ∂z |p .

并且我们知道 ∂x ∂
|p , ∂y

|p , ∂z

|p 在 p 点处是线性无关的, 这是一个具体的例子.
在流形上用公理化的方式来定义切向量, 具体说是泛函 v : Cp∞ 7→ R. 称之为一
个切向量, 如果 v 满足线性性和导子性. 即
1)、(线性性质)v(αf + βg) = αv(f ) + βv(g).
2)、(导子性质)v(f g) = f (p)v(g) + g(p)v(f ).

上节课为了证明 Tp M 的基是 ∂ui , 我们首先证明了引理 f (q) = f (p) +
∑n
i=1 (u
i
− ui0 )gi (u). 利用这个引理说明了任意的切向量 v 都可用 ∂
∂ui 的线性组
−1
∂f ◦φ
合表出. 并且形式的记 ∂f
∂xi |p := ∂xi |φ(p) 左边是形式记号, 右边是实质意义.
在 数 学 意 义 上 严 格 的 论 述 余 切 空 间 之 前, 我 们 先 简 单 的 谈 一 谈 不 那 么
严格而直观的对余切空间的理解. 有了线性空间 Tp M , 自然我们会问这个
线性空间的对偶空间是什么, 即切空间的对偶空间是什么? 既然切向量有了
公理化的定义, 我们自然希望微分 dui 也能够通过切向量来定义. 这样的话
我们的微分就能摆脱几何直观和外围空间的束缚了. 按照第四章的记号, 记
(Tp M )⋆ , 为方便书写记为 Tp⋆ M 为 Tp M 的对偶空间, 称之为余切空间. 所谓
余代表的是次要的意思. 中文的字面意思大致是次要的切空间. 首先注意到
∂uj
duj |p ( ∂u

i |p ) = ∂ui |p = δij . 如果 duj 是 Tp M 上的线性函数. 并且有刚刚的
δij , 我们就似乎可以断言:duj |p 就是 ∂
∂ui |p 的对偶基, 也是 Tp⋆ M 的一组基.
i |p , 考虑

下面验证 duj 的的确确是 Tp M 上的线性函数, 对任意的 v = ξi ∂u
duj (v) = duj |p (ξi ∂u

i |p ) = ξi du |p ( ∂ui |p ) = ξi du ( ∂ui ) |p ) = ξi du (v). 这说
j ∂ j ∂ j
∑n
明上述的断言直观上应该是成立的. 对任意的 (df ) |p = i=1 ai dui , 显然该式两
∑n ∂f
i |p , 有 aj = ∂ui |p . 带入有 df = i=1 ∂ui |p du |p . 这样子
∂ ∂f
边同时作用于 ∂u i

得到的余切空间和微分和微积分里面的微分形式上看上去是一样的, 但我们并
没有用微积分里面的方法而得到了相同的结果. 不得不说是数学家的精妙思维
得到的相当漂亮的结果. 下面, 我们将严格的定义余切空间, 但我们还得做点铺

93
5.4. 切映射 CHAPTER 5. 微分流形初步

垫说明为什么要这么定义.
∑n
首先我们刚刚说明了对任意的 f ∈ Cp∞ , 则 (df ) |p = i=1 ai dui . 另一方面
∑n ∑n
对于给定的 i=1 ai dui , 只要构造 φ(u) = ai ui 即有 (dφ) |p = i=1 ai dui . 这说
{ }
明了 Tp⋆ M = (df ) |p | f ∈ Cp∞ 但需要注意的是 C ∞ 7→ R 的映射 f 7→ (df ) |p .
并不是一个单射. 事实上, 若 (df ) |p = (dg) |p , 只需要 (d(f − g)) |p = 0, 只需
要 f − g 的微分为零, 但这并不能推出 f = g, 最简单的情况莫过于相差一个常
数. 我们想要找到一个一一对应, 但这个一一对应的空间不是 Cp∞ , 应该是它的
某个商空间, 考虑 N 为具有 (d(h)) |p = 0 的所有函数 h 构成的空间, 我们猜测
d : C ∞ /N 7→ Tp⋆ M 之间的映射应该是一一映射. 而定义域为一个商空间, 在抽
象代数中我们知道一个商空间唯一确定了一个等价关系, 一个等价关系也唯一
确定了一个商空间. 于是, 我们按照上面的思路确定了一种等价关系, 根据这种
等价关系定义了我们称之为微分的东西.

定义 5.9. 设 f, g ∈ Cp∞ , 定义 f 等价于 g, 当且仅当对任意的 v ∈ Tp M ,v(f ) =


v(g), 记这样的等价类为 [f ]

定义 5.10. 有了上述等价类, 我们考虑映射 [f ] : Tp M 7→ R, 定义 [f ] (v) := v(f )

容易验证, 上述定义是合理的, 即与代表元的选取无关, 事实上, 如果有


[f ] = [g], 则根据等价类的定义, 对任意的 v ∈ Tp M, 有 v(f ) = v(g), 注意到
[f ] (v) = v(f ), [g] (v) = v(g). 于是有 [f ] (v) = [g] (v). 这就说明了定义的合理
性. 并且上述映射显然是 Tp M 上的线性映射. 事实上, 对任意的 v1 , v2 ∈ Tp M ,
有 (v1 + v2 )(f ) = (v1 )(f ) + (v2 )(f ) = [f ] (v1 ) + [f ] (v2 ). 对 任 意 的 λ ∈ R,
有 [f ] (λv) = (λv)(f ) = λ(v)(f ) = λ[f ](v). 这就说明了 [f ] 的线性性, 于是
[f ] ∈ Tp⋆ M.
有了单个元素/概念的定义, 自然, 我们会想这些元素组成的全体作为一个
集合, 上面能否赋予一些比较好的结构. 首当其冲的便是线性结构. 而这种定义
方式和之前也是一样的.

命题 5.2. 上述等价类的全体构成了 Tp⋆ M, 于是将上述等价类全体记为 Tp⋆ M (称


之为余切空间), 在 Tp⋆ M 上可赋予线性结构使之成为一线性空间.

证明. 定义如下的线性结构, 对任意的 [f ] , [g] ∈ Tp⋆ M, 定义

([f ] + [g])(v) := ([f ])(v) + [g] (v) = v(f ) + v(g).∀ v ∈ Tp M.

对任意的 λ ∈ R, 定义

(λ [f ])(v) := λ · [f ] (v) = λv(f ).∀ v ∈ Tp M.

容易证明, 按上述定义的线性结构 Tp⋆ M 的确成一线性空间.

94
CHAPTER 5. 微分流形初步 5.4. 切映射

注解 5.16. 为了在形式的记号上表明 [f ] 的的确确是属于 Tp⋆ M , 而且能够体


现这种对偶的关系, 能够体现它作为余切向量 (切向量的对偶), 我们记 [f ] =
[ ]
(df ) |p . 特别地, 在 p ∈ U , 在 (U, ui ) 作为局部坐标系时, 有 ui = (dui ) |p ,
∂uj j ∑n
duj |p ( ∂u

i |p ) = ∂ui |p = δi . 进一步对任意的 (df ) |p = i=1 ai du , 显然该式两
i
∑n
i |p , 有 aj = ∂ui |p . 带入有 df = i=1 ∂ui |p du |p . 这便是
∂ ∂f ∂f
边同时作用于 ∂u i

我们最初的想法及其结果. 称 (df ) |p 为函数 f 在 p 处的微分.

在拓扑流形上, 我们只能研究连续函数, 连可微函数都无从谈起, 我们关注


空间本身, 也要关注空间能够” 长出来” 的东西 (即能够在空间上定义的函数等).
因为没有可微函数, 我们自然会想是不是这个流形本身有什么局限性, 这时候我
们引入了微分结构这个概念, 这直接导致了流形上可以定义可微函数. 但这些可
微函数都是从 M 7→ R 上的, 自然地, 如果我们把值域也换成一个流形的话, 这
种所谓的光滑映射又具有哪些性质呢? 这个概念本身又可以引出/附带哪些比较
有意思的想法和概念呢? 接下来, 我们便要围绕着流形到流形之间的光滑映射为
中心, 引入光滑映射所附带的诸多其他有有趣的概念.

定义 5.11. 设 M 和 N 分别是 M 维光滑流形和 n 维光滑流形, 映射 F : M m 7→


N n 的光滑映射, 对 p ∈ M, 设 (U, φ) 为 p 处的一个局部相容坐标卡,(U, ui ) 为
对应的坐标系.(V, ψ) 为 F (p) 处的一个局部相容坐标卡,(V, v i ) 为对应的坐标系.
称 F 在 p 处是可微 (光滑) 的, 如果 Fe = ψ ◦ F ◦ φ−1 : φ(U ) 7→ ψ(V ) 在 φ(p) 的
一个开邻域内是可微 (光滑) 的.

注解 5.17. 上述定义的方式, 其实之前早就提到过了, 我们无法定义流形上的光


滑映射, 我们就通过同胚映射把光滑映射拉到欧式空间中去定义它的光滑性. 本
质上还是把不熟悉未知的东西转化成已知而熟悉的东西.

设函数 Fe = (v 1 , v 2 , · · · , v n ) = (F 1 , F 2 , · · · , F n ) = (F 1 (u1 , u2 .·, um ), F 2 (u1 , u2 .·, um ), ·, F n (u1 , u2 , · · · , um )).


在上述记号下 Fe 的 Jacobian 为
 1 
∂F 1 1
∂F
· · · ∂F
 ∂F 2
∂u1 ∂u2 ∂un
2 
 1 ∂F 2
· · · ∂F 
 ∂u ∂u2 ∂un 
 . . . . 
 .. .. .. .. 
 
∂F m ∂F m ∂F m
∂u 1 ∂u 2 · · · ∂u n

i
定义 5.12. 在上述记号下, 定义 rank(F ) := rank( ∂F
∂uj ) 为光滑映射 F 的秩, 特
别地, 若 rank(F ) = min {m, n}, 则称 F 是满秩的, 若 rank(F ) < min {m, n}
则称 F 不是满秩的, 不满秩的点称之为临界点.

定义 5.13. 若 F : M m 7→ N n 的 C r 阶可微映射, 并且 F 为同胚, 则称 F 为 C r


阶微分同胚.

95
5.4. 切映射 CHAPTER 5. 微分流形初步

下面这道习题留给大家作为练习. 偶尔做点题能够避免大脑永不思考.

习题 5.1. 证明: 若 F 为 M m 到 M n 的微分同胚, 则 m = n.

注解 5.18. 之前关于切空间写了这么多, 都没有一点点直观的理解, 先谈谈对切


向/切向量的直观理解, 曲线上某点的切向量就是把曲线在这一点处给光滑的搫
直了, 所以切线可以看做曲线在一点处的线性化, 但我们知道曲线可以看做是映
射的图像/值域. 所以切线就相当于这个映射在一点处的线性化. 这时候我们自
然要问, 对于一般的映射, 是否具有切映射 (一点出的线性化) 这个概念? 如果
有, 又应该如何定义切映射 (一点出的线性化)?
同样的, 切平面也是曲面在一点处的线性化得到的, 所以, 因为曲面和曲线
是低维的流形, 流形是高维曲面曲线的推广, 于是, 切空间是流形在一点处线性
化得到的. 对于一般的流形而言, 我们自然也会考虑流形之间的映射的切映射又
该如何定义?.
谈到这个问题, 不得不再次谈及我们的导数, 先看单变量函数的导数,f ′ (x) =
df ∂
y = dx . 在我们来在形式意义上改写这个符号, 因为 ∂ui 是切向量, 当它是单
d d d
变量变成导数的时候 dy , dx 也是切向量, 而我们又可以将 y 等同于 dy , 于是
d df d d
dy = dx , 把右边的 f 移动一下, 移动到 d 的外面, 就有 dy = (df ) dx , 也就是

d d d d
dy = f⋆ dx , 或者写成 dy = f dx . 左边是 y 的切向量, 右边是 x 的切向量. 这说

明 f 即导数是把定义域中的切向量映射为值域中的切向量, 这便是用微分流形
的观点去看导数和切向量. 而导数本身就是函数在一点处的线性化 (切映射), 这
就得到了切映射把定义域中的切向量映射为值域的切向量.
从这个受到启发, 我们自然会问, 一般的光滑映射的切映射是否仍然把定义
域中切向量映射到值域中的切向量. 我们引出如下定义.

定义 5.14. 设 F : M m 7→ N n 为流形之间的光滑映射, 则 F 诱导了一个切映射

F⋆ : Tp M 7→ TF (p) N.

对任意的 v ∈ Tp M, g ∈ Cp∞ 定义

F⋆ (v)(g) = v(g ◦ F ).

为切映射.

注解 5.19. 我们分析一下上述定义的合理性与自然性, 我们知道 F⋆ (v) ∈ TF (p) N,


作为一个切向量, 如果我们想要具体的看看它作用之后会发生什么, 必然会选
取一个函数, 自然地, 我们选取 g ∈ CF∞(p) , 显然有 F⋆ (v)(g) ∈ R, 虽然我们不
知道它的值具体是什么, 但我们能知道它是一个实数, 注意到 v 也是一个切向

96
CHAPTER 5. 微分流形初步 5.4. 切映射

量, 我们能否找到一个 M 7→ R 的函数, 使得 v 作用于这个函数上面是一个


实数, 我们下面就来寻找这样的函数, 当然这个函数肯定的与 F 和 g 有关, 注
意到 F : M 7→ N, g : N 7→ R, 于是 g ◦ F : M 7→ R, 这不是正好符合我们的
期待吗? 事实上这个函数只在 p 的邻域附近有定义, 这就更符合我们的期待
了. 于是我们找到了这个函数 g ◦ F , 进一步我们用 v(g ◦ F ) ∈ R. 自然我们就
F⋆ (v)(g) = v(g ◦ F ). 所以这个定义既是自然地, 又是合理的.

有了切映射, 因为切映射是定义在 Tp M 上的, 有了映射, 自然会想这个映射


具体是怎样的. 我们想要对这个切映射有着详细的了解, 显然因为切映射是线性
的 (切映射的几何意义是光滑映射在一点处的线性化), 于是我们只需要对这个
切映射在线性空间 Tp M 的一组基上的作用了解清楚即可. 我们之前已经证明过
∂ ∂
Tp M 作为线性空间的一组基是 ∂ui , 我们要问 F⋆ ( ∂u i ) 是什么?

问题 5.2. 设 (V ; v i ) 是光滑流形 N 在点 F (p) 的任意一个容许局部坐标系


(对应的坐标卡为 (V, ψ)), 并且存在光滑流形 M 在点 p 处的容许局部坐标系
(U, ui )(对应的坐标卡为 (U, φ).) 使得 f (U ) ⊆ V. 设 F 在局部坐标系的表示为

ψ ◦ F ◦ φ−1 = (F 1 (u1 , u2 , · · · , um ), F 2 (u1 , u2 , · · · , um ), · · · , F n (u1 , u2 , · · · , um ))


= (v 1 , v 2 , · · · , v n ).


求 F⋆ ( ∂u i ) 的表达式?

证明. 对任意的 g ∈ Cp∞ , 有

∂ ∂ ∂
F⋆ ( i
)(g) = i
(g ◦ F ) |p = (g ◦ F ◦ φ−1 ) |φ(p)
∂u ∂u ∂ui

= (g ◦ ψ −1 ◦ ψ ◦ F ◦ φ−1 ) |φ(p)
∂ui

= (g ◦ ψ −1 ◦ (F 1 , F 2 , · · · , F n )) |φ(p)
∂ui
∂g ◦ ψ −1 ∂F j
= j
|ψ(q) |φ(p)
∂v ∂ui
∂g ∂F j
= | q |φ(p)
∂v j ∂ui
∂F j ∂g
= |φ(p) |q
∂ui ∂v j
∂F j ∂
= i
|φ(p) |q (g).
∂u ∂v j
∂F j

于是得到 F⋆ ( ∂u i) = ∂ui |φ(p) ∂
∂v j |q .

97
5.4. 切映射 CHAPTER 5. 微分流形初步

注解 5.20. 将上述得到的结果写成矩阵的形式, 即
   1  
∂F 2 ∂F n
∂ ∂F
· · · ∂
 ∂u   ∂u   ∂v 
1 1 ∂u 1 ∂u 1 1

 ∂ 2   ∂F 21 ∂F 22 · · · ∂F 2n   ∂ 2 
 ∂u   ∂u ∂u ∂u   ∂v 
F⋆  .  =  . .. .. ..   . 
 ..   .. . . .   . 
    . 
1 2 n

∂um
∂F
∂un
∂F
∂un · · · ∂F
∂un

∂v n

它们之间恰好相差了一个 Jacobi 矩阵.

在微分几何中, 定义任何概念以及讲述任何定理的时候, 我们自然要问, 它


们的几何意义是什么, 这里也是, 我们自然会问, 诱导切映射的集合意义是什么?

问题 5.3. 是 f 是 M m 7→ N n 的光滑映射,γ 是 M 上的一条光滑曲线.γ(t0 ) =


p,γ ′ (t0 ) = v.F (p) = F (γ(t0 )) = q. 求 F⋆ (γ ′ (t0 )) 与 ψ ◦ γ 在 t0 处的切向量之间
的关系. 由此说明切映射的几何意义.
dui ∂
证明. 设 v = dt ∂ui , 记 ψ ◦ F ◦ γ(t) = (F 1 (t), F 2 (t), · · · , F n (t)). 于是

dψ ◦ F ◦ γ(t) dF 1 dF 2 dF n
=( , ,··· , ).
dt dt dt dt

dψ ◦ F ◦ γ(t) dF i ∂
= .
dt dt ∂v i
考虑
dui ∂
F⋆ (v) = F⋆ ( )
dt ∂ui
dui ∂
= F⋆ ( i )
dt ∂u
dui ∂F j ∂
= |φ(p) |q
dt ∂u i ∂v j
dF i ∂
= .
dt ∂v j
这说明 F⋆ 将原曲线的切向量映射为像曲线的切向量.

注解 5.21. 值得一提的是, 刚刚的证明的结论和我们最开始引入切映射是一致


的, 虽然做法不尽相同, 但是都能得到切映射把原切空间的切向量映射为像空间
的切向量. 数学有时候就是这样, 用不同的观点去看待同一个问题会得相同的结
果, 虽然看问题的深度各有特色, 但是最后都殊途同归.

注意到映射
F⋆ : Tp M 7→ TF (p) N.

98
CHAPTER 5. 微分流形初步 5.4. 切映射

如果我们对这个式子整体取一个对偶会怎样? 我们知道对偶有时候会把式子反
过来 (比如最优化中的线性规划和对偶规划, 把条件和结论倒置一下.), 于是形
式意义上, 我们得到上述式子的对偶式子

F ⋆ : TF⋆ (p) N 7→ Tp⋆ M.

注意到这种形式意义上的映射是余切空间之间的映射, 于是我们称这种映射为
余切映射, 又称之为拉回映射 (拉回是指相对于切映射的拉回映射).

定义 5.15. 设 F : M m 7→ N n 的光滑映射, 称 F 诱导的映射 F ⋆ : TF⋆ (p) N 7→


Tp⋆ M. 为余切映射, 其定义为对任意的余切向量 (1 形式)ω ∈ TF (p) N, 有 F ⋆ (w) ∈
Tp M, 使得对任意的 v ∈ Tp M. 有

F ⋆ (ω)(v) = ω(F⋆ (v)).

注解 5.22. 首先还是阐述一下上述定义的合理性,F ⋆ (ω) ∈ Tp⋆ M, 则 F ⋆ (ω)(v) ∈


R. 我们想办法构造一个映射使得该映射与 F, ω, v 有关并且是一个实数, 注意到
ω 是一个 F (p) 的余切向量, 如果能构造一个 F (p) 处的切向量, 那么 ω 作用于
这个切向量肯定能得到一个实数. 巧合的是 F⋆ (v) ∈ TFp M. 这样就有我们需要
的那个实数的定义方式了 ω(F⋆ (v))

还是按照之前的思路, 自然我们要问 F ⋆ (dv i ) =?

问题 5.4. 条件同上, 求余切映射 F ⋆ 在局部坐标系的表达式?

证明. 根据定义, 我们有

F ⋆ (dv i |F (p) ) = F ⋆ (dv i |F (p) ) · 1



= F ⋆ (dv i |F (p) ) · |p duj |p
∂uj

= F ⋆ (dv i |F (p) )( j |p )duj |p
∂u

= dv i |F (p) (F⋆ ( j |p ))duj |p
∂u
∂F k ∂
= dv i |F (p) ( j |φ(p) |q )duj |p
∂u ∂v k
∂F k ∂
= |φ(p) dv i |F (p) ( k |F (p) )duj |p
∂uj ∂v
∂F k
= |φ(p) δki duj |p
∂uj
∂F i
= |φ(p) duj |p
∂uj
这也是 Jacobian 矩阵.

99
5.4. 切映射 CHAPTER 5. 微分流形初步

值得一提的是, 在微分拓扑的基本概念中, 有个横截性的概念, 也是与切映


射有关. 某种意义上可以说是线性空间直和的推广, 具体的内容请参见张筑生的
《微分拓扑新讲》

100
第6章 子流形理论

本章从局部与整体的概念开始说起, 局部性质就是关于一个邻域成立的性
质, 而整体性质是指在一个流形的任何一个点都成立的性质 (并且这种性质与点
的选取无关). 为了将局部的光滑函数眼延拓到整体的光滑函数, 我们介绍了截
断函数, 并且以截断函数为工具可证明诸多流形上的定义是整体的和合理的. 为
了说清楚什么是向量场引出了切从的概念, 并把函数图推广成了切从的截面. 和
切向量类似, 我们也站在泛函的观点去看切向量场, 证明了切向量场是函数空间
到函数空间的满足线性性质和导子性质的算子. 为了把对函数的方向导数推广
成对切从的截面的方向导数 (也就是切向量场的方向导数). 又把三维欧式空间
的曲面曲线作为欧式空间的子集的几何的概念推广为流形上的子流形几何. 如
同矩阵的有理标准型和 Jordan 标准型定理一样, 为了让子流形有更好的和更简
单的坐标表示, 我们证明了浸入的标准型定理作为子流形的表示定理. 因为流形
上不仅有映射, 还有切向量场, 所以我们也从切向量场的角度研究了子流形, 首
先证明了非零向量场的局部标准型定理, 接着依据常微分的积分曲线的存在性
问题推广到积分流形的存在性问题, 借助 possion 括号积的性质, 证明了著名的
关于子流形可积性的 F robenius 定理. 布置了一道习题, 该习题是二维积分曲面
存在的具体的结论, 一方面我用具体的指标表示出来了该结论. 也用 Einstein
求和约定写出来了该结论.

6.1 截断函数
2019-11-13
本节要点/问题提示: 
 1
e (x−r2 )(x−r1 ) , x ∈ [r12 , r22 ].
2 2

1)、如何说明 g(x) = ? 的各阶导函数


 0, x < r12 或x > r22 .
存在且连续, 但 g(x) 的泰勒展开不收敛于自身?

101
6.1. 截断函数 CHAPTER 6. 子流形理论

2)、构造一个函数使得它在小球上的值为常数 1, 大球之外的区域为另一个
常数 0, 并且该函数还是光滑的?
3)、证明曳物线方程不能二阶光滑延拓?
4)、把 2) 中构造的函数如何搬到流形上面去? 先写出结论, 在给出证明?
5)、如何将流形上局部的光滑函数延拓为整体上的光滑函数?
6)、什么是局部紧空间? 什么是紧致邻域? 为什么说流形是局部紧空间?
回顾: 关于整体与局部的一些说明, 我们现在的这门微分几何又被称之为整
体微分几何. 与之相对比的是本科的局部微分几何. 整体与局部这个概念, 第一
次接触是在连续和一致连续的时候, 我们知道连续是一个逐点的概念, 而一致连
续是个整体的概念, 之所以这么说是因为连续的时候, 我们的 δ 的选取不仅与 ϵ
有关, 还与 x0 有关. 但在一致连续的第定义里面, 我们的 δ 是与 x0 的选取没有
任何关系的. 这就是一致性. 也是整体性, 同样的, 收敛和一致收敛也是. 所谓整
体, 是指的与局部的点的选取没有关系的数学概念. 而局部就是我们通常在一个
点或者一个邻域处的讨论结果.
本节的中心问题就是局部上的光滑函数如何延拓成整体上的光滑函数, 这
一节的主要证明与拓扑有一点点关系.

例 6.1. 曳物线方程

√ a a + a2 − x2
y = − a − x + ln(
2 2 √ )
2 a − a2 − x2
这样的函数在 (a, 0) 处不能二阶光滑. 将图像旋转便能得到伪球面/喇叭面 (伪

图 6.1: 曳物线图像

球面上的三角形内角和不再等于 π).

为了能够得到光滑函数, 我们从局部上的常值函数但整体上是光滑函数开
始讨论, 这便是下面的引理.

引理 6.1. 设 Br1 与 Br2 为 Rn 中的两个以原点为中心的同心开球, 则 Rn 上的


光滑函数 F (F ∈ C ∞ ), 使得

F |B(r1 ) = 1, F |Rn \B(r2 ) = 0.

102
CHAPTER 6. 子流形理论 6.1. 截断函数

1
证明. 注意到一个特殊的函数 e x2 (它是一个任意阶导数都存在但是在 0 附近的
泰勒展开却不收敛于它自身的函数), 进一步我们利用这个函数去构造我们引理
中所需要的函数.
考虑如下函数

 1
e (x−r2 )(x−r1 ) , x ∈ [r12 , r22 ].
2 2

g(x) =
 0, x < r12 或x > r22 .

可以证明 g(x) 的各阶导数都存在且连续. 于是 g(x) ∈ C ∞ (R), 令


∫ +∞
g(t)dt
G(x) = ∫x+∞
−∞
g(t)dt

显然 G(x) ∈ C ∞ (R) 且 0 ≤ G(x) ≤ 1. 若 n ≥ 2, 令 F (x) = G(| x |), 容易看出


F (x) ∈ C ∞ (Rn ) 并且

F | Br1 = 1, F |Rn \B(r2 ) = 0.

这便是我们所需要的函数.
{
e− x , x > 0
1

注解 6.1. 关于特殊的函数 f (x) = 的一些说明. 如果在 0


0, x ≤ 0.
处作 T aylor 展开, 通过导数的定义可以算出 f (x) 在 0 处的任意阶导数为
f (k) (0) = 0. 则 T aylor 展开式
1 (2) 1
f (0) + f (1) (0)x + f (0)x2 + f (3) (0)x3 + · · · .
2! 3!
显然是 0, 并不收敛于 f (x). 也就是说, 一个函数的 T aylor 展开式收敛到自身是
有条件的. 不仅如此, 在此提一点关于上述导数的计算的一些事实. 因为上述导
数的计算需要一定的技巧性, 而且通过计算可以说明 f (x) 是一个 R 上的光滑
函数.
注意到如下事实, 利用 L′ Hospital 法则, 对任意的实系数多项式 P (x), 都

lim P (x)e−x = 0.
x→+∞

于是有
1
lim P ( )e− x = 0.
1

x x→0+0

为了证明 f (x) 的光滑性, 我们需要对 x < 0 和 x ≥ 0 分别求导之后在 0 这个点


能光滑/连续的衔接起来. 通过归纳很容易证明
{
e− x Pn ( x1 ), x > 0
1
(n)
f (x) =
0, x ≤ 0.

103
6.1. 截断函数 CHAPTER 6. 子流形理论

其中
1
P0 ( ) = 1
x
1 1 ′ 1 1
Pn+1 ( ) = (Pn ( ) − Pn ( )) 2
t t t t
于是 Pn 是一个 2n 次多项式. 结合上述讨论知 f n (x) 存在且连续.

注解 6.2. 上述的证明是一个构造性证明, 一般而言, 数学里面的证明有两种, 一


种是构造性证明. 一种是存在性证明. 构造性证明会把具体的函数通过技巧给构
造出来, 而存在性证明是论述需要证明的东西的存在而不给出具体的形式.

下面在介绍一些拓扑的概念, 本节的证明过程中需要用到这些. 首先回顾紧


致性, 在拓扑学中的紧致是指的对拓扑空间 M 任意的开覆盖都有有限子覆盖
(比如数学分析中的有限覆盖定理用拓扑的语言说便是有限闭区间是紧致的). 当
然如果 M 是整个流形自然更好, 这便是紧致流形, 但往往很多拓扑空间 (或者
流形) 并没有这么好的性质, 于是我们退而求其次, 既然不是能拓扑空间 (流形)
整体, 那么自然就想先在局部上说某个邻域是紧致的. 最特殊的自然是欧式空间,
我们知道欧式空间的拓扑决定了紧致区域必然是有界闭区域. 局部上来说任意
一个有界闭区域都是紧致的, 但由于欧式空间本身并不有界, 自然欧式空间整体
并不紧致. 这启发我们定义如下概念.

定义 6.1. 拓扑空间 M 称为是局部紧致的, 如果任意的 p ∈ M, 存在 p 的邻域


U (p), 使得 U (p) 的闭包是紧致的.

我们不加证明的给出下列结论. 该结论的证明请参阅拓扑学教材.

定理 6.1. 设 M 是局部紧致的 Hausdorf f 空间, 则对任意的 p ∈ M 以及 p 的


任意邻域 U (p), 存在 p 的邻域 V , 使得 p ∈ V ⊆ V̄ ⊆ U (p).

上面所证明的引理是在欧式空间中的结论, 自然我们想得到流形上的相关
结果, 这便是下面引理.

引理 6.2. 设 M n 为光滑流形,U 和 V 是 M n 中的开集.Ū 紧致并且 Ū ∩ V̄ = ∅,


则存在 f ∈ C ∞ (M ) 使得 f |U = 1, f |V = 0.

证明. 对任意的 p ∈ U, 存在 p 处的局部坐标卡 (Xp , φp ) 使得 p ∈ Xp ⊆ M /V̄ ,


由于 M n 局部同胚于欧式空间, 则 M n 局部紧, 即 M n 的每一个点 p 处都存在
一个紧致邻域 (闭区域), 于是存在 p 的邻域 Up 和 Wp 使得

p ∈ Up ⊆ U¯p ⊆ Wp ⊆ W̄p ⊆ Xp ⊂ M /V̄ .

104
CHAPTER 6. 子流形理论 6.1. 截断函数

不失一般性, 假定 φp (Up ) 和 φp (Wp ) 是中心在原点的开球且 φp (Up ) ⊆ φp (Wp ),


由上述引理, 存在 F |φp (Up ) = 1, F |Rn /φ = 0. 且 F ∈ C ∞ (Rn ), 构造函数
p (Wp )

{
F (φp (x)), x ∈ Xp
fp (x) =
0, x ∈
/ Xp .

fp ∈ C ∞ (M n ) 显然只需要说明 x 在 Xp 的边界附近光滑即可说明 fp (x) ∈

图 6.2: 邻域图

C ∞ (M ), 事实上, 由于上述邻域的包含关系以及 F (x) 的光滑性知 f (x) 在 Xp


的边界附近肯定是光滑的. 进一步有 fp |Up = 1, fp |M n /W = 0. 考虑到 Ū 是紧致
p

的, 设 U1 , U2 , c · · · , Uk 为 Ū 的一个有限开覆盖. 相应的 fp 分别为 f1 , f2 , · · · , fk .


有 fi |Ui = 1, fi |M n /W = 0. 考虑函数 f = 1 − (1 − f1 )(1 − f2 ) · · · (1 − fk ). 对任
i

意的 p ∈ U, 存在 Ui0 使得 p ∈ Ui0 , 进一步 Ui0 上有 fi0 |Ui0 = 1, fi0 |M n /W = 0.


i0

于是 f |Ui0 = 1. 由 p 的任意性知 f |U = 1. 同样地考虑 p ∈ V 于是 p ∈


/ M n /V,
而 Wi ⊆ M n /V, 这说明 x ∈
/ Wi , 即 x ∈ M /Wi 对任意的 1 ≤ i ≤ k. 于是
x ∈ ∩M /Wi , 这说明 fi (x) = 0. 即 f (x) = 0. 于是 f |V = 0.

推 论 6.1. 设 U, V 为 M n 中 的 开 集 且 V̄ ⊆ U , 则 存 在 f ∈ C ∞ (M ) 使 得
f |V = 1, f |M /U = 0.

证明. 因 为 V̄ ⊆ U , 我 们 得 到 V̄ ∩ (M /U ) = ∅ 有 上 述 引 理 使 得 f |V =
1, f |M /U = 0 存在 (称这种函数为截断函数).

我们在上述引理的基础上, 可以回答之前的那个问题了, 如何把局部上的光


滑函数延拓到整体的光滑函数. 这便是下面的定理.

定理 6.2. 设 U 是 M n 中的一个开集,f ∈ C ∞ (U ), 存在 p 的邻域 V ⊆ U 以及


fb ∈ C ∞ (M ), 使得 fb |V = f |V .

证明. 对任意的 p ∈ U, 存在 p 的邻域 V 和 U 以及 f ∈ C ∞ (M ) 使得

p ∈ V ⊆ V̄ ⊆ W ⊆ W̄ ⊆ U.

105
6.2. 光滑向量场 CHAPTER 6. 子流形理论

由上述推论知, 存在 g |V = 1, g |M /W = 0. 令
{
d g(x)f (x), x∈U
f (x) =
0, x∈
/ U.

可仿造上述引理的证明画出邻域图可知 fb ∈ C ∞ (M ). 显然这样的 fb 满足
fb |V = f |V .

6.2 光滑向量场
2019-11-15(老师调课了所以时间变了)
本节要点/问题提示:
1)、切从的拓扑结构和微分结构分别是什么?
2)、切向量场的通俗定义和严格定义?
3)、如何从函数推广到截面?
4)、如何站在泛函的观点看待切向量场? 证明满足线性性和导子性的函数
空间的算子是切向量场的要点是什么?
6)、为什么要引入 P ossion 括号积? 如何自然的引入这个定义?
7)、如何将方向导数推广成 Lie 导数?Lie 导数有什么性质?
8)、什么是 Lie 群? 什么是 Lie 代数?
回 顾: 这 里 的 回 顾 主 要 是 想 要 和 大 家 一 起 从 切 向 量 走 向 切 向 量 场. 设

v = (f (x, y), g(x, y)) = f (x, y)] ∂x |p +g(x, y) ∂y

|p . 这是切向量, 什么是向量场
呢? 什么是切向量场? 回顾一下在物理里面的电磁场和重力场, 电磁场就是一些
曲线, 每一条曲线是光滑的, 所谓重力场是说的地球上每一个地方都有重力, 也
就是每一个点指定一个向量. 这便是通俗的对向量场的解释, 一个场地的向量,
也就是每一个点指定一个切向量. 但这种直观的理解是不够的, 我们需要做的事
情是用数学的语言把向量场给说清楚, 所以我们先回顾常微分方程中的一些结
果. 在 ODE 中, 给定一个平面区域 D 中的一些点 (x(t), y(t)), 考虑如下的常微
分方程组 {
dx
dt = f (x, y)
dy
dt = g(x, y)
由存在唯一性定理知, 在充分小的邻域中上述常微分方程的解是存在且唯一的.
它的解称之为积分曲线, 当出现奇点的时候, 上述积分曲线的形态各有千秋. 回
到我们之前的问题上, 我们想要说清楚什么是向量场, 考虑映射 p 7→ Tp M 的整
体化, 即 M 7→ X. 上面的 X 是什么东西? 使得该映射若在 M 上指定点 p,X 中
可以指定一个点 p 使得它退化为一个切空间? 注意到 M 不过是 p 放在一起在

106
CHAPTER 6. 子流形理论 6.2. 光滑向量场

给它一个微分流形结构, 受此启发我们定义 X 为 Tp M 放在一起得到的集合, 然


后想办法赋予一定的微分流形结构. 关于放在一起的直观理解: 把切空间看做一
条线 (一根纤维). 基于这样的思路, 我们引出如下定义. 也就是切空间的整体化:
切从. 但在此之前先把关于向量场的直观理解写成定义.

定义 6.2. 若 M n 作为光滑流形上每一点给定一个切向量, 则在 M 上给定了一


个切向量场. 即映射 ν 把任意的点 p ∈ M 映射成 ν(p) ∈ Tp M. 一般我们也简称
向量场.

为了表达 ν 的像空间, 我们基于上述回顾的思想定义集合 T M = ∪p∈M Tp M,


称之为切从. 我们先给出 T M 的拓扑结构, 在此基础之上给出 T M 的微分结构.
拓扑结构:
1)、定义投影映射 π : T M 7→ M , 具体来说 π(Xp ) = p,∀ Xp ∈ Tp M (或
者 写 为 π(Tp M ) = p). 设 (U, φ) 为 M 的 任 一 局 部 坐 标 系, 在 局 部 坐 标 下

Xp = y i ∂u i

2)、定义 (同胚) 映射 θ : π −1 (U ) = ∪p∈U Tp M 7→ U × Rn , 具体的表达式为

θ(Xp ) = (p, y 1 , y 2 , · · · , y n ), ∀ Xp ∈ Tp M.

显然 θ 是满射, 也是单射. 即 θ 是一一映射, 这个映射 θ 诱导了 π −1 (U ) 上的拓


扑, 使得该拓扑在 θ 下为同胚. 具体来说, 我们这么定义 T M 上的拓扑:
3)、定义拓扑: 设 {(Uα , φα )} 为 M 的微分结构.V 是 T M 中的开集当且仅
当 θα (V ∩ π −1 (Uα )) 为开集.
不难验证上述定义的 T M 拓扑是合理的. 并且在这个拓扑下 π 为开映
射,T M 是 A2 和 T2 的.
微分结构:
1)、定义同胚映射 (坐标映射): 设 {(Uα , φα )} 为 M 的微分结构, 考虑如下
复合映射
(φα , id) ◦ θα : π −1 (U ) 7→ R2n
((φα , id) ◦ θα ) (Xp ) = (φ(p), y 1 , y 2 , · · · , y n ), ∀ Xp ∈ Tp M.
在上述拓扑结构下该复合映射 (φα , id) ◦ θα 显然为同胚. 即 T M 的坐标映射, 于
{ }
是 π −1 (Uα ), (φα , id) ◦ θα 为 T M 的坐标覆盖.
2)、考虑转换映射的光滑性. 当 π −1 (Uα ) ∩ π −1 (Uβ ) ̸= ∅, 坐标转换函数

((φβ , id) ◦ θβ ) ◦ ((φα , id) ◦ θα )−1 : φα (Uα ∩ Uβ ) × Rn 7→ φβ (Uα ∩ Uβ ) × Rn .

具体写出来便是
( )
(x, a) 7→ φβ ◦ φ−1 −1 −1
α , J(φβ ◦ φα )(φα )(φα (x)a) , ∀ x ∈ φα (Uα ∩ Uβ ), a ∈ R .
n

107
6.2. 光滑向量场 CHAPTER 6. 子流形理论

因为 φβ ◦ φ−1 −1
α 是光滑的, 所以转换映射的 Jacobian 行列式 J(φβ ◦ φα ) 是光滑
的. 进一步上述的坐标转换函数是光滑的.
{ }
于是 π −1 (Uα ), (φα , id) ◦ θα 为 T M 的光滑结构 (微分结构).
这样我们就说明楚了什么是切从, 其中的 π 称为丛投影. 切从本身是切空间
的整体化. 我们引入他一方面是局部作成整体的需要, 另一方面也是我们上面的
ν 的像空间的需要.
从曲率的角度上看, 微分流形是欧式空间的推广. 欧式空间的数值函数就是
普通的函数, 流行上也有了函数的概念, 如同泛函分析中把函数推广为泛函和算
子一样. 我们也希望把函数推广为截面. 首先从我们站在其他的角度来看待函数,
希望借助这种角度来探索什么是函数的本质. 以此推广流形上的函数.
具体来说, 考虑 f : R 7→ R 且 f 光滑. 自然在 R2 上就是一条光滑曲线. 如
图所示 我们不在把函数的值域看成一个点, 我们把值域看做是定义域上每一点

图 6.3: 高观点下的函数图 f (x)

的切空间中的一个元素. 因为 R1 的切空间同胚于 R1 , 所以我们可以认为 y 是


x 处的切空间 Rx 处的一个切向量. 如果按照上述记号, 设 s : R1 7→ T R1 . 的一
个映射, 具体写出来就是 s(x) = Tx R = R1 (实际上可以写为 y × Tx R). 注意到
我们在上面提到过的投从投影,π(Tx R) = x. 于是便有了

π ◦ s(x) = x.

我们在这里自然会提一个反问题, 满足所有的 s : M 7→ T M 使得 π ◦ s = id :
R1 7→ R1 是否一定是函数呢? 如果不是到底是结论的问题还是函数的定义太狭
隘了? 我们能否从这里入手推广函数呢? 于是我们有如下定义.

定义 6.3. 设 s 是 M 7→ T M 的一个映射, 如果 π ◦ s = id : M 7→ M. 则称 s 为
T M 的一个截面.

108
CHAPTER 6. 子流形理论 6.2. 光滑向量场

直观理解, 截面是函数图的推广, 我们需要站在更高的观点去看函数, 如同


我们有了直观的切向量场的定义. 但我们也需要更高的观点去看切向量场. 其实
截面这个词很形象, 光滑的截出来一个我们需要的那个高维的” 面”. 切从的一个
截面就是切向量场. 这便是 (切) 向量场的另一种定义.

定义 6.4. 设 ν : M 7→ T M 为 C k 映射, 如果满足 ν ◦ π = id : M 7→ M. 则称 ν


为 M 上的 C k 向量场.

有了更高观点的定义, 我们还可以从泛函 (算子) 的角度去看, 之所以会有


这种 idea, 是因为我们之前用泛函的观点去看切向量, 而现在与切从紧密相关的
切向量场的定义是与切空间有关的. 因为切向量场就是在流形上每一点指定一
个切向量. 切向量能站在泛函分析的角度看, 自然我们也尝试用泛函分析的观点
看切向量场, 回顾一下之前泛函观点下的切向量是满足线性性质和导子性质的
泛函 v : C ∞ (M ) 7→ R. 不妨设 X 是切向量场, 则对任意的 p ∈ M, 有 X(p) 是 p
处的一个切向量, 作用于 f ∈ C ∞ 有 (X(p))(f ) ∈ R. 当 p 变化时, 上述每一点
p 指定了一个值, 在 X 的作用下 X(f ) 是另外一个光滑函数 X(f ) ∈ C ∞ (M ).
自然地我们用泛函的观点来看切向量场是一个满足线性性质和导子性质的算子
X : C ∞ (M ) 7→ C ∞ (M ). 但在这里涉及了光滑函数 (无穷次可微函数), 所以我
们还得来定义什么是向量场的可微性.

定义 6.5. 设 X 是一个向量场, 对任意的 f ∈ C ∞ (M ) 和固定的 p ∈ M. 定


义 X(f )(p) := Xp (f ), 其中 Xp 是 M 中 X 在 p 的切向量. 若对任意的函数
f ∈ C ∞ (M ), 都有 X(f ) 是光滑函数, 则称 X 是 M 上的可微向量场. 特别地,
若 X(f ) 是光滑的.

上述是不涉及局部坐标系的整体定义, 我们还可以在局部坐标系下来说明
这件事情, 因为任何切向量都可以写为自然基底的线性组合, 自然, 任何切向量
场也能写为自然基向量场的线性组合. 这启发我们如下定义 (切) 向量场的可微
性.

定义 6.6. 向量场 X 称为 C r 的, 如果对任意的 p ∈ M, 存在 p 附近的局部坐标


域 (U, ui ), 使得当 X 有局部表示

n

X |U = ξi
i=1
∂ui

时,ξi 在 p 处是 C r 的.

可以证明上述两种定义是等价的. 限于篇幅不在这里证明. 作为补偿这里给


出常见的一些向量场.

109
6.2. 光滑向量场 CHAPTER 6. 子流形理论

例 6.2. 基向量场
如果 ui 为邻域 U 的局部坐标系, 则


: U 7→ π −1 (U )
∂ui
∂ ∂
(p) = |p
∂ui ∂ui
显然它是局部上的光滑向量场.

例 6.3. 梯度场
考虑流形 Rn 的切从 T Rn , 因为 Tp Rn = Rn (Rn 的切空间就是它自身), 于
是 T Rn = Rn × Rn . 于是 Rn 上的向量场形如

X(p) = (p, Xp ), p ∈ Rn .

其中 Xp ∈ Tp Rn = Rn . 因此我们不妨就用通常的映射 Rn 7→ Rn 来表示 Rn 上
的向量场. 对任意的 f ∈ C ∞ (Rn ), 则 ∇f (x) = ( ∂u
∂f
1 , ∂u2 , · · · , ∂un ) 定义了一个
∂f ∂f

向量场, 称之为 f 的梯度场.

我们接着按照我们的思路继续探索. 有了单个的光滑向量场. 自然我们希望


把所有的光滑向量场放在一起作成一个集合, 并且赋予它某些好的结构. 这是从
局部过渡到整体最基本的想法. 当然, 最为简单的结构便是线性结构, 这是因为
切向量的全体具有线性结构. 自然我们希望这种结构对于光滑切向量场而言同
样的被保持.

命题 6.1. 流形 M 上的光滑向量场的集合记为 X(M ), 在 X(M ) 上可定义自然


的线性结构使其成为线性空间.

证明. 在流形 X(M ) 上定义如下的线性结构

(X + Y )(f ) := X(f ) + Y (f ), ∀ X, Y ∈ X(M ), f ∈ C ∞ (M ).

(λX)(f ) := λX(f ), ∀ X ∈ X(M ), f ∈ C ∞ (M ), λ ∈ R.

容易验证上述结构所构成的的确是一线性空间.

下面的命题刻画了切向量场的线性性质和导子性质.

命题 6.2. 设切向量场 v ∈ X(M ), 则 v : C ∞ 7→ C ∞ 满足下列两条性质.


1)、(线性性质)v(αf + βg) = αv(f ) + βv(g).
2)、(导子性质)v(f g) = f (p)v(g) + g(p)v(f ).

110
CHAPTER 6. 子流形理论 6.2. 光滑向量场

证明. 只需要逐点验证即可. 例如验证 2)

v(f g)(p) = vp (f g)
= f (p)vp (g) + g(p)vp (f )
= f (p)v(g)(p) + g(p)v(f )(p)
= (f v(g) + gv(f ))(p)

其他的情况可类似验证.

相应的, 我们要提一个反问题. 如果映射 v : C ∞ (M ) 7→ C ∞ (M ) 满足线性


性质和导子性质, 是否一定有 v ∈ X(M )? 回答是肯定的, 这便是下面的定理.

定理 6.3. 若 v : C ∞ (M ) 7→ C ∞ (M ) 满足下列两条性质.
1)、(线性性质)v(αf + βg) = αv(f ) + βv(g).
2)、(导子性质)v(f g) = f (p)v(g) + g(p)v(f ).
则 v ∈ X(M ), 即 v 是 M 上的向量场.

证明. 欲证 v ∈ X(M ), 只需要说明 v 在 M 上的任意一个点 p,v(p) 都指定


了 一 个 切 向 量 vp , 为 此 对 任 意 的 f ∈ C ∞ (M ), 令 vp (f ) := α(fb)(p), 其 中
p
fb ∈ C ∞ (M ), 并且存在 p 的邻域 V , 使得 fb |V = f |V . 我们说明上述定义是合
理的 (well − def ined). 事实上, 若还有 gb ∈ C ∞ (M ) 使得 fb |V = gb |V , 我们断言
v(fb)(p) = v(b
g )(p).
事实上, 由于 M 局部紧致, 于是存在 p 的邻域 V, W 使得 p ∈ W ⊆ W̄ ⊆ V,
其中 W̄ 紧致. 则存在截断函数 h, 使得 h |W = 1, h |M /V = 0. 注意到

h(fb − gb) = 0, h(fb − gb) ∈ C ∞ (M )

考虑
v(0) = v(0 + 0) = v(0) + v(0).

于是 v(0) = 0. 将 v 作用于 h(fb − gb) 有

v(h(fb − gb)) = v(h)(fb − gb) + h(v(fb − gb))


v(h(fb − gb)) = v(h)(fb − gb) + h(v(fb) − v(b
g )) = 0.

进一步
h(v(fb) − v(b
g )) = 0.

111
6.2. 光滑向量场 CHAPTER 6. 子流形理论

在 p 处取值, 注意到 h(p) = 1. 于是 v(fb)(p) = v(b


g )(p). 这说明断言成立. 即上
述定义是合理的, 注意到 vp : Cp∞ (M ) 7→ R. 并且 vp 根据定义显然满足线性
性质和导子性质. 于是 vp 是 p 处的一切向量. 这说明 v 是一光滑向量场. 即
v ∈ X(M )

注解 6.3. 上述定理和切向量的相关定理是可以对应的. 首先有了向量场的直观


定义: 每一点处指定一个切向量 (切向量的直观定义: 割线的极限), 在过渡到向
量场的数学严格定义 (切从的截面), 后又给出了向量场的性质命题: 线性性质和
导子性质. 切向量本身是曲线在某一点求导, 自然也具有线性性质和导子性质.
接着把向量场看成函数空间 v : C ∞ (M ) 7→ C ∞ (M ) 的算子, 对于切向量而言把
切向量看成泛函 vp : Cp∞ (M ) 7→ R.

注解 6.4. 从证明过程可以看出, 若 f ∈ C ∞ (U ), v |U (f ) = v(fb |U ). 由上述定


理的证明,v |U 与延拓 fb 的选取是无关的. 所以 v : C ∞ (U ) 7→ C ∞ (U ). 即 v 具
有局部性特别的, 若 (U, ui ) 为一坐标域, 则在局部坐标系下 v |U = v i ∂u

i,v ∈
i

C ∞ (U ).

有了光滑向量场, 显然两个光滑向量场相加仍然是光滑向量场, 两个向量场


的复合 (乘积) 是否是光滑向量场呢? 答案是否定的.

命题 6.3. X, Y ∈ X(M ), 定义 X ◦ Y (f ) := X(Y (f )), ∀ f ∈ C ∞ (M ), 则 X ◦ Y


不是光滑向量场.

证明. 要说明 X ◦ Y 不是向量场, 只需要说明他不满足线性性质或者导子性质.


下面我们一一验证即可. 线性性质, 对任意的 α, β ∈ R, 任意的 f, g ∈ C ∞ (M ).

(X ◦ Y )(αf + βg) = X(Y (αf + βg))


= X(αY (f ) + βY (g))
= αX(Y (f )) + βX(Y (g))
= αX ◦ Y (f ) + βX ◦ Y (g).

即线性性质成立. 下面考虑导子性质, 对任意的 f, g ∈ C ∞ (M ). 有

(X ◦ Y )(f g) = X(Y (f g))


= X(Y (f )g + f Y (g))
= X(Y (f )g) + X(f Y (g)))
= X(Y (f ))g + X(g)Y (f ) + X(f )Y (g) + f X(Y (g))
= X(Y (f ))g + f X(Y (g)) + X(g)Y (f ) + X(f )Y (g).

这说明导子性质不成立. 即 X ◦ Y 不是光滑向量场.

112
CHAPTER 6. 子流形理论 6.2. 光滑向量场

由此, 既然 X ◦ Y 不是光滑向量场. 我们就想办法定义一种乘法, 这种乘法


使得 X“乘”Y 仍然是光滑向量场. 下面我们就着手做这件事情.
注意到 (X ◦ Y )(f g) 中 X(g)Y (f ) + X(f )Y (g) 是多出来的而且关于 X, Y
是对称的, 我们想要消去这一项. 自然会想到把 X, Y 调换位置. 注意到下面两个
式子

(X ◦ Y )(f g) = X(Y (f ))g + f X(Y (g)) + X(g)Y (f ) + X(f )Y (g). (6.1)

(Y ◦ X)(f g) = Y (X(f ))g + f Y (X(g)) + Y (g)X(f ) + Y (f )X(g). (6.2)

两个式子相减有

(X ◦ Y )(f g) − (Y ◦ X)(f g) = X(Y (f ))g + f X(Y (g)) − (Y (X(f ))g + f Y (X(g)))


= f (X(Y (g)) − X(Y (g))) + g(X(Y (f )) − Y (X(f ))).
(6.3)
受上述式子启发, 我们定义

[X, Y ] = X ◦ Y − Y ◦ X

自然的有

[X, Y ] (f ) = X ◦ Y (f ) − Y ◦ X(f ) = X(Y (f )) − Y (X(f )).

特别的, 对任意的 p ∈ M, 有

[X, Y ]p (f ) = Xp ◦ Yp (f ) − Yp ◦ Xp (f ) = Xp (Yp (f )) − Yp (Xp (f )).

根据 (6.3) 式, 有

[X, Y ] (f g) = f [X, Y ] (g) + g [X, Y ] (f ).

这说明 [X, Y ] 满足导子性质. 我们希望 [X, Y ] 是一个光滑向量场, 自然希望它


满足线性性质. 下面便验证线性性质.

[X, Y ] (αf + βg) = (X ◦ Y − Y ◦ X)(αf + βg))


= (X(Y )(αf + βg) − Y ((X)(αf + βg))
= (X(Y )(αf + βg) − Y ((X)(αf + βg))
= αX(Y (f )) + βX(Y (g)) − αY (X(f )) − βY (X(g))
= α(X(Y (f )) − Y (X(f ))) + β(X(Y (g)) − Y (X(g)))
= α [X, Y ] (f ) + β [X, Y ] (g).

这就证明了下述命题.

113
6.2. 光滑向量场 CHAPTER 6. 子流形理论

定义 6.7. 设 X, Y ∈ X(M ), 定义如下的称之为 P ossion 括号积

[X, Y ] = X ◦ Y − Y ◦ X

命题 6.4. 设 X, Y ∈ X(M ), 则 [X, Y ] ∈ X(M ).

由于 X 是全体光滑向量场, 向量场是在流形的每一点处指定了一个切向量.
方向导数是某个函数对某个向量求导, 现在我们已经把流形上的函数推广成了
截面, 函数是定义域上的切从的截面, 把定义域换成流形, 流形的切从的截面便
是向量场. 于是我们想把之前方向导数是某个函数对某个向量求导推广成某个
向量场对某个向量求导. 这就是 Lie 导数的概念.
先站在更高的观点考虑函数的微分, 设函数 f : M 7→ R, f ∈ C ∞ (M ). 对任
意的 X ∈ X(M ). Lie 导数的记号为 (LX f ), 对任意的 p ∈ M, 有 (既然 Lie 导
数是方向导数的推广, 对于普通的函数它自然也是方向导数, 根据方向导数的定
义, 我们有)
(LX f )(p) = ∇f · Xp
∂f
= Xi
∂xi
∂f
= Xj i δji
∂x
∂f ∂
= Xj i dxi ( j )
∂x ∂x
∂ ∂f
= (Xj j )( i dxi )
∂x ∂x
= Xp (dfp )
= (Xp (f ))(p)
刚刚对 p 是任意的, 于是 LX f = X(f ), 注意到

[X, f ] = [X, 1 · f ] = 1 · X(f ) + f (X(1)) = X(f ) = (LX f ).

受此启发, 我们能否对任意的向量场 X, 定义 LXp Y := [X, Y ] |p ? 这就引出了下


列定义.

定义 6.8. 定义向量场 Y 沿着 Xp 方向的 Lie 导数

LXp Y := [X, Y ] |p .

为了引出 Lie 代数及其性质, 我们首先说明什么是 Lie 群, 李群既具有群结


构, 又是微分流形, 并且群运算也是 C r 的.Lie 代数则是 Lie 群在单位元处的切
空间, 外加一个满足反交换性、双线性、Jacobi 恒等式的代数运算.

114
CHAPTER 6. 子流形理论 6.3. 子流形

性质 6.1. 1)、(反称性)[X, Y ] = − [Y, X] .


2)、(双线性性)[a1 X1 + a2 X2 , b1 Y1 + b2 Y2 ] = a1 b1 [X1 , Y1 ] + a1 b2 [X1 , Y2 ] +
a2 b1 [X2 , Y1 ] + a2 b2 [X2 , Y2 ] .
3)、(Jacobi 恒等式)[[X, Y ] , Z] + [[Y, Z] , X] + [[Z, X] , Y ] = 0. 故 [, ] 是 Lie
代数.

证明. 显然自行验证即可.

例 6.4. (R3 , ×) 是三维李代数.

例 6.5. (GL(n), AB − BA) 是矩阵李代数.

6.3 子流形
2019-11-27
本节要点/问题提示:
1)、反函数定理的内容? 应用于流形可得到怎样的结论?
2)、什么是光滑映射的秩? 如何证明它的定义是合理的?
3)、什么是浸入? 什么是 入? 什么是淹没?
4)、什么是临界点? 什么是正则点?
6)、什么是标准浸入? 浸入的局部标准型定理的内容是什么?
7)、如何证明浸入的标准型定理? 要点在哪里?
8)、向量场的局部标准型定理的内容是什么?
8)、如何证明浸入的标准型定理? 要点是什么?
导语: 有了流形, 我们自然希望得到这个流形的子流形的相关结果和性质.
以曲面和曲线为例, 以三维欧式空间为流形, 曲面和曲线为子流形. 但我们知道
在古典微分几何之中本来就是通过研究映射来研究曲线曲面的, 现在也按照相
同的思路来研究子流形. 因为研究映射比研究流形要来的简单, 所以我们希望通
过研究一些映射的性质来研究子流形. 所以我们先回顾反函数和隐函数定理, 并
以此为工具为得到了子流形的标准型定理. 但流形上不仅有映射, 还有切向量场,
于是我们也研究了非零切向量场在局部的标准型定理.
回顾: 先回顾反函数定理, 设 F : U ⊆ Rn 7→ Rn 的函数 F (x1 , x2 , · · · , xn ) =
(F 1 (x1 , x2 , · · · , xn ), F 2 (x1 , x2 , · · · , xn ), · · · , F n (x1 , x2 , · · · , xn )), 其中 (x1 , x2 , · · · , xn ) ∈
∂(F 1 ,F 2 ,··· ,F n )
U. 若 p 处的 Jacobian 行列式 ∂(x1 ,x2 ,··· ,xn ) ̸= 0, 记 y i = F i (x1 , x2 , · · · , xn ),
则 由 反 函 数 定 理 知, 存 在 p 的 邻 域 V ⊆ U. 使 得 F 在 V 上 有 反 函 数, 即
F −1 (y 1 , y 2 , · · · , y n ) = (x1 , x2 , · · · , xn ). 即存在 G1 , G2 , · · · , Gn ∈ C(Rn ) 使得
xi = Gi (x1 , x2 , · · · , xn ). 且反函数 G = F −1 = G = (G1 , G2 , · · · , Gn ) 在 F (p)

115
6.3. 子流形 CHAPTER 6. 子流形理论

的邻域 F (V ) 也是连续的, 特别地, 若 F 光滑, 则 G 也光滑, 于是 F : V 7→ F (V )


的微分同胚. 当然, 与反函数定理相对的有隐函数定理, 并且两者可以相互推出.
有了反函数定理, 结合微分几何基本思想: 利用映射来研究空间形式. 进一
步我们得到下面的定理.

定理 6.4. 设 F : M n 7→ N n 是光滑流形之间的光滑映射, 对任意的 p ∈ M, 存在


p 附近的邻域容许坐标卡 (U, φ) 和 F (p) 附近的容许坐标卡 (V, ψ), 则 F 的局
部表示 F̄ = ψ ◦ F ◦ φ−1 : φ(U ) 7→ ψ(V ) 且 F (U ) ⊆ V. 记 F̄ (x1 , x2 , · · · , xn ) =
∂(y 1 ,y 2 ,··· ,y n ) ′
(y 1 , y 2 , · · · , y n ), p ∈ M, 若 ∂(x1 ,x2 ,··· ,xn ) |φ(p) ̸= 0, 则存在 p 的邻域 U ⊆ V 使
′ ′
得 F : U 7→ F (U ) ⊆ V 为微分同胚.

我们想定义光滑映射是否退化, 由于线性变换是否退化与某个基下的矩阵
是否满秩有关, 而且秩与基的选取是无关的, 根据这种思路, 首先我们已经有了
F 的局部坐标表示 F̄ , 我们想要借助 F̄ 构造一个矩阵, 自然我们会想到 Jacobi
矩阵. 由此我们引出如下定义.

定义 6.9. 设 F : M m 7→ N n 为光滑映射,F̄ = (y 1 , y 2 , · · · , y n ) = (F 1 , F 2 , · · · , F n )
为 F 的局部表示 ψ ◦ F ◦ φ−1 . 则 F̄ 的 Jacobi 矩阵
 1 
∂F 1 ∂F 1
∂F
···
 ∂F
∂x1 ∂x2 ∂xm

∂F i  1 2
∂F 2
··· ∂F 2 
 ∂x ∂x2 ∂xm 
|φ(p) =  . . .. .. 
∂xj  .. .. . . 
 
∂F n ∂F n ∂F n
∂x1 ∂xm ··· ∂xm

∂F i
于是 ∂xj 在 φ(p) 处的秩称之为 F 在 p 处的秩.

下面的命题回答了上面的定义是否是有意义的.

命题 6.5. F 在 p 处的秩与局部坐标映射 φ 和 ψ 的选取无关.

证明. 由转换映射的光滑性经过简单计算即可得到此结论. 事实上, 设 f 有另外


一局部坐标表示 F¯1 = ψ1 ◦ F ◦ φ−1 . 注意到 1

F¯1 = ψ1 ◦ F ◦ φ−1
1 = ψ1 ◦ ψ
−1
◦ ψ ◦ F ◦ φ−1 ◦ φ ◦ φ−1
1 = ψ1 ◦ ψ
−1
◦ F̄ ◦ φ ◦ φ−1
1 .

相应的 Jacobi 为
∂ F¯1i ∂(ψ1 ◦ ψ −1 ◦ F̄ ◦ φ ◦ φ−11 )
j
=
∂xj ∂xi
∂(ψ1 ◦ ψ ◦ F̄ ) ∂(φ ◦ φ−1
−1
1 )
l
= l i
∂u ∂x
∂(ψ1 ◦ ψ −1 )j ∂ F̄ k ∂(φ ◦ φ−11 )
l
=
∂v k ∂ul ∂xi

116
CHAPTER 6. 子流形理论 6.3. 子流形

写成矩阵的形式. 我们得到 P = AQB 由于转换函数的光滑同胚性知 A, B 是非


退化的, 于是 rank(P ) = rank(AQB). 即秩与坐标的选取无关.

有了上述命题, 按照线性代数的思路, 我们接下来得定义什么是满秩, 什么


是临界点.

定义 6.10. 若 rank(F ) |p = min {m, n}, 则称 F 在 p 处满秩, 若 rank(F ) |p <


min {m, n}, 则称 p 为 F 的临界点.

为了介绍映射正则点的概念, 我们先引入子流形, 当某一个流形是另外一个


流形的子流形的时候, 他们不一定处于相同的空间位置上面, 也就是并不一定有
一个包含另外一个的坐标表示. 但是我们可以通过适当的映射, 把一个流形完全
的映射到另外一个流形的一部分上. 这就是浸入子流形 (即把某个流形放到另外
一个像水一样的流形里面能放进去, 放的这个动作便是上述的映射) 另一方面,
一个流形可以完全的被另外的一个流形所淹没. 这启发我们引入如下子流形的
定义.

定义 6.11. 若 F : M m 7→ N n 为光滑映射, 若对任意的 p ∈ M , 有 rank(F ) |p =


m, 则称 F 为浸入, 并且称 M 为 N 的浸入子流形 (这时必然有 n ≥ m). 特别
地, 若 F : M 7→ F (M ) ⊆ N 的同胚, 则称 M 为 N 的正则子流形. 若对任意的
p ∈ M , 有 rank(F ) |p = n, 则称 F 为淹没 (这时必然有 n ≤ m).

下面引入正则点的概念. 映射正则点是微分流形上一类特殊的点, 指微分流


形上的那种点, 可微映射 F 在该点处是淹没映射.

定义 6.12. 设 F : M m 7→ N n 是光滑映射, 其中 n ≤ m, 对 p ∈ M , 若 F 在 p 处
是淹没, 即 rank(F ) |p = n, 则称 p 为 F 的正则点.

显然, 一个点不可能既是临界点, 又是正则点.


有了浸入, 从线性代数的角度上讲, 矩阵有有理标准型和 Jordan 标准型,
也就是说矩阵有着某种性质非常好的表示, 泛函分析中也有 Resiz 表示定理, 所
以我们希望这种好的性质的表示也能用于研究流形到流形之间的映射上, 一般
的映射而言没有什么特别好的表示定理/标准型定理, 但对于淹没和浸入而言,
都有相应的标准型定理. 即任何一个浸入都有比较好的表示. 这便是下面的定理.
但在给出定理之前, 先给出标准浸入的例子.

例 6.6. F : Rm 7→ Rn , m ≤ n,F 为光滑映射, 具体的表达式为

F (x1 , x2 , · · · , xm ) = (x1 , x2 , · · · , xm , 0, · · · , 0).

称之为标准浸入.

117
6.3. 子流形 CHAPTER 6. 子流形理论

下面的定理说明了任何一个浸入都可以在局部上表示为标准侵入的形式.
称之为浸入的局部标准型定理.

定理 6.5. F : M m 7→ N n 为光滑映射, 其中 n ≥ m,rank(F ) |p = m. 则存在 p 的


局部容许坐标系 (U, xi ) 和 F (p) 的局部容许坐标系 (V, y i ), 使 F (U ) ⊆ V 有局
部表示
F (x1 , x2 , · · · , xm ) = (x1 , x2 , · · · , xm , 0, · · · , 0).

证明. 设 F 的 局 部 表 示 F̄¯ = ψ ◦ F ◦ φ−1 为 y i = F i (x1 , x2 , · · · , xm ), i =


1, 2, · · · , n. 由 p ∈ M 知存在 p 的邻域 U , 其中上述 (x1 , x2 , · · · , xm ) ∈ φ(U ),p ∈
U, 因 F 在 p 处是满秩的, 不妨设 φ(p) = 0,

∂(F 1 , F 2 , · · · , F m )
|φ(p) ̸= 0.
∂(x1 , x2 , · · · , xm )

定义映射 F̄ : φ(U ) × In−m 7→ F̄ (φ(U ) × In−m ) = ψ(V ) ⊆ Rn . 其中 In−m =


{ m+1 m+2 ∑n }
(x ,x , · · · , xn ) | i=1 (xi )2 < δ, δ > 0 具体写出来便是

F̄ = (F̄ 1 , F̄ 2 , · · · , F̄ n ) = (y 1 , y 2 , · · · , y n ).

其中 y i = F i (x1 , x2 , · · · , xm ), i = 1, 2, · · · , m, y i = F i (x1 , x2 , · · · , xm ) + xi , i =
m + 1, m + 2, · · · , n 注意到
[( ) ]
∂F i ∂Fi
∂xj l×l ∗
( j )n×n =
∂x 0 In−l
′ ′ ′
∂ F¯i
显然 ∂xj ̸= 0, 由前面的定理知, 存在 U ⊆ U, In−m ⊆ In−m 使得 F̄ : φ(U ) ×
′ ′ ′ ′ ′
In−m 7→ F̄ (φ(U ) × In−m ) = ψ(V ) 为微分同胚. 将 (V , F̄ )−1 ◦ ψ 作为 F (p) = q
处的坐标卡, 注意到 F 新的局部坐标表示为

Fb = (F̄ )−1 ◦ ψ ◦ F ◦ φ−1 = (F̄ )−1 ◦ F̄¯

于是
Fb(x1 , x2 , · · · , xm ) = (F̄ )−1 ◦ F̄¯ (x1 , x2 , · · · , xm )
= (F̄ )−1 ◦ F̄ (x1 , x2 , · · · , xm , 0, · · · , 0)
= (x1 , x2 , · · · , xm , 0, · · · , 0).

上述定理的几何意义实际上可以说成 F 的局部表示可以把 φ(U ) 中的曲线


变成 ψ(V ) 中的直线, 一个例子便是圆上去掉一个点作为曲线用参数表示可以映

118
CHAPTER 6. 子流形理论 6.3. 子流形

射为 [0, 2π)(作为直线). 微分几何利用微积分为工具之后始终要回到其几何意义


上面. 我们刚刚从流形上的映射的角度研究了子流形, 现在我们想从流形上的切
向量场的角度来研究子流形, 前面的那个定理说明了任何一个浸入与标准浸入
之间的关系, 对于切向量场, 我们同样想要研究任何一个非零向量场与标准切向
量场之间的关系, 根据这个思路, 我们有如下的称之为向量场的局部标准型定理.

定理 6.6. 设 X ∈ X(M ), 若点 p ∈ M, Xp ̸= 0, 则存在 p 附近的坐标邻域 (V, y i )


使得 X |V = ∂
∂y 1

分析:首先, 从局部坐标上看, 我们想办法 X |U = ξ i ∂x



i (不妨设 ξ
1
̸= 0.) 若
U ∩ V ̸= ∅, p ∈ U ∩ V, 有带入

∂ ∂
ξi = ,
∂xi ∂y 1

注意到基坐标转换公式
∂ ∂xi ∂
=
∂y 1 ∂y 1 ∂xi

带入上式得
∂ ∂ ∂xi ∂
ξ i
= 1
= .
∂x ∂y ∂y 1 ∂xi

于是
∂xi
= ξ i = ξ i (x1 , x2 , · · · , xm ).
∂y 1

从 ODE 的角度给出上式方程的解的存在性, 以此证明结论.

证明. 考虑如下常微分方程组
{
dxi
dt = ξ i (x1 , x2 , · · · , xm ), 1 ≤ i ≤ n.
x1 (0) = 0, xα = y α , 2 ≤ α ≤ m.

用向量的形式写出, 即
{
dx
dt =ξ
x(0, 0, · · · , 0) = (0, y , · · · , y m ).
2

由解的存在唯一性定理知, 存在 p 的邻域 U 使得 xi = xi (t, y 2 , · · · , y m ) 满足上


述常微分方程组. 并且由解对初值的光滑依赖性, 则解关于 (0, y 2 , · · · , y m ) ∈ U

119
6.3. 子流形 CHAPTER 6. 子流形理论

是光滑的, 考虑 xi = xi (y 1 , y 2 , · · · , y m ) 在 0 处的 Jacobian 矩阵
 
∂x1 ∂x1 ∂x1
∂y 1 ∂y 2 ··· ∂y m
 
∂(x1 , x2 , · · · , xm )  ∂x2 ∂x2
··· ∂x2 
 ∂y 1 ∂y 2 ∂y m 
| 0 =  . .. .. ..  |0
∂(y 1 , y 2 , · · · , y m )  .. . . . 
 
∂xm ∂xm ∂xm
dy 1 ∂y 2 ··· ∂y m
 
dx1 ∂x1 ∂x1
dy 1 ∂y 2 ··· ∂y m
 
 dx2 ∂x2
··· ∂x2 
 dy 1 ∂y 2 ∂y m 
= . .. .. ..  |0
 .. . . . 
 
dxm ∂xm ∂xm
dy 1 ∂y 2 ··· ∂y m
 
∂x1 ∂x1
ξ1 ∂y 2 ··· ∂y m
 2 
ξ ∂x2
··· ∂x2 
 ∂y 2 ∂y m 
= . .. .. ..  |0
 .. . . . 
 
∂xm ∂xm
ξn ∂xm ··· ∂y m
 
ξ1 0 ··· 0
 2 
ξ 1 ··· 0
 
= . .. .. .. 
 .. . . .
 
ξn 0 ··· 1

由 假 设 ξ 1 ̸= 0, 于 是 Jacobi 行 列 式 不 等 于 0, 由 反 函 数 定 理 知 存 在 y i =
y i (x1 , x2 , · · · , xn ). 此时

∂ ∂xi ∂ ∂
X |V = ξ i i
= 1 i
= .
∂x ∂y ∂x ∂y 1

接下来我们通过切向量场来研究子流形, 如同我们在 ODE 里面的积分曲


线的存在性一样: 给定一些连续的切向量场 (即在某些点处给定切向量), 问是否
存在这样的积分曲线, 使得该曲线每一点的切向量都是上述指定的切向量? 这是
一维的情形, 同样的, 在空间中指定两个切向量场, 是否存在一个积分曲面, 使得
该曲面每一点的切空间都是有上述两个切向量场在该点处的切向量所张成的?
由于微分流形是可微曲线和曲面的高维推广, 我们自然会问高维的情况是怎样
的? 是否有类似积分曲线的存在唯一性的一个定理呢? 积分流形是否一定存在
呢? 存在的条件是什么? 我们下一节将回答这个问题.

120
CHAPTER 6. 子流形理论 6.4. FROBENIUS 定理

6.4 Frobenius 定理
2019-12-4
本节要点/问题提示:
1)、P ossion 括号积有哪些性质? 在局部坐标系下的表达式是什么?
2)、P ossion 括号积与切映射可交换吗? 如何证明?
3)、积分曲面的存在性问题如何叙述? 高维的叙述又是怎样?
4)、积分曲面的存在能够推出怎样的条件?
6)、什么是分布? 什么是 F robenius 条件?
7)、什么是同余式? 满足 F robenius 条件的一定可积吗?
8)、可积的一定满足 F robenius 条件吗?
8)、如何证明老师布置的习题?
回顾:P ossion 括号积相关的性质, 对任意的 X, Y ∈ X(M ), 定义 [X, Y ] =
X ◦ Y − Y ◦ X. 称之为 P ossion 括号积, 它具有如下性质:
1)、(反称性)[X, Y ] = − [Y, X] .
2)、(双线性性)[a1 X1 + a2 X2 , b1 Y1 + b2 Y2 ] = a1 b1 [X1 , Y1 ] + a1 b2 [X1 , Y2 ] +
a2 b1 [X2 , Y1 ] + a2 b2 [X2 , Y2 ] .
3)、(Jacobi 恒等式)[[X, Y ] , Z] + [[Y, Z] , X] + [[Z, X] , Y ] = 0. 故 [, ] 是 Lie
代数. 我们在给出一些 P ossion 括号积常用的性质.

性质 6.2. 1)、[X, f Y ] = X(f )Y + f [X, Y ] .


2)、[f X, Y ] = −Y (f )X − f [Y, X] .
2)、[f X, gY ] = f X(g)Y − gY (f )X + f g [X, Y ] .

证明. 1)、[X, f Y ] = X(f Y ) − f Y (X) = X(f )Y + f X(Y ) − f Y (X) = X(f ) +


f (X(Y ) − Y (X)) = X(f )Y + f [X, Y ] .
2)、[f X, Y ] = − [Y, f X] = −Y (f )X − f [Y, X] .
3)、[f X, gY ] = f X(g)Y + g [f X, Y ] = f X(g)Y + g(−Y (f ) − f [Y, X]) =
f X(g)Y − gY (f )X − f g [Y, X] .

注解 6.5. 有了 P ossion 括号积的抽象性质, 我们现在想在具体的情况下是怎


[ ∂ ]
i , ∂xj , 其中 i ̸= j, 对任意的

样的, 特别地, 在局部坐标系 (U, xi ) 下, 考虑 ∂x
[ ] ∂2f ∂2f
f ∈ C ∞ (M ), ∂x
∂ ∂
i , ∂xj

(f ) = ∂x i ( ∂xj ) − ∂xj ( ∂xi ) = ∂xi ∂xj − ∂xj ∂xi = 0. 于是
∂f ∂ ∂f
[ ∂ ∂
]
∂xi , ∂xj = 0. 这是特殊的情况, 一般的光滑向量有局部表示, 我们下面在局部
下计算 P ossion 括号积.

问题 6.1. 设 X, Y ∈ X(M ), 在局部坐标系 (U, xi ) 下有 X = ξ i ∂x


∂ i ∂
i , Y = ξ ∂xi .

求 [X, Y ] |U 的表达式.

121
6.4. FROBENIUS 定理 CHAPTER 6. 子流形理论

[ ∂ ]
证明. 在局部坐标系 (U, xi ) 下有,[X, Y ] |U = [X |U , Y |U ] = ξ i ∂x j ∂
i , η ∂xj =
2 j 2 i
i (η ∂xj ) − η ∂xj (ξ ∂xi ) = ξ η ∂xi ∂xj + ξ ∂xi − η j ξ i ∂x∂j ∂xi − η j ∂x
∂ j ∂ j ∂ i ∂ i j ∂ i ∂η ∂ ∂ξ ∂
ξ i ∂x ∂xj j ∂xi =
j i j j
ξ i ∂η ∂
∂xi ∂xj − ∂ξ ∂
η j ∂x j ∂xi = (ξ i ∂η
∂xi − ∂ξ
η i ∂x ∂
i ) ∂xj .

上面是 P ossion 括号积与函数的性质, 我们还得介绍一下 P ossion 括号


积更为重要的性质, 刚刚是一般的函数, 我们想研究 P ossion 括号积的可交换
性,P ossion 括号积与一般的映射是不能交换的, 但是却在和一种特殊的映射可
交换: 即光滑映射的切映射是可交换的. 这实际上与切映射是线性映射有关.

命题 6.6. 对任意的 X, Y ∈ X(M ), F : M 7→ N 是光滑映射. 则 F⋆ [X, Y ] =


[F⋆ X, F⋆ Y ] .

证明. 对任意的 f ∈ C ∞ (N ), p ∈ M, q = F (p), 规定 (F⋆ X)q=F (p) = F⋆ Xp , 进一


步 (F⋆ [X, Y ])(q) = F⋆ ([X, Y ] |p ). 于是

(F⋆ [X, Y ])(q)(f ) = (F⋆ ([X, Y ] |p ))(f )


= ([X, Y ] |p )(f ◦ F ).

另一方面, 注意到对任意的 x ∈ M, 有

((F⋆ Y )(f ) ◦ F )(x) = ((F⋆ Y )(f ) ◦ F )x


= ((F⋆ Y )(f ))(F (x))
= ((F⋆ Y )(f ))F (x)
= F⋆ (Yx )(f )
= Yx (f ◦ F )
= (Y (f ◦ F ))(x)

这说明 ((F⋆ Y )(f ) ◦ F ) = Y (f ◦ F ). 于是

[F⋆ X, F⋆ Y ]q (f ) = ((F⋆ X)(F⋆ Y ) − (F⋆ Y )(F⋆ X))q (f )


= (F⋆ X)q ((F⋆ Y )(f )) − (F⋆ Y )q ((F⋆ X)(f ))
= (Xp )((F⋆ Y )(f ) ◦ F ) − Yp ((F⋆ X)(f ))
= Xp (Y (f ◦ F )) − Yp (X(f ◦ F ))
= (Xp (Y ) − Yp (X))(f ◦ F )
= ([X, Y ] |p )(f ◦ F )
= (F⋆ [X, Y ])(q)(f ).

即 F⋆ [X, Y ] |q (f ) = [F⋆ X, F⋆ Y ] |q (f ). 由 f 的 任 意 性 知 F⋆ [X, Y ] |q =


[F⋆ X, F⋆ Y ] |q . 再由 q 的任取性知 F⋆ [X, Y ] = [F⋆ X, F⋆ Y ] .

122
CHAPTER 6. 子流形理论 6.4. FROBENIUS 定理

有了 P ossion 括号积的性质, 我们回到之前的问题上, 可积子流形的存在性


问题在一维的时候就是积分曲线的存在性问题, 这个我们在常微分方程里面已
经有所涉猎, 所以我们转向二维的积分曲面的存在性问题, 即给定定义在某些点
处的切空间, 问是否存在这样的曲面, 使得该曲面在曲面上的点处的切空间恰好
是我们给定的切空间? 这便是下面的例子.

例 6.7. 设 γ : D ⊆ R2 7→ R3 , 其中 (u, v) ∈ D, γ(u, v) 是曲面 γ(D) 上的点, 为


方便叙述, 把该曲面放入三维空间中,r(u, v) = (x(u, v), y(u, v), z(u, v)). 考虑其
偏导, 我们有
∂γ(u, v) ∂x(u, v) ∂y(u, v) ∂z(u, v)
=( , , ) = (xu , yu , zu )
∂u ∂u ∂u ∂u
∂γ(u, v) ∂x(u, v) ∂y(u, v) ∂z(u, v)
=( , , ) = (xv , yv , zv ).
∂v ∂v ∂v ∂v
之前的流形之间的映射有切映射, 自然 γ : D 7→ γ(D) 作为流形之间的映射也具
有切映射 γ⋆ . 根据前面的讨论,D 中的自然坐标 (u, v) 作成自然坐标系的切向量
∂ ∂
∂u , ∂v , 则 γ⋆ 把上述两个切向量映射为曲面 γ(D) 上的切向量, 而曲面放在三维
欧式空间中, 于是有
∂ ∂ ∂ ∂
γ⋆ ( ) = xu + yu + zu .
∂u ∂x ∂y ∂z
∂ ∂ ∂ ∂
γ⋆ ( ) = xv + yv + zv .
∂v ∂x ∂y ∂z
形式上的, 我们记  
[ ] [ ] ∂

xu yu zu  ∂x 
γ⋆ ( ∂u
)= ∂

xv yv zv  ∂y 
∂v ∂
∂z

这是正面的对曲面求导. 现在我们提出反问题: 对于 R3 中的给定的两个光滑向


量场 X1 和 X2 , 在基下的表示有
 

 ∂x 
X1 = (f11 (x, y, z), f21 (x, y, z), f31 (x, y, z))  ∂ 
 ∂y 

∂z
 

 ∂x 
X2 = (f1 (x, y, z), f2 (x, y, z), f3 (x, y, z)) 
2 2 2 ∂ 
 ∂y 

∂z

我们有下述问题.

123
6.4. FROBENIUS 定理 CHAPTER 6. 子流形理论

问题 6.2. 记号同上, 如果存在曲面 Σ = γ(D) ∈ R3 使得 Σ 在每一个点处的切


平面 Tp Σ = span {X1 , X2 } , 则 X1 , X2 应该满足怎样的条件?

证明. 还是放在三维坐标系中来讨论, 由于 Tp Σ = span {X1 , X2 } , 于是 γ⋆ ( ∂u



)

和 γ⋆ ( ∂v ) 可以由 X1 和 X2 线性表出, 进一步
{

γ⋆ ( ∂u ) = a11 X1 + a21 X2 .

γ⋆ ( ∂v ) = a12 X1 + a22 X2 .

a1 a2
1
其中 a11 , a12 , a21 , a22 均为 Σ 上的光滑函数. 并且 11 ̸= 0. 根据前面的 P ossion
a2 a22
[∂ ∂]
括号积的性质,P ossion 括号积可以与切映射交换, 并且注意到 ∂u , ∂v = 0. 于
[ ∂ ∂
] [∂ ∂]
是 γ⋆ ( ∂u ), γ⋆ ( ∂v ) = γ⋆ ( ∂u , ∂v ) = γ⋆ (0) = 0. 带入上式有
[ 1 ]
a1 X1 + a21 X2 , a12 X1 + a22 X2 = 0.

利用 P ossion 括号积的性质 [f X, gY ] = f X(g)Y − gY (f )X + f g [X, Y ] 展开上


述式子,
[ 1 ] [ ] [ ] [ ]
a1 X1 , a12 X1 + a11 X1 , a22 X2 + a21 X2 , a12 X1 + a21 X2 , a22 X2 = 0.

进一步展开有

a11 X1 (a12 )X1 − a12 X1 (a11 )X1 + a11 a12 [X1 , X1 ] + a11 X1 (a22 )X2 − a22 X2 (a11 ) + a11 a22 [X1 , X2 ] +
a21 X2 (a12 )X1 − a12 X1 (a21 )X2 + a12 a21 [X2 , X1 ] + a21 X2 (a22 )X2 − a22 X2 (a21 )X2 + a21 a22 [X2 , X2 ] = 0.

化简表达式可得

(a11 a22 − a12 a21 ) [X1 , X2 ] + λ1 X1 + λ2 X2 = 0.

这说明
λ1 X1 + λ2 X2 = (−a11 a22 + a12 a21 ) [X1 , X2 ] .


λ1 X1 + λ2 X2
[X1 , X2 ] = .
a12 a21 − a11 a22
进一步说明 [X1 , X2 ] 可以被 X1 , X2 线性表出.

上面的例子说明可积子流形存在能够推出 [X1 , X2 ] ∈ span {X1 , X2 }, 我们


想问可积子流形的存在于 {Xi , Xj } ∈ span {Xi , Xj } 有什么关系? 为了搞清楚
这个问题, 我们还得做一些铺垫, 引入一些我们所需要的定义.
为了考虑给定多个向量场的情况, 并且保证这些向量场张成的切空间是某
个切空间的子空间, 我们引入如下定义.

124
CHAPTER 6. 子流形理论 6.4. FROBENIUS 定理

定 义 6.13. 设 M 为 光 滑 流 形, 若 对 每 一 点 p ∈ M , 都 指 定 一 个 k 维 切 子
空 间 Lkp , 则称 Lkp 是 M 上的 一 个 k 维 (光滑) 分 布. 局 部 上 看, 对 每 一 点
p ∈ M , 存 在 p 的 邻 域 U 以及 U 上处 处 线性 无 关 的 k 个 (光 滑) 向 量 场
X1 , X2 , · · · , Xk , 使得在每一点 q ∈ U , 都有 Lkp 由 X1 (q), X2 (q), · · · , Xk (q) 张
成, 即 Lkp = span {X1 (q), X2 (q), · · · , Xk (q)} . 换而言之

LkU = span {X1 , X2 , · · · , Xk }

又称 Lk 为 U 上的 (光滑) 切子空间场.

现在可以用上述定义来重新叙述那个问题了, 设 Lk 是 k 维分布, 是否存在


积分流形 N , 使得 N 在每一点 Tp N 的切空间恰好是 Lk (p)? 为方便表述. 我们
引出下述定义.

定义 6.14. Lk 是定义在 U 上的光滑分布, 若对任意的 p ∈ U , 存在过 p 的子流


形 V ⊆ U, 使得 Tp V = Lk (p), 则称 Lk 在 U 上是完全可积, 则称 Lm 在 U 上
完全可积的, 而 V 称 Lk 过 p 的积分流形.

有了上述铺垫, 我们来继续讨论子流形的可积性问题, 我们之前在积分曲面


的存在性问题那里, 曾经提问:[Xi , Xj ] ∈ span {X1 , X2 , · · · , Xk } . 与积分流形存
在的关系是什么? 我们先部分的回答问题, 这便是下面的定理.

定理 6.7. 设 Lk 是 V ⊆ M n 上的光滑分布, 且 X1 , X2 , · · · , Xk 是 V 上的处处


线性无关的向量场.LkV = span {X1 , X2 , · · · , Xk }, 若对任意的 p ∈ V, 存在局部
{ ∂ }
1 , ∂u2 , · · · , ∂uk
∂ ∂
坐标系 (U, ui ) 使得 Lkp = span ∂u (换而言之 Lk 在 U 上完全
可积), 则 [Xi , Xj ] ≡ 0 (mod Xl ).

注解 6.6. 对上述同余式做一些备注, 在初等数论中, 同余式是重要的概念. 所


谓同余是指的余数相同, 比如 7 ≡ 2 mod 5. 换种表述便是 7 ∈ {2 + 5 · n}
这 里 的 同 余 式 的 也 就 是 [Xi , Xj ] 可 以 用 Xl 线 性 表 出. 也 就 是 [Xi , Xj ] ∈
span {X1 , X2 , · · · , Xk } 这个表述也称为 F robenius 条件.

证明. 对任意的 p ∈ V , 取局部坐标系 (U, ui ), 1 ≤ i ≤ n, 使得 V (根据浸入子流


{ }
形的标准型定理)V = (u1 , u2 , · · · , uk , uk+1 , · · · , un ) | uα = const, k + 1 ≤ α. ,
j −1
于是 Xi = bji ∂u

j , 取 bj = (ai )
i
.有 ∂
∂ui = aji Xj . 其中 det(aij ) ̸= 0, det(bij ) ̸= 0.

125
6.4. FROBENIUS 定理 CHAPTER 6. 子流形理论

作括号积 [ ]
∂ ∂
[Xi , Xj ] = bki k , blj l
∂u ∂u
∂ ∂ ∂ ∂
= bki k (blj l ) − blj l (bki k )
∂u ∂u ∂u ∂u
l k
∂b j ∂ ∂b ∂
= bki k l − blj il k
∂u ∂u ∂u ∂u
l
∂b j ∂ ∂bl ∂
= bki k l − bkj ki
∂u ∂u ∂u ∂ul
l
∂bj ∂bl ∂
= (bki k − bkj ki ) l
∂u ∂u ∂u
l l
∂b j k ∂bi
= aml (b k
i − bj )Xm .
∂uk ∂uk
于是 [Xi , Xj ] ≡ 0 (mod Xl ).

现在我们要提一个反问题: 即若 [Xi , Xj ] ≡ 0 (mod Xl ). 则 Xi 所定义的分


布 Lk 是否是完全可积的? 回答是肯定的, 这就是下面的定理.

定理 6.8. 设 Lk 为 V ⊆ M n 是 V 上的光滑分布,X1 , X2 , · · · , Xk 是 V 上线性


无关的 k 个光滑向量场, 若 [Xi , Xj ] ≡ 0 (mod Xl ). 则对任意的 p ∈ V, 存在 p
{ ∂ }
的坐标邻域 (U, ui ), 1 ≤ i ≤ n, 使 Lk |U = span ∂u 1 , ∂u2 , · · · , ∂uk
∂ ∂
, 即 Lk 在
U 上完全可积.

证明. 为简便起见, 只考虑 k = 2 的情况, 设 X1 , X2 为 V 上线性无关的光滑


向量场. 因此 X1 , X2 在 V 上处处不为零. 由向量场的局部标准型定理知, 存
{ } ′
在局部坐标系 y A 使得 X2 = ∂y∂ 2 . 进一步考虑 X1 = X1 − X1 (y2 )X2 . 注

意到 X1 (y1 ) = X1 (y1 ) − X1 (y2 )X2 (y1 ) = X1 (y1 ) − X1 (y2 ) ∂y
∂y1
2
= X1 (y1 ). 而且
′ ∂y2
X1 (y2 ) = X1 (y2 )−X1 (y2 )X2 (y2 ) = X1 (y2 )−X1 (y2 ) ∂y 2
= X1 (y2 )−X1 (y2 ) = 0.

这说明可用 X1 替代 X1 (这就是为什么要作用于坐标系函数
{ } y1 , y2 的原因), 进

一步有 span {X1 , X2 } = span X1 , X2 . 注意到 [X1 , X2 ] ∈ span {X1 , X2 } . 于
{ ′ } [ ′ ]
是 [X1 , X2 ] ∈ span X1 , X2 . 而且由 P ossion 括号积的线性性质 X1 , X2 =
[X1 − X1 (y2 )X2 , X2 ] = [X1 , X2 ]−[X1 (y2 )X2 , X2 ] = [X1 , X2 ]+X2 (X1{(y2 ))X2}
+

X1 (y2 ) [X2 , X2 ] = [X1 , X2 ]+X2 (X1 (y2 ))X2 ∈ span {X1 , X2 } = span X1 , X2 ,
[ ′ ] { ′ } [ ′ ] ′
即 X1 , X2 ∈ span X1 , X2 . 于是 X1 , X2 = aX1 + bX2 . 依 P ossion 括号
[ ′ ] ′ ′
积的定义, X1 , X2 (y2 ) = X1 (X2 (y2 )) − X2 (X1 (y2 )) = X1 ( ∂y ∂y2
) − X2 (0) =
[ ′ ] ′
2

X1 (1) − X2 (0) = 0 − 0 = 0. 另一方面 X1 , X2 (y2 ) = aX1 (y2 ) + bX2 y2 = b. 于


[ ′ ] ′
是 b = 0. 进一步有 X1 , X2 = aX1 , 再次做坐标变换 y A = y A (x1 , x2 , · · · , xn ).
′ ′

使得 X1 = ∂x1 ,

再设 X2 = ξ A ∂x∂A = ξ 1 ∂x 1
+ ξ α ∂x∂α = ξ 1 X1 + ξ α ∂x∂α . 令

126
CHAPTER 6. 子流形理论 6.4. FROBENIUS 定理

′ ′
X2 = X2 − ξ 1 X1 = ξ α ∂x∂α , 其中 2 ≤ α ≤ n. 同理我们得到 span {X1 , X2 } =
{ ′ } { ′ ′
}
span X1 , X2 = span X1 , X2 . 做 P ossion 括号积

[ ] [ ]
′ ∂ 1 ∂ α ∂
X1 , X2 = ,ξ +ξ
∂x1 ∂x1 ∂xα
[ ] [ ]
∂ 1 ∂ ∂ α ∂
= ,ξ + ,ξ
∂x1 ∂x1 ∂x1 ∂xα
[ ] [ ]
∂ 1 ∂ 1 ∂ ∂ ∂ α ∂ α ∂ ∂ .
= (ξ ) +ξ , + (ξ ) +ξ ,
∂x1 ∂x1 ∂x1 ∂x1 ∂x1 ∂xα ∂x1 ∂xα
∂ξ 1 ′ ∂ξ α ∂
= X +
∂x1 1 ∂x1 ∂xα

= aX1 .

即有方程
{
∂ξ 1
∂x1 = a.
∂ξ α
∂x1 = 0, 2 ≤ α ≤ n.

这说明 ξ α 与 x1 无关, 即 ξ α = ξ α (x2 , x3 , · · · , xn ), 于是 X2 也只与 (x2 , x3 , · · · , xn )

有关. 则存在坐标变换 wα = wα (x2 , x3 , · · · , xn ). 使得 X2 = ∂
∂w2 . 即 w1 =
′ ′
x1 , wα{ = wα (x}2 , x3 , · · · {
, xn ). 于是
}X =
∂ ∂
∂w1 , X2 = ∂w2 . 于是 L2 = span {X1 , X2 } =
′ ′ ′ { ∂ ∂
}
span X1 , X2 = span X1 , X2 = span ∂w1 , ∂w 2 .

{ ∂ }
1 , ∂u2 , · · · , ∂uk ⇐⇒ Lk 是完全可积的.
∂ ∂
注解 6.7. 对一般的 k,Lk = span ∂u
{ 1 2 }
⇐⇒ V = u , u , · · · , um , um+1 = const, · · · , un = const .

于是我们得到如下定理. 这便是著名的 F robenius 定理

定理 6.9. 设 Lk 为 V ⊆ M n 是 V 上的光滑分布,X1 , X2 , · · · , Xk 是 V 上线性无


关的 k 个光滑向量场, 则对任意的 p ∈ V, 存在 p 的坐标邻域 (U, ui ), 1 ≤ i ≤ n,
{ ∂ }
使 Lk |U = span ∂u 1 , ∂u2 , · · · , ∂uk
∂ ∂
(即 Lk 在 U 上完全可积) 的充分必要条件
是 [Xi , Xj ] ≡ 0 (mod Xl ).

∂ ∂ ∂ ∂
习题 6.1. 设 X1 = f11 (x, y, z) ∂x +f12 (x, y, z) ∂y +f13 (x, y, z) ∂z , X2 = f21 (x, y, z) ∂x +
∂ ∂
f22 (x, y, z) ∂y + f23 (x, y, z) ∂z . 并 且 X1 , X2 线 性 无 关. 并 且 假 设 [X1 , X2 ] =
λX1 + µX2 , 给出 fji 需要满足的条件.
[ ]
∂ ∂
证明. 注意到 ∂x , ∂y = 0. 类似地 x, y, z 都有这种结论. 借助 P ossion 括号积

127
6.4. FROBENIUS 定理 CHAPTER 6. 子流形理论

的性质.
[ ]
∂ 2 ∂ 3 ∂ 1 ∂ 2 ∂ 3 ∂
[X1 , X2 ] = f11
+ f1 + f 1 , f2 + f2 + f2
∂x ∂y ∂z ∂x ∂y ∂z
[ ] [ ] [ ]
1 ∂ 1 ∂ 1 ∂ 2 ∂ 1 ∂ 3 ∂
= f1 ,f + f1 ,f + f1 ,f +
∂x 2 ∂x ∂x 2 ∂y ∂x 2 ∂z
[ ] [ ] [ ]
∂ ∂ ∂ ∂ ∂ ∂
f12 , f21 + f12 , f22 + f12 , f23 +
∂y ∂x ∂y ∂y ∂y ∂z
[ ] [ ] [ ]
∂ ∂ ∂ ∂ ∂ ∂
f13 , f21 + f13 , f22 + f13 , f23
∂z ∂x ∂z ∂y ∂z ∂z
[ ∂ ] [∂ ∂]
注意到 f ∂u ∂
, g ∂v ∂
= f ∂u ∂
(g) ∂v − g ∂v
∂ ∂
(f ) ∂u − f g ∂v , ∂u = f ∂u ∂v − g ∂v ∂u .
∂g ∂ ∂f ∂

带入上式有 [ ]
∂ ∂ ∂f 1 ∂ ∂f 1 ∂
f11 , f21 = f11 2 − f21 1 .
∂x ∂x ∂x ∂x ∂x ∂x
[ ]
∂ ∂ ∂f 2 ∂ ∂f 1 ∂
f11 , f22 = f11 2 − f22 1 .
∂x ∂y ∂x ∂y ∂y ∂x
[ ]
1 ∂ 3 ∂ ∂f 3 ∂ ∂f 1 ∂
f1 , f2 = f11 2 − f23 1 .
∂x ∂z ∂x ∂z ∂z ∂x
[ ]
2 ∂ 1 ∂ ∂f 1 ∂ ∂f 2 ∂
f1 , f2 = f12 2 − f21 1 .
∂y ∂x ∂y ∂x ∂x ∂y
[ ]
2 ∂ 2 ∂ ∂f 2 ∂ ∂f 2 ∂
f1 , f2 = f12 2 − f22 1 .
∂y ∂y ∂y ∂y ∂y ∂y
[ ]
2 ∂ 3 ∂ ∂f 3 ∂ ∂f 2 ∂
f1 , f2 = f12 2 − f23 1 .
∂y ∂z ∂y ∂z ∂z ∂y
[ ]
3 ∂ 1 ∂ ∂f 1 ∂ ∂f 3 ∂
f 1 , f2 = f13 2 − f21 1 .
∂z ∂x ∂z ∂x ∂x ∂z
[ ]
∂ ∂ ∂f 2 ∂ ∂f 3 ∂
f13 , f22 = f13 2 − f22 1 .
∂z ∂y ∂z ∂y ∂y ∂z
[ ]
∂ ∂ ∂f 3 ∂ ∂f 3 ∂
f13 , f23 = f13 2 − f23 1 .
∂z ∂z ∂z ∂z ∂z ∂z
把这些式子带入 [X1 , X2 ] 中, 得到

∂f21 ∂f 1 ∂f 1 ∂f 1 ∂f 1 ∂f 1 ∂
[X1 , X2 ] = (f11 − f21 1 − f22 1 − f23 1 + f12 2 + f13 2 ) +
∂x ∂x ∂y ∂z ∂y ∂z ∂x
2 2 2 2 2
∂f ∂f ∂f ∂f ∂f ∂f 2 ∂
(f11 2 − f21 1 + f12 2 − f22 1 − f23 1 + f13 2 ) +
∂x ∂x ∂y ∂y ∂z ∂z ∂y
3 3 3 3 3
∂f ∂f ∂f ∂f ∂f ∂f 3 ∂
(f11 2 + f12 2 − f22 1 − f21 1 + f13 2 − f23 1 ) .
∂x ∂y ∂y ∂x ∂z ∂z ∂z

128
CHAPTER 6. 子流形理论 6.4. FROBENIUS 定理

∂ ∂ ∂ ∂ ∂
另一方面 [X1 , X2 ] = λX1 + µX2 = λ(f11 ∂x + f12 ∂y + f13 ∂z ) + µ(f21 ∂x + f22 ∂y +
∂ ∂ ∂ ∂
f23 ∂z ) = (λf11 + µf21 ) ∂x + (λf12 + µf22 ) ∂y + (λf13 + µf23 ) ∂z . 对比两式可得如下
方程组


1 1 1 1 1 1
1 ∂f2 1 ∂f1 2 ∂f1 3 ∂f1 2 ∂f2 3 ∂f2
 λf1 + µf2 = f1 ∂x − f2 ∂x − f2 ∂y − f2 ∂z + f1 ∂y + f1 ∂z .
 1 1

∂f 2 ∂f 2 ∂f 2 ∂f 2 ∂f 2 ∂f 2
λf12 + µf22 = f11 ∂x2 − f21 ∂x1 + f12 ∂y2 − f22 ∂y1 − f23 ∂z1 + f13 ∂z2 .


 λf 3 + µf 3 = f 1 ∂f23 + f 2 ∂f23 − f 2 ∂f13 − f 1 ∂f13 + f 3 ∂f23 − f 3 ∂f13 .
1 2 1 ∂x 1 ∂y 2 ∂y 2 ∂x 1 ∂z 2 ∂z

不妨选第一个和第二个方程解出 λ, µ 带入第三个式子. 我们得到如下结果


∂f21 ∂f11 ∂f11 3 ∂f1
1 1
2 ∂f2
1
3 ∂f2
f22 (f11 ∂x − f21 ∂x − f22
∂y − f2 ∂z + f1 ∂y + f1 ∂z )
λ= −
f11 f22 − f12 f21
∂f22 ∂f 2 ∂f 2 ∂f 2 ∂f 2 ∂f 2
f21 (f11 ∂x − f21 ∂x1 + f12 ∂y2 − f22 ∂y1 − f23 ∂z1 + f13 ∂z2 )
.
f11 f22 − f12 f21
∂f22 ∂f12 ∂f22 2 ∂f1
2 2
3 ∂f1
2
3 ∂f2
f11 (f11 ∂x − f21 ∂x + f12∂y − f2 ∂y − f2 ∂z + f1 ∂z )
µ= −
f11 f22 − f12 f21
∂f21 ∂f 1 ∂f 1 ∂f 1 ∂f 1 ∂f 1
f12 (f11 ∂x − f21 ∂x1 − f22 ∂y1 − f23 ∂z1 + f12 ∂y2 + f13 ∂z2 )
f11 f22 − f12 f21
带入第三个式子有
∂f21 ∂f 1 ∂f 1 ∂f 1 ∂f 1 ∂f 1
(f22 (f11 − f21 1 − f22 1 − f23 1 + f12 2 + f13 2 )−
∂x ∂x ∂y ∂z ∂y ∂z
2 2 2 2 2 2
∂f ∂f ∂f ∂f ∂f ∂f
f21 (f11 2 − f21 1 + f12 2 − f22 1 − f23 1 + f13 2 ))(f13 )+
∂x ∂x ∂y ∂y ∂z ∂z
2 2 2 2 2
∂f ∂f ∂f ∂f ∂f ∂f 2
(f11 (f11 2 − f21 1 + f12 2 − f22 1 − f23 1 + f13 2 )−
∂x ∂x ∂y ∂y ∂z ∂z
1 1 1 1 1 1
∂f ∂f ∂f ∂f ∂f ∂f
f12 (f11 2 − f21 1 − f22 1 − f23 1 + f12 2 + f13 2 ))(f23 )
∂x ∂x ∂y ∂z ∂y ∂z
3 3 3 3 3
∂f ∂f ∂f ∂f ∂f ∂f 3
= f11 2 + f12 2 − f22 1 − f21 1 + f13 2 − f23 1 .
∂x ∂y ∂y ∂x ∂z ∂z
即为 fji 需要满足的条件.

我们下面用 Einstein 求和约定来解决这个题目. 这也是老师的解法.

证明. 设在 U 上,Xi = fiA ∂x∂A , 其中 fiA 为 U 上给定的光滑函数, 由 Xi 是


线性无关的得到 det(fij ) ̸= 0. 令 gij = (fji )−1 , 设 [Xi , Xj ] = Cij
k
Xk , 利用定
[ A ∂ ] A
B ∂fj
A
B ∂fi
义 [Xi , Xj ] = fi ∂xA , fj ∂xB = (fi ∂xB − fj ∂xB ) ∂xA . 注意到 [Xi , Xj ] =
B ∂ ∂

k A ∂
k
Cij Xk = Cij fk ∂xA . 对比上述式子就有
∂fjA A
B ∂fi
k A
Cij fk = (fiB − f j ).
∂xB ∂xB

129
6.4. FROBENIUS 定理 CHAPTER 6. 子流形理论

把上式分解成两部分
{ ∂f l ∂f l
k l
Cij fk = (fiB ∂xBj − fjB ∂xBi ).
∂f α ∂f α
k α
Cij fk = (fiB ∂xjB − fjB ∂xiB ).

∂f l ∂f l
由第一个式子 Cij
k
= glk (fiB ∂xBj − fjB ∂xBi ). 带入第二个式子得到

∂fjl l
B ∂fi
∂fjα α
B ∂fi
fkα glk (fiB − f j ) = (f B
i − f j ).
∂xB ∂xB ∂xB ∂xB
这就是需要满足的条件.

130
第 7 章 流形上的微积分

本章主要是想把欧式空间中的微积分搬到流形上面来. 之所以会考虑流形
上的微积分, 原因很简单: 把流形的局部性质联系到整体性质的最简单的方式就
是外微分形式的积分. 为了定义积分, 我们做了大量的准备工作. 先把切从推广
成了张量从, 向量场推广成了张量场. 之前判断张量的时候都是在局部坐标系下
写出张量表示然后做坐标变化, 观察张量表示是否能满足张量的坐标变换公式.
但每次都在坐标系下计算时比较麻烦的事情, 为了方便判断一个算子是否是张
量我们引出了 C ∞ (M ) 线性性, 证明了满足 C ∞ (M ) 线性性的算子指定了一个
张量场. 并应用于 P ossion 括号积和曲率和挠率张量. 为了寻找一种性质比较好
的微分算子引出了外微分, 证明了外微分的存在唯一性定理, 并在一定程度上刻
画了 d 的核空间, 又给出了微分公式, 并以此给出了 F robenius 定理的对偶形
式. 最后给出了上述对偶形式在微分方程中的应用, 然后介绍了一些特殊的外微
分式和单位分解定理, 由此引出了外微分式的积分的定义, 从局部的积分到整体
的积分的必要工具就是单位分解, 可以说成是数学分析中有限覆盖定理在流形
上的函数版本. 给出了 Stokes 定理并站在这个角度重新看待数学分析中的曲面
曲线积分的联系和区域积分与曲面积分的联系.

7.1 光滑张量场
2019-12-11
本节要点/问题提示:
1)、如何从切从过渡到张量从?
2)、张量从的拓扑结构和微分结构分别是什么?
3)、什么是张量场? 什么是光滑张量场?
4)、一个点处的切向量如何扩张成 M 上的光滑向量场?
6)、df 作为张量的新观点的要点在哪儿?

131
7.1. 光滑张量场 CHAPTER 7. 流形上的微积分

7)、张量新判别法则是如何叙述的?
8)、P ossion 括号积是张量场吗? 为什么?
8)、证明挠率 T (X, Y ) 是张量? 曲率 R(X, Y, Z) 是张量 (习题)?
回顾: 我们知道切向量放在一起便是切空间, 切空间放在一起便是切从, 切
从的截面就是向量场. 向量场就是在每一点指定一个切向量. 而且向量是一阶张
量, 因此我们就想对张量也进行相同的处理: 把张量放在一起构成张量空间, 张
量空间放在一起便是张量从, 张量从的截面就是张量场. 张量场就是在每一点指
定一个张量. 按照这种想法, 我们引出下列概念.
设 M n 是一个 n 维光滑流形,p ∈ M ,Tp M 是 M 在 p 处的切空间,Tp⋆ M 是
Tp M 的对偶空间. 作张量积并记为 Tsr (p) = Tp M ⊗ · · · ⊗ Tp M ⊗ Tp⋆ M ⊗ · · · ⊗ Tp⋆ M ,
| {z } | {z }
r s
对任意的 p 的邻域 U , 有 Tsr (U ) = ∪p∈U Tsr (p), 称之为 U 上的 (r, s) 型张量
从,Tsr (M ) = ∪p∈M Tsr (p), 称之为 M 上的 (r, s) 型张量从.
考虑到 Tsr 现在还是一盘散沙的集合, 没有为其赋予任何结构, 自然就想赋
予一些结构使得我们能够在这个集合 Tsr (M ) 上研究连续函数和可微函数, 这自
然就需要我们赋予 Tsr (M ) 赋予拓扑结构和微分结构:
拓扑结构:
1)、定义自然投影 π : Tsr (M ) 7→ M, 具体来说 π(Tsr (p)) = p. 若 (Uα , φα ) 为
M 的坐标卡, 在局部坐标下设 ξ = ξji11 ij22···i
···js δi1 ⊗ · · · ⊗ δir ⊗ δ · · · ⊗ δ
r j1 js
∈ Tsr (p).
2)、定义 (同胚) 映射 θα : π −1 (Uα ) = ∪p∈U Tp M 7→ Uα × Rn
r+s
, 具体的表
达式为

θ(ξ) = (p, ξ11···1 , ξ11···1 , · · · , ξji11 ij22···i


11···1 21···1
···js , · · · , ξnn···n ), ∀ ξ ∈ Ts (Uα ).
r nn···n r

显然 θ 是满射, 也是单射. 即 θ 是一一映射, 这个映射 θ 诱导了 π −1 (U ) 上的拓


扑, 使得该拓扑在 θ 下为同胚. 具体来说, 我们这么定义 T M 上的拓扑:
3)、定义拓扑: 设 {(Uα , φα )} 为 M 的微分结构.V 是 Tsr (M ) 中的开集当且
仅当 θα (V ∩ π −1 (Uα )) 为开集.
不难验证上述定义的 Tsr (M ) 拓扑是合理的. 并且在这个拓扑下 π 为开映
射,Tsr (M ) 是 A2 和 T2 的.
微分结构:
1)、定义同胚映射 (坐标映射): 设 {(Uα , φα )} 为 M 的微分结构, 考虑如下
复合映射
(φα , id) ◦ θα : π −1 (Uα ) 7→ Rn × Rn
r+s
.

具体写出来便是

((φα , id) ◦ θα ) (ξ) = (φα (p)), ξ11···1 , ξ11···1 , · · · , ξji11 ij22···i


11···1 21···1
···js , · · · , ξnn···n )∀ ξ ∈ Ts (Uα ).
r nn···n r

132
CHAPTER 7. 流形上的微积分 7.1. 光滑张量场

在上述拓扑结构下该复合映射 (φα , id) ◦ θα 显然为同胚. 即 T M 的坐标映射, 于


{ }
是 π −1 (Uα ), (φα , id) ◦ θα 为 T M 的坐标覆盖.
2)、考虑转换映射的光滑性. 当 π −1 (Uα ) ∩ π −1 (Uβ ) ̸= ∅, 坐标转换函数

((φβ , id) ◦ θβ ) ◦ ((φα , id) ◦ θα )−1 : φα (Uα ∩ Uβ ) × Rn 7→ φβ (Uα ∩ Uβ ) × Rn .

具体写出来便是
( )
(x, a) 7→ φβ ◦ φ−1
α , 张量坐标变换公式 , ∀ x ∈ φα (Uα ∩ Uβ ), a ∈ R .
n

因为 φβ ◦ φ−1
α 是光滑的, 而张量的坐标变换公式自然也是光滑的. 进一步上述
的坐标转换函数是光滑的.
{ }
于是 π −1 (Uα ), (φα , id) ◦ θα 为 T M 的光滑结构 (微分结构).

定义 7.1. Tsr (M ) 按照上述结构作成的集合称之为 M 上 (r, s) 型张量丛.

定义 7.2. 若映射 τ : M n 7→ Tsr (M ) 使得 τ (p) ∈ Tsr (p)(即 π ◦ τ (p) = p), 则称 τ


为 M 上的张量场.

同样地, 我们也在局部坐标系下定义什么是光滑张量场.
···ir ∂
定义 7.3. 若 (U, xi ) 为 p 处的一个坐标邻域,τ |U = τji11ji22 ···j s ∂xi1
⊗ · · · ∂x∂ir ⊗
···ir
dxj1 ⊗ · · · ⊗ dxjs , 其中 τji11ji22 ···j s
∈ C ∞ (U ). 则 τ 在 U 上光滑, 若 τ 在 M 上每
点光滑, 则称 τ 为 M 上的光滑张量场.

之前我们把局部的光滑函数扩充到整体的光滑函数, 同样地, 在这里, 我们


把局部的一个点处的切向量 (局部的切向量场), 扩充成整体的切向量场. 这就是
下面的引理.

引理 7.1. 设 p ∈ M, Xp ∈ Tp M, 存在 X ∈ X(M ), 使得 Xp = a. 其中 X 称为 a
的扩张.

证明. 设 (U, xi ) 是 p 的一个局部坐标系,a ∈ Tp M , 于是在局部坐标系下有


i |p , 由 于 M

a 的 局 部 坐 标 表 示 a = ai ∂x n
是 局 部 紧 的, 于 是 存 在 p 的 邻
域 V, W , 使 得 p ∈ V ⊆ V̄ ⊆ W ⊆ W̄ ⊆ U. 取 相 应 的 截 断 函 数 h 使 得
h |V = 1, h |M /W = 0, h ∈ C ∞ (M ), 定义
{
(h · ai ) ∂x

i , q ∈ U.
X=
0, q ∈
/ U.

则 X ∈ X(M ), 并且 Xp = ai ∂x

i |p = a.

133
7.1. 光滑张量场 CHAPTER 7. 流形上的微积分

我们都知道张量就是一个多重线性函数, 从代数的角度上讲, 如果一个张量,


用基底写出来后, 在坐标变换下满足张量的坐标变换公式便可以称之为是张量,
但有的时候取了基底之后再坐标变换下一个量可能会很复杂, 甚至于产生相当
∂f
的计算量. 举例来说, 设 fi = ∂xi , 显然 fi dxi 是张量, 这是因为一阶微分的形式
∂2f
不变性. 但二阶微分 fij = ∂xi ∂xj 所构成的 fij dxi dxj 一般不是张量, 因为高阶
导数一般都没有形式不变性, 二阶导数是张量当且仅当它是平坦的, 但他的计算
是复杂的, 我们能否用其他方式来判断一个量是张量? 这就是本节问题的核心:
张量的新判断法则. 我们先看几个例子. 我们的基本思路是, 从基底发生变化寻
找规律到其他的量发生变换以寻找其规律 (与函数有关等).

例 7.1. fi dxi 作为张量的新观点.


记 df = fi dxi , 之前我们是把 xi 变成 ui , 但现在我们不改变 x, 我们让切向
量场去作用, 进一步来说考虑如下等式

∂ ∂ ∂ ∂ ∂ ∂ ∂ ∂
df ( + ) = (fi dxi )( j + l ) = fi (dxi )( j )+fi (dxi )( l ) = df ( j )+df ( l ).
∂xj ∂xl ∂x ∂x ∂x ∂x ∂x ∂x

进一步考虑 g ∂x j 被 df 作用之后的结果. 注意到 df 作用在 g 上的结果为 gdf ,

于是有
∂ ∂
df (g ) = gdf ( j ).
∂xj ∂x
这说明 df 具有关于向量场的函数线性性.

注解 7.1. 上述是对偶的去看 df 的方式, 从最初的基变换作成张量坐标变换公


式, 到现在关于切向量场的函数线性性. 从改变坐标到变换被作用的切向量场.
多角度的去看待同一个东西, 不同的观点往往会有很有意思的概念存在. 这也可
以说是站在泛函的角度去看张量: 用算子的角度去看张量.

上面的例子说明,df 是关于函数空间 C ∞ 线性的, 即满足函数线性性. 这就


是我们接下来的定义:C ∞ 线性性. 首先记 A1 (M ) 为 M 上的 (0, 1) 型光滑张量
场, 即光滑一阶协变张量场.

定义 7.4. 映射 τ : A1 (M ) × · · · × A1 (M ) × X(M ) × · · · × X(M ) 7→ C ∞ (M ), 如


| {z } | {z }
r s
果对任意的 f, g ∈ C ∞ (M ), 任意的 Xi , Yi ∈ X(M ), 任意的 αj , β j ∈ A1 (M ). 有

τ (X1 , · · · , Xi−1 , f Xi + gYi , Xi+1 , · · · , Xr , α1 , α2 , · · · , αs )


= f τ (X1 , · · · , Xi−1 , Xi , Xi+1 , · · · , Xr , α1 , α2 , · · · , αs )+
gτ (X1 , · · · , Xi−1 , Yi , Xi+1 , · · · , Xr , α1 , α2 , · · · , αs ).

134
CHAPTER 7. 流形上的微积分 7.1. 光滑张量场

τ (X1 , X2 , · · · , Xr , α1 , · · · , αj−1 , f αj + gβ j , αj+1 , · · · , αs )


= f τ (X1 , X2 , · · · , Xr , α1 , · · · , αj−1 , αj , αj+1 , · · · , αs )+
gτ (X1 , X2 , · · · , Xr , α1 , · · · , αj−1 , β j , αj+1 , · · · , αs )

则称 τ 具有 r + s 重 C ∞ 函数线性性.

之前的例子说明了 df 具有函数线性性, 但具有函数线性性的是否一定是张


量呢? 回答是肯定的, 这就是下面的定理.

定理 7.1. r+s 重 C ∞ (M ) 映射 τ : A1 (M ) × · · · × A1 (M ) × X(M ) × · · · × X(M ) 7→


| {z } | {z }
r s
C ∞ (M ) 在 M 上指定了一个 (r, s) 型张量场.

证明. 为简便起便, 只考虑 (r, s) = (0, 1) 的情形, 设线性映射 τ : X(M ) 7→


C ∞ (M ) 具有函数线性性, 则 τ ∈ A1 (M ). 我们断言 τ 具有局部性, 即若 X |U =
Y |U , 则 τ (X) |U = τ (Y ) |U . 事实上, 设 p ∈ U ⊆ M, 存在 p 的邻域 U , 使得
p ∈ V ⊆ V̄ ⊆ W ⊆ W̄ ⊆ U, 以及相应的截断函数 C ∞ (M ),h |V ≡ 1, h |M /U = 0,
自然 h · (X − Y ) ∈ X(M ), 注意到 τ (0) = τ (0 + 0) = τ (0) + τ (0) = 2τ (0), 这说明
τ (0) = 0. 考虑到 (X −Y ) |U = 0, 必然有 h·(X −Y ) |V = 0, 于是 τ (h·(X −Y )) =
τ (0) = 0, 由 τ 的函数线性性知 τ (h · (X − Y )) = hτ ((X − Y )) = 0, 在 p 处取
值有 h(p) = 0, 于是 τ (X)(p) = τ (Y )(p) 由 p 的任意性知 τ (X) |U = τ (Y ) |U .
因为 τ 具有局部性. 我们定义 τp (Xp ) := τ (X)(p), 由 τ 的局部性知该定义是
合理的. 下面说明 τp ∈ Tp⋆ M , 即 τp 是 Tp M 上的线性函数. 事实上, 对任意的
Xp , Yp ∈ Tp M 有

τp (Xp + Yp ) = τ (X + Y )(p) = (τ (X) + τ (Y ))(p) = τp (Xp ) + τp (Yp ).

τp (λXp ) = τ (λX)(p) = λτ (X)(p) = λτp (Xp ).

这说明 τp ∈ Tp⋆ M, 于是 τ 指定了一个 (0, 1) 张量场.

事实上, 将上述的值域换成 X(M ), A1 (M ). 结论也是成立的, 于是我们有下


列两个推论. 但这两个推论的证明从略.

推论 7.1. r+s 重 C ∞ (M ) 映射 τ : A1 (M ) × · · · × A1 (M ) × X(M ) × · · · × X(M ) 7→


| {z } | {z }
r s
X(M ) 在 M 上指定了一个 (r, s) 型张量场.

推论 7.2. r+s 重 C ∞ (M ) 映射 τ : A1 (M ) × · · · × A1 (M ) × X(M ) × · · · × X(M ) 7→


| {z } | {z }
r s
A1 (M ) 在 M 上指定了一个 (r, s) 型张量场.

135
7.1. 光滑张量场 CHAPTER 7. 流形上的微积分

根据上述定理, 也就是可以看作是张量 (张量场) 的一个判别法则, 下面来


看一些具体的应用, 这是我们摆脱基坐标变换去判断张量场的非常重要的一步.
另外值得一提的是, 这个条件是充要的, 即任何一个张量都具有函数线性性. 具
有函数线性性的算子也是张量.

例 7.2. P ossion 括号积是否是张量场?


注意到 [, ] : X(M ) × X(M ) 7→ X(M ). 有上述推论知 [X, Y ] 是否有函数线性
性即可判断, 事实上有 P ossion 括号积的性质有 [X, f Y ] = X(f )Y + f [X, Y ] ̸=
f [X, Y ] . 即 P ossion 括号积不具有函数线性性, 由此说 P ossion 括号积不是张
量.

例 7.3. 向量场的协变导数, 挠率张量, 曲率张量.


首先定义什么是联络, 联络是指的满足下面三条性质的算子:D : X(M ) ×
X(M ) 7→ X(M ). 对任意的 X, Y, Z ∈ X(M ), 任意的 f, g ∈ C ∞ (M ), 有
1)、Df X+gY Z = f DX Z + gDY Z.
2)、DX (Y + Z) = DX Y + DX Z.
3)、DX (f Y ) = f DX Y + X(f )Y.
则 D 称之为联络,DX 称之为协变导数. 我们记 D(X, Y ) = DX Y. 显然 D 不
是张量场. 进一步通过 D 我们构造一些常见的张量场, 定义 T : X(M )×X(M ) 7→
X(M ). 具体来说 T (X, Y ) = DX Y − DY X − [X, Y ] , 我们证明 T 是张量, 显然
T (X, Y ) = −T (Y, X). 考虑 T (f X, Y ) = Df X Y − DY f X − [f X, Y ] = f DX Y −
f DY X − Y (f )X + Y (f )X + f [Y, X] = f DX Y − f DY X − Y (f )X + Y (f )X −
f [X, Y ] = f DX Y − f DY X − f [X, Y ] = f (DX Y − DY X − [X, Y ]) = f T (X, Y ).
进一步 T (X, f Y ) = −T (f Y, X) = −f T (Y, X) = f T (X, Y ). 而线性性质是显然
的, 又上述推论知 T 是张量, 称之为挠率张量. 在局部坐标系 (U, xi ) 下, 考虑
[ ∂ ]
i , ∂xj ) = D ∂ ∂xj − D ∂ ∂xi −
∂ ∂ ∂ ∂ ∂ ∂ k ∂
T ( ∂x i ∂x j ∂x
∂xi , ∂xj , 记 D ∂ i ∂xj := Γij ∂xk , 于是
∂x

i , ∂xj ) = Tij ∂xk = (Γij − Γji ) ∂xk .



D ∂
∂xi := Γkji ∂x∂ k , 进一步记 T ( ∂x
∂ ∂ k ∂ k k ∂
∂xj

定义曲率算子 R(X, Y ) : X(M ) → 7 X(M ), 如下 R(X, Y )Z = (DX DY −


e
DY DX − D[X,Y ] )Z, 记 R(X, e
Y, Z) = R(X, Y )Z. 则称 R(X, Y, Z) 为曲率张量
(证明留作习题).

e
习题 7.1. 证明上述定义的 R(X, Y, Z) 为 (曲率) 张量.

证明. 对任意的 X, Y, Z ∈ X(M ), 注意到 R(X, Y ) = −R(Y, X), 考虑对任意的

136
CHAPTER 7. 流形上的微积分 7.2. 外微分与外微分式

f ∈ C ∞ (M ), 有

e X, Y, Z) = (Df X DY − DY Df X − D[f X,Y ] )Z


R(f
= Df X DY Z − DY Df X Z − D[f X,Y ] Z
= f DX DY Z − DY f DX Z − D−Y (f )X+f [X,Y ] Z
= f DX DY Z − (f DY DX Z + Y (f )DX Z) − (−Y (f )DX Z + f D[X,Y ] Z)
= f (DX DY Z − DY DX Z − D[X,Y ] Z)
e
= f R(X, Y, Z).

e
由 于 R(X, e Y, X, Z)) =
f Y, Z) = (R(X, f Y )Z) = (−R(f Y, X)Z) = (−R(f
e X, Z) = f R(X,
−f R(Y, e Y, Z). 下考虑

e
R(X, Y, f Z) = (DX DY − DY DX − D[X,Y ] )f Z
= DX DY f Z − DY DX f Z − D[X,Y ] f Z
= DX (f DY Z + Y (f )Z) − DY (f DX Z + X(f )Z) − (f D[X,Y ] Z + [X, Y ] (f )Z)
=(X(f )DY Z + f DX DY Z + X(Y (f ))Z + Y (f )DX Z)
− (f DY DX Z + Y (f )DX Z + Y (X(f ))Z + X(f )DY Z)
− (X(Y (f )) − Y (X(f )) + f D[X,Y ] Z)
= f (DX DY Z − DY DX Z − D[X,Y ] Z)
e
= f R(X, Y, Z).

e 是张量.
于是 R

注解 7.2. 解释一下上述名词, 习惯上我们把一阶协变导数相减得到的张量称之


为挠率张量, 二阶协变导数相减得到的量称之为曲率张量. 上述曲率张量还具有
一些其他的性质, 这在黎曼几何中是基础的概念! 还有一些重要的东西比如第一
第二 Bianchi 恒等式. 之所以会有协变导数这个概念, 是因为普通的求导在流形
上不构成张量场, 所以我们找到了一中求导性质比较好的求导: 协变导数. 协变
的意思是作为张量而言具有协变性, 也就是这个张量与坐标变换的规律相同. 这
是一种对向量场求导的方式, 具体的几何解释请参见下学期我黎曼几何的笔记.

7.2 外微分与外微分式
2019-12-18

137
7.2. 外微分与外微分式 CHAPTER 7. 流形上的微积分

本节要点/问题提示:
1)、什么是外微分式?
2)、外微分算子的存在唯一性定理的内容是什么?
3)、该定理的证明思路是什么? 要点在哪里?
4)、如何描述外微分算子 d 的核空间? 有哪些例子?
6)、微分公式的内容? 它是为了描述什么?
7)、多个向量场的微分公式是怎样的? 如何利用微分公式的定义去证明外
微分算子的四个性质?
8)、F robenius 的对偶形式是怎样的?
回顾: 之前规定了 A1 (M ) 为 M 上 1 次外形式的全 体, 这一节我们 主
要讲微分形式, 先回顾什么是 r 次微分形式. 一个光滑的 r 次外形式场称
之 为 一 个 外 微 分 式, 即 在 流 形 的 每 一 点 处 指 定 一 个 r 次 外 形 式, 用 映 射 的
语言说 τ : M 7→ Λr (Tp⋆ M ), 其中 τp 是 Tp M 上的 r 次外形式 (即 r 重反
对 称 线 性 函 数),τp : Tp M × · · · × Tp M 7→ R 是 r 重 反 线 性 的. 由 前 面 的 定
| {z }
r
理,τ : X(M ) × · · · × X(M ) 7→ C ∞ (M ) 是 r 次外微分式当且仅当 τ 是 C ∞ (M )
| {z }
r
线性的而且是反称的. 特别地, 在局部坐标系 (U, xi ) 下, 有 τ |U = r! τi1 i2 ···ir dx ∧
1 i1

dx ∧ · · · ∧ dx , 其中 τi1 i2 ···ir dx
i2 ir i1
是 U 上的光滑函数. 事实上, 按照上述记
号,A(M ) 可以看作是 Λ(M ) 的截面, 即对任意的 τ ∈ A(M ), 若存在映射 π, 使得
π ◦ τ = id : M 7→ M, 进一步有 π : Λ(M ) 7→ M, 具体来说 π(Λr (Tp M )) = p, 现
在我们记 Ar (M ) 为 r 次外形式的全体. 并且作形式 A(M ) := ⊕nr=0 Ar (M ), 规
定 A0 = C ∞ (M ), 同样地,Λ(M ) = ⊕nr=0 Λr (M ),Λ0 = C ∞ (M ). 偶对 (Λ(M ), ∧)
构成了一个外代数, 其中 ∧ : Λ(M ) 7→ Λ(M ). 下面考虑映射 d : A(M ) 7→ A(M ),
具体来说 d : Ar (M ) 7→ Ar+1 (M ). 本节主要研究满足一些性质的这种微分运算,
称之为外微分.

定义 7.5. 一个光滑的 r 次外形式场称之为一个外微分式, 即在流形的每一点处


指定一个 r 次外形式, 用映射的语言说 τ : M 7→ Λr (Tp⋆ M ), 其中 τp 是 Tp M 上
的 r 次外形式.

关于外微分, 我们有下面的存在唯一性定理.

定理 7.2. n 维光滑流形 M 上存在唯一的外微分算子 d : A(M ) 7→ A(M ), d(Ar (M )) ⊆


Ar+1 (M ). 满足下列性质:
1)、对 ∀ α, β ∈ A(M ), 有 d(α + β) = dα + dβ.
2)、对 ∀ α ∈ Ak (M ), β ∈ A(M ), 有

d(α ∧ β) = dα ∧ β + (−1)k α ∧ dβ.

138
CHAPTER 7. 流形上的微积分 7.2. 外微分与外微分式

3)、对 ∀ f ∈ C ∞ (M ) = A0 (M ),df 是普通微分.


4)、对 ∀ α ∈ A(M ), 有 d(dα) = 0.

证明. 先假定 d 存在, 证明 d 的唯一性. 首先断言 d 具有局部性. 即对任意的


α, β ∈ Ar (M ), 若存在邻域 U , 使得 α |U = β |U , 则 d(α) |U = d(β) |U . 事实上,
对任意的 p ∈ U, 存在 p 的邻域 V , 使得 p ∈ V ⊆ V̄ ⊆ U. 其中 V̄ 紧致, 以及相应
的截断函数 h ∈ C ∞ (M ), 使得 h |V ≡ 1, h |M /U ≡ 0, 显然 h·(α−β) ∈ Ar (M ), 并
且 h · (α − β) ≡ 0, 有 1) 知 d(0) = d(0 + 0) = d(0) + d(0) = 2d(0), 即 d(0) = 0, 于
是 d(h·(α −β)) = 0, 根据 2) 和 3) 有,d(h·(α −β)) = dh·(α −β)+h·d(α −β), 在
p 点处有 α(p) = β(p), h(p) = 1, 于是有 d(α−β) = 0, 进一步 dα = dβ. 由 p 的任
意性知 d(α) |U = d(β) |U . 因此我们定义 d(α |U ) = d(α) |U , 我们在局部上考虑
d 的表达式, 取局部坐标系 (U, xi ), 考虑 α |U = 1
r! αi1 i2 ···ir dx
i1
∧ dxi2 ∧ · · · ∧ dxir ,
其中 αi1 i2 ···ir 关于指标是反称的, 由上述的 4) 知 d(dx ) = 0, 于是 d(dxi1 ∧ i

dxi2 ∧ · · · ∧ dxir ) = d(dxi1 ) ∧ (dxi2 ∧ · · · ∧ dxir ) − dxi1 ∧ d(dxi2 ∧ · · · ∧ dxir ) =


0 − dxi1 ∧ d(dxi2 ∧ · · · ∧ dxir ) = 0. 最后一步等于 0 是因为上述步骤可以一直这
么做下去. 于是有

d(α |U ) = d(α) |U
1
= d( αi1 i2 ···ir dxi1 ∧ dxi2 ∧ · · · ∧ dxir )
r!
1
= d(αi1 i2 ···ir dxi1 ∧ dxi2 ∧ · · · ∧ dxir )
r!
1 1
= d(αi1 i2 ···ir ) ∧ (dxi1 ∧ dxi2 ∧ · · · ∧ dxir ) + αi1 i2 ···ir d(dxi1 ∧ dxi2 ∧ · · · ∧ dxir )
r! r!
1
= d(αi1 i2 ···ir ) ∧ (dxi1 ∧ dxi2 ∧ · · · ∧ dxir )
r!
1 ∂αi1 i2 ···ir j
= dx ∧ (dxi1 ∧ dxi2 ∧ · · · ∧ dxir ).
r! ∂xj

这说明在每一个局部 (U, xi ) 上 d 的表达式是唯一的. 这就说明 d 是唯一的. 下说


明 d 存在, 在每一个 (U, xi ) 上, 定义 d 如下: 若 α |U = r! αi1 i2 ···ir dx ∧dx ∧· · ·∧
1 i1 i2

1 ∂αi1 i2 ···ir
dxir , 进一步有 d(α |U ) = r! ∂xj dxj ∧ (dxi1 ∧ dx ∧ · · · ∧ dx ). 我们说明 d
i2 ir

是合理的 (整体的), 即说明: 若在 (V, y j ) 上, 当 U ∩ V ̸= ∅ 时,d(α |U ) = d(α |V ),


考虑坐标转换函数 y i = y i (x1 , x2 , · · · , xn ), 显然

∂(y 1 , y 2 , · · · , y n )
̸= 0.
∂(x1 , x2 , · · · , xn )
∑ ∂y i ∂xj
由反函数存在定理, 存在反函数 xi = xi (y 1 , y 2 , · · · , y n ), 注意到 ∂xj ∂y k = δki ,
设 α |V = r! α^
1
i1 i2 ···ir dx
i1
∧ dxi2 ∧ · · · ∧ dxir , 而在相交区域上必然有 (α |V )U ∩V =

139
7.2. 外微分与外微分式 CHAPTER 7. 流形上的微积分

1 ∂ αi^
α |U ∩V = (α |U )U ∩V , 显然 d(α |V ) = r!
1 i2 ···ir
∂y j dy j ∧ (dy i1 ∧ dy i2 ∧ · · · ∧ dy ir ),
我们只需证明在相交区域 V ∩ U 上, 有

1 ∂ α^
i1 i2 ···ir 1 ∂αi1 i2 ···ir j
dy j ∧(dy i1 ∧dy i2 ∧· · ·∧dy ir ) = dx ∧(dxi1 ∧dxi2 ∧· · ·∧dxir ).
r! ∂y j r! ∂xj

即可. 事实上, 注意到 V ∩ U 上 α 的表达式是相同的, 即


1 1
αi i ···i dxi1 ∧ dxi2 ∧ · · · ∧ dxir = α^i i ···i dy ∧ dy ∧ · · · ∧ dy .
i1 i2 ir
r! 1 2 r r! 1 2 r
∂y j ∂
显然在 U ∩ V 上 ∂
∂xi = ∂xi ∂y j , 因此得到

∂ ∂ ∂
αi1 i2 ···ir = α( , , · · · , ir )
∂xi1 ∂xi2 ∂x
∂y j1 ∂ ∂y j2 ∂ ∂y jr ∂
= α( i1 j1 , i2 j2 , · · · , ir jr )
∂x ∂y ∂x ∂y ∂x ∂y
∂y j1 ∂y j2 ∂y jr ∂ ∂ ∂
= i i
· · · ir α( j1 , j2 , · · · , jr )
∂x ∂x1 2 ∂x ∂y ∂y ∂y
∂y j1 ∂y j2 ∂y jr
= · · · ir α^ j1 j2 ···jr .
∂xi1 ∂xi2 ∂x
j1 ∂y j2 jr
∂(αi1 i2 ···ir ) ( ∂y i ··· ∂y ir αj^
1 j2 ···jr )
∂x 1 ∂xi2
两边同时用求偏导有 ∂xj = ∂x
∂xj . 进一步
j1 j2 jr
∂(αi1 i2 ···ir ) ∂( ∂y ∂y
∂xi1 ∂xi2 ∂xir α^
· · · ∂y j1 j2 ···jr )
j
= j
∂x ∂x
∂y j1 ∂y j2 ∂y jr ∂ α^ j1 j2 ···jr
= i1 · · · +
∂x ∂xi2 ∂xir ∂xj
j1 j2 jr
∂( ∂y ∂y
∂xi1 ∂xi2
· · · ∂y
∂xir )
α^j1 j2 ···jr j
∂x
∂y j1 ∂y j2 ∂y jr ∂ α^ j1 j2 ···jr
= i1 · · · ir +
∂x ∂xi2 ∂x ∂xj
j1
∂y j2 jr
∂( ∂y∂xi1 ∂xi2
· · · ∂y
∂xir )
α^j1 j2 ···jr
∂xj
j1
∂y ∂y j2
∂y ∂ α^
jr
j1 j2 ···jr
= i1 · · · ir +
∂x ∂xi2 ∂x ∂xj
∂y j1 ∂y j2 ∂ 2 y ik ∂y jr
α^j1 j2 ···jr ( · · · · · · ).
∂xi1 ∂xi2 ∂xjk ∂xj ∂xir
∑n ∂ 2 y ik
而 ik ,j=1 ∂xjk ∂xj dx
j
∧ dxik = 0. 于是

∂y j1 ∂y j2 ∂ 2 y ik ∂y jr
α^
j1 j2 ···jr ( · · · · · · )dxj ∧ (dxi1 ∧ dxi2 ∧ · · · ∧ dxir ) = 0.
∂xi1 ∂xi2 ∂xjk ∂xj ∂xir

140
CHAPTER 7. 流形上的微积分 7.2. 外微分与外微分式

进一步

∂(αi1 i2 ···ir ) j ∂y j1 ∂y j2 ∂y jr ∂ α^
j1 j2 ···jr
j
dx ∧ (dxi1 ∧ dxi2 ∧ · · · ∧ dxir ) = i i
· · · ir
∂x ∂x ∂x1 2 ∂x ∂xj
dxj ∧ (dxi1 ∧ dxi2 ∧ · · · ∧ dxir ).

这说明
1
d(α |U ) |U ∩V = d(αi1 i2 ···ir ) ∧ (dxi1 ∧ dxi2 ∧ · · · ∧ dxir )
r!
1 ∂(αi1 i2 ···ir ) j
= dx ∧ (dxi1 ∧ dxi2 ∧ · · · ∧ dxir )
r! ∂xj
1 ∂y j1 ∂y j2 ∂y jr ∂ α^j1 j2 ···jr
= i i
· · · ir dxj ∧ (dxi1 ∧ dxi2 ∧ · · · ∧ dxir )
r! ∂x ∂x1 2 ∂x ∂xj
1 ∂y j1 ∂y j2 ∂y jr
= · · · d(α^j1 j2 ···jr ) ∧ (dx ∧ dx ∧ · · · ∧ dx )
i1 i2 ir
r! ∂xi1 ∂xi2 ∂xir
1 ∂y j1 ∂y j2 ∂y jr
= d(α^ j1 j2 ···jr ) ∧ i i
· · · ir (dxi1 ∧ dxi2 ∧ · · · ∧ dxir )
r! ∂x ∂x 1 2 ∂x
1
= d(α^ j1 j2 ···jr ) ∧ (dy ∧ dy ∧ · · · ∧ dy )
i1 i2 ir
r!
= d(α |V ) |U ∩V .

于是 d 是整体定义的. 这就说明了 d 的存在性.

注解 7.3. 这个证明和其他的证明有那么一点点的不一样, 这个是先证明唯一性,


在证明存在性. 证明的关键在于张量的坐标变换公式和二阶导有关的外积求和为
零. 上述的证明思想和一阶微分的形式不变性是一样的. 回顾一阶形式不变性的
∂f ∂f
内容, 以二元函数为例, 设 f 是单连通区域 D 上的可微函数,df = ∂x dx + ∂y dy,
∂(u,v)
在非退化坐标变换 ∂(x,y) 下, ∂f
∂x =
∂f
∂u
∂x + ∂v
∂u
∂f ∂v ∂f ∂f ∂u
∂x , ∂y = ∂u ∂y + ∂f ∂v
∂v ∂y , 带入
∂f ∂f ∂f ∂f ∂f ∂u
df 中 得 到 df = ∂x dx + ∂y dy =
∂u
( ∂u
∂x + ∂v
∂v
∂x )dx + ( ∂u ∂y + ∂f ∂v
∂v ∂y )dy =
∂f ∂u ∂u ∂f ∂v ∂v ∂f ∂f
∂u ( ∂x dx + ∂y dy) + ∂v ( ∂x dx + ∂y dy) = ∂u du + ∂v dv. 这便是形式不变性, 上面
的证明虽然做法不一样但思想是一样的. 这样子做才能说明 d 的定义是整体的
和合理的.

下面的两个例子在一定程度上说明了什么是外微分算子 d 的 kernal(核空
间), 以增进我们对外微分算子 d 的认识.

例 7.4. 对任意的 f, g ∈ C ∞ (M ), 考虑一阶微分 df = ∂f i


∂xi dx , dg = ∂g j
∂xj dx . 考虑
如下的行列式
∂f ∂g
∂(f, g) i ∂xi
= ∂x ∂g
∂(x , x ) ∂fj
i j
∂x ∂xj

141
7.2. 外微分与外微分式 CHAPTER 7. 流形上的微积分

则有如下恒等式:
∂ ∂(f, g) ∂ ∂(f, g) ∂ ∂(f, g)
+ + = 0.
∂xk ∂(xi , xj ) ∂xi ∂(xj , xk ) ∂xj ∂(xk , xi )

证明. 考虑 α = df ∧ dg ∈ A2 (M ), 显然 d(α) = d(df ∧ dg) = d(df ) ∧ dg − df ∧


1 ∂(f,g)
d(dg) = 0 − 0 = 0. 一方面 α = df ∧ dg = ∂f ∂g
∂xi ∂xj dx
i
∧ dxj = 2 ∂(xi ,xj ) dx
i
∧ dxj ,
∂(f,g)
于是 d(α) = 1 ∂
2 ∂xk ∂(xi ,xj ) dx
k
∧ dxi ∧ dxj , 作用在相应的标准切向量场上结合
dα = 0 即可得到此结论 (在下面的例子中我们将看见更加一般的证明).

下面的例子上上述例子的推广, 目的在于刻画 dα = 0.

例 7.5. 设 (U, xi ) 是流形 M n 的一局部坐标系,α ∈ Ar (M ) 是 r 次外微分式, 在


局部坐标系下有
1
α |U = αi i ···i dxi1 ∧ dxi2 ∧ · · · ∧ dxir .
r! 1 2 r
∑r+1 ∂αi
1 i2 ···ik ir+1
ˆ
证明:dα |U = 0, 有 k=1 (−1)
k+1
∂xik
= 0.

证明. 我们知道
1
α |U = αi i ···i dxi1 ∧ dxi2 ∧ · · · ∧ dxir
r! 1 2 r

= αi1 i2 ···ir dxi1 ∧ dxi2 ∧ · · · ∧ dxir .
1≤i1 <i2 <···<ir ≤n

用 d 作用之有

d(α |U ) = d( αi1 i2 ···ir dxi1 ∧ dxi2 ∧ · · · ∧ dxir )
1≤i1 <i2 <···<ir ≤n

= d(αi1 i2 ···ir dxi1 ∧ dxi2 ∧ · · · ∧ dxir )
1≤i1 <i2 <···<ir ≤n

n ∑ ∂(αi1 i2 ···ir ) j
= dx ∧ (dxi1 ∧ dxi2 ∧ · · · ∧ dxir )
j=1 1≤i1 <i2 <···<ir ≤n
∂xj
∑n ∑ ∂(αi1 i2 ···ir ) j
= dx ∧ (dxi1 ∧ dxi2 ∧ · · · ∧ dxir )
∂xj
j=1,j̸=i1 ,i2 ,··· ,ir 1≤i1 <i2 <···<ir ≤n
∑n ∑ ∂(αi1 i2 ···iˆk ···ir ) ik
= dx ∧ dxi1 ∧ dxi2 · · · ∧ dxˆik · · · ∧ dxir+1
∂xik
ik ̸=i1 ,i2 ,··· ,ir+1 i1 <i2 ···<ik−1 <ik+1 ···<ir+1


r+1 ∑ ∂(αi1 i2 ···iˆk ···ir ) i1
= (−1)k+1 dx ∧ dxi2 · · · ∧ dxik · · · ∧ dxir+1 .
∂xik
k=1 i1 <i2 ···<ik−1 <ik <ik+1 ···<ir+1

142
CHAPTER 7. 流形上的微积分 7.2. 外微分与外微分式

对上述等号成立做一些说明: 倒数第三个等式是因为任何两个相同的 dxl 做外


积 dxl ∧ dxl = 0, 所以 j 等于 i1 , i2 , · · · , ir 中的任何一个时都等于 0, 故直接要
求 j 与它们不相等. 倒数第二个式子是做了一个所谓替换的过程, 首先将 j 插
入到按照大小顺序排好的 i1 < i2 < · · · < ir 之中. 不妨设 ik−1 < j < ik , 于
是有大小关系式 i1 < i2 < · · · < ik−1 < j < ik < · · · < ir , 在此做变量替换
j1 = i1 , · · · , jk−1 = ik−1 , jk = j, jk+1 = ik , · · · , jr+1 = ir+1 , 为了使得符号和之
前相同我们把所有的 jl 再次改写成 il , 即令 i1 = j1 , · · · , ir+1 = jr+1 . 再次把符
号带回原等式即可, 让 dxk 移动到 dxk−1 到 dxik+1 之间, 由于每一次对换会产
生一个负号, 一共需要进行 k − 1 次对换, 习惯上我们写成 (−1)k+1 . 之所以最后
一个等式前面写了个 k 从 1 到 r + 1 求和是因为 j 在插入到 i1 , i2 , · · · , ik 之间
时, 一共有 k + 1 种情况 (这是排列组合的知识). 由 d(α |U ) = 0. 我们得到如下
式子


r+1 ∑ ∂(αi1 i2 ···iˆk ···ir ) i1
(−1)k+1 dx ∧ dxi2 · · · ∧ dxir+1 = 0.
∂xik
k=1 i1 <i2 ···<ik−1 <ik <ik+1 ···<ir+1

由于在上述限制条件下 dxi1 ∧ dxi2 · · · ∧ dxir+1 是一组基中的元素, 必然是线性


∑r+1 ∂(α
2 ···ik ···ir
ˆ )
无关的, 于是只能有 k=1 (−1)k+1 i1 i∂x ik = 0.
特别地当 r = 2, 即 α |U = 1
2! aij dx
i
∧ dxj , 按照本例的结论有恒等式

∂ajk ∂aik ∂aij


− + = 0.
∂xi ∂xj ∂xk
当 r = 3 时有,α |U = 1
3! aijk dx
i
∧ dxj ∧ dxk , 按照本例的结论, 有恒等式

∂ajkl ∂aikl ∂aijl ∂aijk


− + − = 0.
∂xi ∂xj ∂xk ∂xl

下面给出所谓的微分公式, 我们取任意的一个一次外形式 ω ∈ A1 (M ), 显然
有 dω ∈ A2 (M ), 我们自然会问, 如果给定任意两个切向量场 X, Y , 那么 dω 作
用于 dω(X, Y ) 上得到的结果会是什么呢? 这就是我们的微分公式所要回答变得
问题.

命题 7.1. 设 ω ∈ A1 (M ), 则 dω ∈ A2 (M ), 对任意的光滑向量场 X, Y ∈ X(M ),


有微分公式 dω = X(ω(Y )) − Y (ω(X)) − w([X, Y ]) 成立.

证明. 注意到对任意的 X, Y ∈ X(M ), 在局部坐标系下均有 X = ai ∂x



i,Y =

∂ i ∂ j ∂ ∂ ∂
bj ∂x j , 由张量的函数线性性知道 dω(X, Y ) = dω(a ∂xi , b ∂xj ) = a b dω( ∂xi , ∂xj ),
i j

143
7.2. 外微分与外微分式 CHAPTER 7. 流形上的微积分

另一方面, 考虑等式的右边
[ ]
∂ j ∂ j ∂ i ∂ i ∂ j ∂
X(ω(Y )) − Y (ω(X)) − ω([X, Y ]) = ai (ω(b )) − b (ω(a )) − ω( a , b )
∂xi ∂xj ∂xj ∂xi ∂xi ∂xj
∂ ∂ ∂ ∂
=ai i (bj ω( j )) − bj j (ai ω( i ))−
∂x ∂x ∂x ∂x
[ ]
j i
i ∂b ∂ j ∂a ∂ ∂ ∂
ω(a −b −a b
i j
, )
∂xi ∂xj ∂xj ∂xi ∂xi ∂xj

这里用到了 ω 的函数线性性, 进一步化简

右边 = X(ω(Y )) − Y (ω(X)) − ω([X, Y ])


∂ ∂ ∂ ∂bj j ∂a
i

=ai bj i
(ω( j
)) + ai
ω( j
) i
− b j
ω( i )−
∂x ∂x ∂x ∂x ∂y ∂x
j i
∂ ∂ ∂b ∂ ∂a ∂
ai bj j (ω( i )) − ai i ω( j ) + bj j ω( i )
∂x ∂x ∂x ∂x ∂x ∂x
i j ∂ ∂ i j ∂ ∂
=ab (ω( j )) − a b (ω( i ))
∂xi ∂x ∂xj ∂x

∂xi (ω( ∂xj )) − ∂xj (ω( ∂xi )) 即可. 不


∂ j ∂ ∂ ∂ ∂ ∂
这说明我们只需验证 dω(ai ∂x i , b ∂xj ) =

妨设 ω = ωi dxi , 则 dω = dω ∧ dxi = ∂x ∂ωi


j dx ∧ dx = ∂xl dx ∧ dx , 考虑 dω
j i ∂ωk l k

i , ∂xj ) 于是有 dω( ∂xi , ∂xj ) = ∂xl dx ∧ dx ( ∂xi , ∂xj ) =


∂ ∂ ∂ ∂ ∂ωk k ∂ ∂
作用于基向量场 ( ∂x l

δl δk
i ∂ωj
∂ωk i
∂xl l = ∂ω k
(δil δjk − δik δjl ) = − ∂ωi
∂xj . 另一方面 ω = ωi dxi = ∂ω
=
δj δjk ∂xl ∂xi ∂xi

∂ωj

ω( ∂x i )dx , 于是
i ∂ ∂
∂xi (ω( ∂xj )) − ∂ ∂
∂xj (ω( ∂xi )) = ∂xi − ∂ωi
∂xj . 这就证明了微分公

下面我们介绍更一般的微分公式, 但证明略去, 这就是下面的命题.

命题 7.2. 设 M n 为 n 为光滑流形,ω ∈ Ar 为 r 次微分形式,dω ∈ Ar+1 , 则对任


意的 X1 , X2 , · · · , Xr+1 , 有微分公式


r+1
dω(X1 , X2 , · · · , Xr+1 ) = (−1)a+1 Xa (ω(X1 , X2 , · · · , X̂a , · · · , Xr+1 ))+
a=1

(−1)a+b ω([Xa , Xb ] , X1 , · · · , X̂a , · · · , X̂b , · · · , Xr+1 )
a<b

注解 7.4. 事实上, 上述公式也可以作为外微分算子的定义. 然后利用该定义去


证明我们在前面的外微分算子具有的那四条性质, 这也是一种方式. 接下来我们
利用外微分来重新描述 F robenius 定理.
首先回顾 F robenius 定理的内容: 设 M n 是 n 维光滑流形,M 上的线性无
关的 k 个向量场 X1 , X2 , · · · , Xk ,Lk = span {X1 , X2 , · · · , Xk } 可积 ⇐⇒ 对任

144
CHAPTER 7. 流形上的微积分 7.2. 外微分与外微分式

意的 i, j, l ∈ {1, 2, · · · , k} 有 [Xi , Xj ] ≡ 0 mod Xl . 现在我们将 X1 , X2 , · · · , Xk


扩充为 n 个线性无关的基 X1 , X2 , · · · , Xk , Xk+1 , · · · , Xn , 设 Xi 的对偶余切场
(一次微分形式) 为 ω 1 , ω 2 , · · · , ω k , ω k+1 , · · · , ω n . 显然我们有

ω r+1 (Xi ) = 0, ω r+2 (Xi ) = 0, · · · , ω n (Xi ) = 0, ∀ i ∈ {1, 2, · · · , k} .

于是我们可以将 Lk 改写, 注意到 Lk 是有 Xi 所张成的, 也就是说明 Lk 是有


那些使得 ω λ (X) = 0, 对任意的 λ ∈ {k + 1, k + 2, · · · , n} 的 X 所张成的, 即
{ }
Lk = span X | ω λ (X) = 0, ∀ λ ∈ {k + 1, k + 2, · · · , n} . 以此为出发点, 我们
得到如下的定理, 称之为 F robenius 的对偶形式.

定理 7.3. 设 Lk 为 V ⊆ M n 是 V 上的光滑分布,X1 , X2 , · · · , Xk 是 V 上线性无


关的 k 个光滑向量场, 则对任意的 p ∈ V, 存在 p 的坐标邻域 (U, ui ), 1 ≤
{ ∂ }
i ≤ n, 使 Lk |U = span ∂u 1 , ∂u2 , · · · , ∂uk
∂ ∂
(即 Lk 在 U 上 完 全 可 积)⇐⇒
[Xi , Xj ] ≡ 0 (mod Xl ) ⇐⇒(将 X1 , X2 , · · · , Xk 扩 充 为 n 个 线 性 无 关 的 基
X1 , X2 , · · · , Xk , Xk+1 , · · · , Xn , 设 Xi 的对偶余切场 (对偶一次微分形式) 为
ω 1 , ω 2 , · · · , ω k , ω k+1 , · · · , ω n .) 若 存 在 一 次 外 微 分 式 φλµ 使 得 dω λ = φλj ∧
ω j , ∀ λ ∈ {k + 1, k + 2, · · · , n} , j ∈ {k + 1, k + 2, · · · , n} .

证明. 首先由微分公式可得

dω λ (Xi , Xj ) = Xi (ω λ (Xj )) − Xj (ω λ (Xi )) − ω λ ([Xi , Xj ])


= −ω λ ([Xi , Xj ])

于是 [Xi , Xj ] ≡ 0 (mod Xl ) ⇐⇒ ω λ ([Xi , Xj ]) = 0. 注意到 dω λ ∈ A2 (W ), 于


是 dω 可被 ω i ∧ ω j 线性表出, 并且 ω i ∧ ω j 前的系数恰好是 dω λ (Xi , Xj )(其中

145
7.2. 外微分与外微分式 CHAPTER 7. 流形上的微积分

1 ≤ i, j ≤ n), 我们有

dω λ = dω λ (Xi , Xj )ω i ∧ ω j

n
= dω λ (Xi , Xj )ω i ∧ ω j
i,j=1
∑ r ∑
r ∑
n
= dω λ (Xi , Xj )ω i ∧ ω j + dω λ (Xi , Xj )ω i ∧ ω j +
i,j=1 i=1 j=r+1
∑n ∑
r ∑n
dω λ (Xi , Xj )ω i ∧ ω +j
dω λ (Xi , Xj )ω i ∧ ω j
i=r+1 j=1 i,j=r+1
∑r ∑
r ∑
n
= dω (Xi , Xj )ω i ∧ ω j +
λ
dω λ (Xi , Xj )ω i ∧ ω j +
i,j=1 i=1 j=r+1
∑n ∑
r ∑n
dω λ (Xj , Xi )ω j ∧ ω +i
dω λ (Xi , Xj )ω i ∧ ω j
j=r+1 i=1 i,j=r+1
∑r ∑
r ∑
n
= dω (Xi , Xj )ω i ∧ ω j +
λ
dω λ (Xi , Xj )ω i ∧ ω j +
i,j=1 i=1 j=r+1
∑n ∑
r ∑n
dω λ (Xi , Xj )ω i ∧ ω +j
dω λ (Xi , Xj )ω i ∧ ω j
j=r+1 i=1 i,j=r+1

进一步我们有
dω λ (Xi , Xj ) = dω λ (Xi , Xj )ω i ∧ ω j


r ∑
r ∑
n
= dω λ (Xi , Xj )ω i ∧ ω j + 2 dω λ (Xi , Xj )ω i ∧ ω j +
i,j=1 i=1 j=r+1
∑n
dω λ (Xi , Xj )ω i ∧ ω j
i,j=r+1
∑ r ∑
n ∑
r ∑
n
= dω λ (Xi , Xj )ω i ∧ ω j + (2 dω λ (Xi , Xj )ω i + dω λ (Xi , Xj )ω i ) ∧ ω j
i,j=1 j=r+1 i=1 i=r+1
∑r ∑n
令 φλj = 2 i=1 dω λ (Xi , Xj )ω i + i=r+1 dω λ (Xi , Xj )ω i , 注意到完全可积 ⇐⇒
dω λ ([Xi , Xj ]) = 0 ⇐⇒ dω λ = φλj ∧ ω , λ, j ∈ {k + 1, k + 2, · · · , n} .
j

例 7.6. 求下列 P af f 方程可积的充分必要条件.


P (x, y, z)dx + Q(x, y, z)dy + R(x, y, z)dz = 0 完全可积的条件?

证明. 令 Ω = P (x, y, z)dx + Q(x, y, z)dy + R(x, y, z)dz = P dx + Qdy + Rdz, 当


Ω = 0 完全可积时, 有 F robenius 定理的对偶形式, 我们有 ⇐⇒ dΩ = φ ∧ Ω, 由

146
CHAPTER 7. 流形上的微积分 7.3. 外微分形式的积分

分解定理我们知道它等价于 Ω ∧ dΩ = 0, 计算

dΩ = d(P dx + Qdy + Rdz)


= dP ∧ dx + dQ ∧ dy + dR ∧ dz
∂P ∂P ∂P
=( dx + dy + dz) ∧ dx+
∂x ∂y ∂z
∂Q ∂Q ∂Q
( dx + dy + dz) ∧ dy+
∂x ∂y ∂z
∂R ∂R ∂R
( dx + dy + dz) ∧ dz
∂x ∂y ∂z
∂P ∂P
= dy ∧ dx + dz ∧ dx+
∂y ∂z
∂Q ∂Q
dx ∧ dy + dz ∧ dy+
∂x ∂z
∂R ∂R
dx ∧ dz + dy ∧ dz
∂x ∂y
∂Q ∂P ∂R ∂Q ∂P ∂R
=( − )dx ∧ dy + ( dy − )dy ∧ dz + ( − )dz ∧ dx
∂x ∂y ∂y ∂z ∂z ∂x

带入 Ω 的表达式有
∂R ∂Q ∂P ∂R ∂Q ∂P
P( dy − ) + Q( − ) + R( − ) = 0.
∂y ∂z ∂z ∂x ∂x ∂y

这便是方程有解的条件.

注解 7.5. 事实上,F robenius 定理的对偶形式比 F robenius 定理有着更广泛的


应用, 原因之一是微分几何中外微分是一种非常基本的运算, 并且这种形式可以
联系 Cartan 引理, 在处理微分方程的解的时候有着出奇的效果, 下节课我们便
会介绍相关的应用和证明.

7.3 外微分形式的积分
2019-12-25
本节要点/问题提示:
1)、一阶 n 元齐次线性全微分方程有解的条件是什么?
2)、一阶线性其次微分方程组有解的条件是什么?
3)、什么是恰当形式? 什么是闭形式?
4)、恰当形式一定是闭形式吗? 闭形式一定是恰当形式吗?
6)、在什么条件下它们两者可以互推?P oincare 引理的内容是什么?

147
7.3. 外微分形式的积分 CHAPTER 7. 流形上的微积分

7)、单位分解定理的内容? 流形可定向的定义?
8)、流形上的外微分形式的积分的定义?
9)、Stokes 定理的内容? 如何用新的观点看待数学分析中的那些积分公式?
回顾: 微分流形这门课, 流形上的微积分是基础内容, 流形上的三大基本建
筑块: 流形的微积分、流形的上同调、流形的椭圆算子. 这学期前面三章相当于
一直在介绍基础性概念, 并没有深入到流形论的核心内容, 但由于学时的限制,
我们做了一些取舍, 其实无论是微分流形也好, 其他课程也好, 很多东西我们都
只能先夯实自己的基础, 把基本的东西理解透彻了, 后面的东西就简单得多. 研
究生的学习课堂最多只能占你们学习总量的三分之一, 无外乎说老师的带领下
你们对很多东西的理解会更深刻, 师傅领进门, 修行靠个人.
下面的例子是上节课最后一个例子的 n 元形式.

例 7.7. 一阶 n 元齐次线性全微分方程
给 出 a1 dx1 + a2 dx2 + · · · + an dxn = 0 方 程 有 解 的 条 件. 其 中 ai ∈
C ∞ (U ), U ∈ Rn (U 是 Rn 中的开集).

证明. 上述方程有解意味着存在 F (x1 , x2 , · · · , xn ) = C 为方程的解. 由上节课


的 F robenius 定理的对偶形式有, 令 ω = a1 dx1 + a2 dx2 + · · · + an dxn , 于是

⇐⇒ ∃ λ ̸= 0, s.t. ω = λdF

⇐⇒ ∃ φ s.t. dω = φ ∧ ω, 其中φ为一次微分形式.

⇐⇒ ω ∧ dω = 0.

(最后一个当且仅当的推导过程中用到了外代数中的分解定理, 在本笔记第四章
∂aj
的最后一个习题) 计算 dω = d(ai dxi ) = dai ∧dxi = ∂xj dx ∧dx
∂ai j i
= ∂xk
dxk ∧dxj .
于是有

⇐⇒ ω ∧ dω = 0.

∂aj i
⇐⇒ ai dx ∧ dxk ∧ dxj = 0.
∂xk

∂aj i
⇐⇒ ai dx ∧ dxj ∧ dxk = 0.
∂xk
∂a
令 aijk = ai ∂xkj , 于是我们 得到 aijk dxi ∧ dxj ∧ dxk = 0. 作 用 在 一 组 基 上

148
CHAPTER 7. 流形上的微积分 7.3. 外微分形式的积分

∂ ∂ ∂
( ∂x l , ∂xm , ∂xn ) 上有

∂ ∂ ∂
0 = aijk dxi ∧ dxj ∧ dxk ( l , m , n )
∂x ∂x ∂x
i j
δl δl δlk

= aijk δmi j
δm k
δm
i
δn δnj δnk

= aijk (δli δm
j k
δn δl + δni δlj δm
i j k
δn + δm k
− δni δm δl − δli δnj δm
j k k i j k
− δm δl δn )
= almn + amnl + anlm − anml − alnm − amln
= aijk + ajki + akij − akji − aikj − ajik .

于是
∂aj ∂ak ∂ak ∂ai ∂ai ∂aj
ai ( k
− j
) + aj ( i − k ) + ak] ( j − ) = 0.
∂x ∂x ∂x ∂x ∂x ∂xi

上述一个方程的情形, 下面我们考虑方程组的情形, 为简单起见, 只考虑两


个方程组的情况, 其他的类似.

例 7.8. 一阶四元其次线性微分方程组.
{
a1 dx1 + a2 dx2 + a3 dx3 + a4 dx4 = 0.
b1 dx1 + b2 dx2 + b3 dx3 + b4 dx4 = 0.

其中 ai 和 bi 均是 U 上的一次可导的函数,U 是 R4 中的开集. 且 (a1 , a2 , a3 , a4 )


与 (b1 , b2 , b3 , b4 )

证明. 令 Ω1 = a1 dx1 +a2 dx2 +a3 dx3 +a4 dx4 , Ω2 = b1 dx1 +b2 dx2 +b3 dx3 +b4 dx4
由 F robenius 定理的对偶形式, 我们得到
{
dΩ1 = φ11 ∧ Ω1 + φ21 ∧ Ω1 .
dΩ2 = φ12 ∧ Ω1 + φ22 ∧ Ω1 .

根据外代数的分解定理, 我们有
{
Ω1 ∧ Ω2 ∧ dΩ1 = 0.
Ω1 ∧ Ω2 ∧ dΩ2 = 0.

∂aj
注意到 dΩ1 = ∂ai
∂xj dx
j
∧ dxi = 1 ∂ai
2 ( ∂xj − ∂xi )dx
j
∧ dxi = 1 ∂ak
2 ( ∂xl − ∂al
∂xk
)dxl ∧
dxk , Ω1 ∧ Ω2 = (ai dxi ) ∧ (bj dxj ) = ai bj dx ∧ dx =
i j 1
2 (ai bj − aj bi )dx ∧ dx =
i j

149
7.3. 外微分形式的积分 CHAPTER 7. 流形上的微积分


a aj i
i
1
dx ∧ dxj . 其中 1 ≤ i, j, k, l ≤ 4 于是
2 bi bj

0 = Ω1 ∧ Ω2 ∧ dΩ1
= Ω1 ∧ Ω2 ∧ (−dΩ1 )

1 ∂ak ∂al ai aj i
= ( l − ) dx ∧ dxj ∧ dxl ∧ dxk
4 ∂x ∂xk bi bj

1 ∂ak ∂al ai aj i
= ( ( l − k) dx ∧ dxj ∧ dxk ∧ dxl
4 ∂x ∂x bi bj

1 ∂a3 ∂a4 a1 a2 1 ∂a4 ∂a3 a1 a2
= ( 4 − 3) − ( − 4)
4 ∂x ∂x b1 b2 4 ∂x3 ∂x b1 b2

1 ∂a3 ∂a4 a2 a1 1 ∂a4 ∂a3 a2 a1
− ( 4− ) + ( − 4)
4 ∂x ∂x3 b2 b1 4 ∂x3 ∂x b2 b1

1 ∂a1 ∂a2 a3 a4 1 ∂a2 ∂a1 a3 a4
+ ( 2− ) − ( − 2)
4 ∂x ∂x1 b3 b4 4 ∂x1 ∂x b3 b4

1 ∂a1 ∂a2 a4 a3 1 ∂a2 ∂a1 a4 a3
− ( 2− ) + ( − 2) )
4 ∂x ∂x1 b4 b3 4 ∂x1 ∂x b4 b3

dx1 ∧ dx2 ∧ dx3 ∧ dx4

于是有


1 ∂a3 ∂a4 a1 a2 1 ∂a4 ∂a3 a1 a2
( − ) − ( − 4)
4 ∂x4 ∂x3 b1 b2 4 ∂x3 ∂x b1 b2

1 ∂a3 ∂a4 a2 a1 1 ∂a4 ∂a3 a2 a1
− ( 4− ) + ( − )
4 ∂x ∂x3 b2 b1 4 ∂x3 ∂x4 b2 b1

1 ∂a1 ∂a2 a3 a4 1 ∂a2 ∂a1 a3 a4
+ ( 2− ) − ( − )
4 ∂x ∂x1 b3 b4 4 ∂x1 ∂x2 b3 b4

1 ∂a1 ∂a2 a4 a3 1 ∂a2 ∂a1 a4 a3
− ( 2− ) + ( − ) ) = 0.
4 ∂x ∂x1 b4 b3 4 ∂x1 ∂x2 b4 b3

150
CHAPTER 7. 流形上的微积分 7.3. 外微分形式的积分

同理去计算 Ω1 ∧ Ω2 ∧ dΩ2 可得到相似的结论:


1 ∂b3 ∂b4 a1 a2 1 ∂b4 ∂b3 a1 a2
( − 3) − ( − 4)
4 ∂x4 ∂x b1 b2 4 ∂x3 ∂x b1 b2

1 ∂b3 ∂b4 a2 a1 1 ∂b4 ∂b3 a2 a1
− ( 4− ) + ( − 4)
4 ∂x ∂x3 b2 b1 4 ∂x3 ∂x b2 b1

1 ∂b1 ∂b2 a3 a4 1 ∂b2 ∂b1 a3 a4
+ ( 2− ) − ( − 2)
4 ∂x ∂x1 b3 b4 4 ∂x1 ∂x b3 b4

1 ∂b1 ∂b2 a4 a3 1 ∂b2 ∂b1 a4 a3
− ( 2− ) + ( − 2) ) = 0.
4 ∂x ∂x1 b4 b3 4 ∂x1 ∂x b4 b3

下面我们介绍一些比较特别的微分形式, 这主要是为了把之前微积分里面
关于函数的概念自然而然的推广到微分形式上, 分别是普通微分常值函数的外
微分算子的外微分式推广和普通微分原函数的外微分式推广.

定义 7.6. 设 α ∈ Ar (M ), 若 dα = 0, 则称 α 为闭形式, 若存在 β ∈ Ar−1 (M ),


使得 α = dβ, 则 α 称之为恰当形式.

由于 d(dα) = 0, 于是任何恰当形式都是闭的, 当闭的是否一定是恰当的呢?


我们有下面的例子.

例 7.9. 考虑 U = R2 /(0, 0) 上的一次外微分式

ydx − xdy
α=
x2 + y 2

首先说明 α 是闭形式, 注意到

α(x2 + y 2 ) = ydx − xdy

151
7.3. 外微分形式的积分 CHAPTER 7. 流形上的微积分

同时外微分有

d(α(x2 + y 2 )) = d(ydx − xdy)


= dy ∧ dx − dx ∧ dy
= −2dx ∧ dy
= (x2 + y 2 )dα + (−1)1 αd(x2 + y 2 )
= (x2 + y 2 )dα + α(2xdx + 2ydy)
ydx − xdy
= (x2 + y 2 )dα − ∧ (2xdx + 2ydy)
x2 + y 2
2y 2 dx ∧ dy − 2x2 dy ∧ dx
= (x2 + y 2 )dα −
x2 + y 2
2y dx ∧ dy + 2x2 dx ∧ dy
2
= (x2 + y 2 )dα −
x2 + y 2
= (x2 + y 2 )dα − 2dx ∧ dy

这说明 dα = 0, 即 α 是闭形式. 咋看之下, 似乎 arctan xy 就是这个微分形式


的原函数, 但这个函数最大的问题在于定义域不是我们所要求的定义域, 因为
y = 0, x ∈ R 是上述函数是没有定义的, 这说明该函数不是我们所需要的原函数,
注意定义域这里, 这是容易忽视的一个点. 我们下面说明 α 不是恰当形式, 即不
存在这样的函数使得 f , 使得 df = α, 其中 f 是 U 上的可导函数. 用反证法, 假
设存在这样的函数 f , 考虑单位圆周 S 1 = {(x = sinθ, y = cosθ) | 0 ≤ θ < 2π} ,
注意到 ∫
f (2π) − f (0) = df
∫S
1

= α
S1
∫ 2π
sinθdcosθ − cosθdsinθ
=
0 sin2 θ + cos2 θ
∫ 2π
= −dθ
0

= −2π

而圆周是闭曲线, 故 f (2π) = f (0), 这与上式矛盾.

到底什么情况下我们能够断言闭形式一定是恰当的呢? 受到上述例子的启
发, 这个条件应该与定义域的某些特征有关, 当定义域满足某些特征的时候, 我
们便可以断言闭形式是恰当的. 这就是著名的 P oincare 引理, 由于时间关系
P oincare 引理就不在这里证明了, 我们引入下面的定义来刻画定义域的特征.

152
CHAPTER 7. 流形上的微积分 7.3. 外微分形式的积分

定义 7.7. 设 U 为 Rn 的开集, 如果对任意的 x1 , x2 ∈ U , 线段 x1 x2 ∈ U, 则称


U 是 Rn 中的星形区域.

引理 7.2. 若 U 是 Rn 中的星形区域, 则对任意的闭的 r 次外形式 α ∈ Ar (U ),


存在 β ∈ Ar−1 (U ), 使得 α = dβ, 即星形区域上的闭形式一定是恰当的.

注解 7.6. 这就更为本质的揭示了为什么上述例子中的一次微分闭形式不是恰
当的, 因为 R2 /(0, 0) 不是星形区域. 那么对于一般的区域,P oincare 引理也能在
某种意义上刻画区域的一些拓扑性质, 实际上我们可以借助代数拓扑的手段对
流形的拓扑性质做一些刻画. 首先我们可以构造一个 (Ar (M ), d) 链复形, 然后
通过这个链复形上的外微分运算的核空间模掉像空间就得到了同调群. 这个同
调群同构于实系数非约化的第 r 个单纯上同调群 (群的维数等于第 k 个 Betti
数). 这就是 de Rham 定理. 它联系了微分结构与拓扑结构, 用微分结构来构造
出的一个上同调群, 但是这个上同调群居然同构于一个与微分结构没有关系的
单纯上同调群.

下面我们要开始定义积分了, 我们终于可以将局部的东西作成整体的东西
了. 对微分几何而言, 局部到整体的过程就是微分几何的发展历程, 当然更多的
时候流形是离不开拓扑的, 所以在流形的上同调也非常重要. 下面的定理是局部
到整体的一个非常重要的定理, 在某种程度上相当于是数学分析中的有限覆盖
定理的推广. 但证明由于时间关系从略.

定理 7.4. 单位分解定理: 设 M n 为 n 维光滑流形, 则 M n 上存在局部有限 (任


意一个点都存在该点的一个邻域只与有限多个覆盖中的子集相交) 的坐标覆盖
{ }
(Uα , xiα ) | α ∈ A , 以及相应的光滑函数族 fα 满足:
1)、suppfα = {x ∈ M | fα (x) ̸= 0.} 紧致
2)、0 ≤ fα ≤ 1
3)、suppfα ⊆ Uα

4)、 α fα ≡ 1.

定义 7.8. 设 M 为 n 维流形, 如果存在坐标覆盖 {(Uα )} 使得对任意的 Uα ∩Uβ ̸=


∂(x1β ,x2β ,··· ,xn
β)
∅, 有 ∂(x1α ,x2α ,··· ,xn > 0. 则称 M 是可定向的.
α)

特别地, 欧式空间是可定向的, 且恰好有两个定向, 更一般的结论有: 任何连


通可定向流形恰好有两个定向. 有了上述的工具, 我们就可以定义积分了.

定义 7.9. 设 M n 为可定向流形,∂M 为该定向的诱导定向.ω ∈ An (M n ), ω |Uα =


aα dx1α ∧ · · · ∧ dxnα , aα ∈ C ∞ (U ), 定义 ω 在 Uα 上的外微分形式的积分如下:
∫ ∫
ω= aα ◦ φ−1
α dxα dxα · · · dxα .
1 2 n
Uα φα

153
7.3. 外微分形式的积分 CHAPTER 7. 流形上的微积分


注意到 ω = ω · 1 = α fα ω, 定义 ω 在 M n 上外微分形式的积分如下:
∫ ∑∫ ∑∫
w := (fα ω) = (fα aα ) ◦ φ−1
α dxα dxα · · · dxα .
1 2 n
Mn α Uα α φα (Uα )

有了上述的积分的定义, 我们现在就要来推广微积分中的 N ewton—Leibniz


公式了, 这就是著名的 Stokes 定理. 由于时间关系证明从略.

定理 7.5. 设 ω ∈ An−1 (M ), D 为可定向光滑流形 M n 中的区域,ω 是 M n 具有


紧致支撑集 (使得 ω ̸= 0 的那些点的闭包是紧致的) 的 n − 1 微分式, 则
∫ ∫
dω = w.
D ∂D

∫ ∫ ∫ ∫
特别地, 若 D = M n , 有 M
dω = ∂M
w, 若 ∂D = ∅, 则 D
dω = ∂D
w = 0.

例 7.10. 设 f ∈ C([a, b]), F (x) 是 f (x) 的原函数, 记 ∂ [a, b] = {b} − {a} . 则有

∫ b ∫ b ∫
f (x)dx = dF (x) = F (x) = F (b) − F (a).
a a ∂[a,b]

这说明 N ewton—Leibniz 是 Stokes 定理的特例.

例 7.11. 设平面单连通区域 D 是以光滑 L 为边界的, 函数 P, Q 是定义在 D 上


具有一阶连续偏导数的, 则有
∫ ∫
P dx + Qdy = P dx + Qdy
L
∫∂D
= d(P dx + Qdy)
∫D
∂P ∂P ∂Q ∂Q
= ( dx + dy) ∧ dx + dx + dy) ∧ dy
∂x ∂y ∂x ∂y
∫ D
∂Q ∂P
= ( − )dx ∧ dy
∂x ∂y
∫D
∂Q ∂P
= ( − )dxdy
D ∂x ∂y

这说明 Green 公式是 Stokes 定理的特例.

例 7.12. 设三维空间中的曲面 D 是以光滑 L 为边界的, 函数 P, Q, R 是定义在

154
CHAPTER 7. 流形上的微积分 7.3. 外微分形式的积分

D 上具有一阶连续偏导数的, 则有

∫ ∫
P dx + Qdy + Rdz = P dx + Qdy + Rdz
L
∫∂D
= d(P dx + Qdy + Rdz)
∫D
∂P ∂P ∂P
= ( dx + dy + dz) ∧ dx+
D ∂x ∂y ∂z
∂Q ∂Q ∂Q
( dx + dy + dz) ∧ dy+
∂x ∂y ∂z
∂R ∂R ∂R
( dx + dy + dz) ∧ dz
∂x ∂y ∂z

∂Q ∂P ∂R ∂Q ∂P ∂R
= ( − )dx ∧ dy + ( dy − )dy ∧ dz + ( − )dz ∧ dx
∂x ∂y ∂y ∂z ∂z ∂x
∫ D
∂Q ∂P ∂R ∂Q ∂P ∂R
= ( − )dxdy + ( dy − )dydz + ( − )dzdx
D ∂x ∂y ∂y ∂z ∂z ∂x

这说明 Stokes 公式是 Stokes 定理的特例.

例 7.13. 设三维空间中的有界区域 D 是以光滑曲面 ∂D 为边界的, 函数 P, Q, R


是定义在 D 上具有一阶连续偏导数的, 则有

∫ ∫
P dydz + Qdzdx + Rdxdy = P dydz + Qdzdx + Rdxdy
L
∫∂D
= P dy ∧ dz + Qdz ∧ dx + Rdx ∧ dy
∫∂D
= d(P dy ∧ dz + Qdz ∧ dx + Rdx ∧ dy)
∫D
∂P ∂Q ∂R
= dx ∧ dy ∧ dz + dy ∧ dz ∧ dx + dz ∧ dx ∧ dy
∂x ∂y ∂z
∫D
∂P ∂Q ∂R
= ( + + )dx ∧ dy ∧ dz
D ∂x ∂y ∂z

这说明 Gauss 公式是 Stokes 定理的特例.

例 7.14. 一般的, 若 D 是 Rn 中的 n 为子流形,∂D 为 D 的诱导定向且 ∂D 为

155
7.3. 外微分形式的积分 CHAPTER 7. 流形上的微积分

ˆ i · · · ∧ dxn , 我们有有
n − 1 维光滑子流形,ω = Pi dx1 ∧ dx2 · · · ∧ dx
∫ ∫
ˆ i · · · dxn =
Pi dx1 dx2 · · · dx ˆ i · · · ∧ dxn
Pi dx1 ∧ dx2 · · · ∧ dx
∂D
∫ ∂D

= P dy ∧ dz + Qdz ∧ dx + Rdx ∧ dy
∫∂D
= ˆ i · · · ∧ dxn )
d(Pi dx1 ∧ dx2 · · · ∧ dx
D
∫ ∑ n
∂Pi i ˆ i · · · ∧ dxn
= i
dx ∧ dx1 ∧ dx2 · · · ∧ dx
D i=1 ∂x
∫ ∑ n
∂Pi
= ( (−1)i+1 i )dx1 ∧ dx2 · · · ∧ dxi · · · ∧ dxn
D i=1 ∂x
∫ ∑ n
∂Pi
= ( (−1)i+1 i )dx1 dx2 · · · dxi · · · dxn
D i=1 ∂x

例 7.15. 设 D 为 Rn 中的区域, 在上例中, 特别的, 取 Pi = (−1)i+1 xi , 则有

∫ ∫ ∑
n
(−1) i+1 i 1 2 ˆ i · · · dxn =
x dx dx · · · dx ( (−1)i+1+i+1 dx1 dx2 · · · dxi · · · dxn
∂D D i=1
∫ ∑n
= ( (−1)i+1+i+1 dx1 dx2 · · · dxi · · · dxn
D i=1

=n dx1 dx2 · · · dxi · · · dxn
D

如果只取固定的一个 i 使得 Pi = (−1)i+1 xi 而其他的都为零. 则有上述积分



式的结果为 D dx1 dx2 · · · dxi · · · dxn , 该式子通常记为 vol(D), 由上述计算知

vol(D) = ∂D ω, 其中 ω 取上述微分形式.

习 题 7.2. 复 变 函 数 f = u + iv,(x, y) ∈ D, 其 中 D 为 R2 中 的 单 连 通 区
域,z = x + iy, 令 ω = f (z)dz, 证明下述结论:
1)、验证 f 解析 ⇐⇒ dω = 0.
∫ ∫
2)、若 f 解析, 则 D ω = D f (z)dz = 0.

证明. 1)、考虑微分形式 ω 的具体表达式

ω = f (z)dz = (u + iv)(dx + idy) = udx − vdy + i(udy + vdx)

156
CHAPTER 7. 流形上的微积分 7.3. 外微分形式的积分

进一步对其外微分有

dω = d(udx − vdy) + id(udy + vdx)


= du ∧ dx − dv ∧ dy + idu ∧ dy + idv ∧ dx
∂u ∂v ∂u ∂v
= dy ∧ dx − dx ∧ dy + i dx ∧ dy + i dy ∧ dx
∂y ∂x ∂x ∂y
∂u ∂v ∂u ∂v
= −( + )dx ∧ dy + i( − )dx ∧ dy1
∂y ∂x ∂x ∂y

于是有
∂u ∂v ∂u ∂v
dω = 0 ⇐⇒ + = 0, − =0
∂y ∂x ∂x ∂y
∂u ∂v ∂u ∂v
⇐⇒ =− , =
∂y ∂x ∂x ∂y
⇐⇒ f 满足Cauchy—Riemann方程

⇐⇒ f 是 D 上的解析函数

2)、若 f 解析, 则 dω = 0. 有 Stokes 定理


∫ ∫ ∫ ∫
f (z)dz = ω= dw = 0 = 0.
D D ∂D D

157
7.3. 外微分形式的积分 CHAPTER 7. 流形上的微积分

158
第8章 后记

本笔记到这里就要和大家说一声再见了, 这本笔记我从 2019 年 10 月 1 日


左右开始写, 中途由于上课考试只写到了 90 多页就放寒假了, 寒假在家期间漫
不经心的写了完了后面的内容 (参考了 [1–4]), 大约在 2020 年 2 月 9 日左右完
工. 实际上我这本笔记的内容只能算是对微分流形的入门而已, 我觉得比较遗憾
的是没有写到上同调理论, 老师授课的时候没怎么讲. 其实我在笔记中一直倡导
一种思维方式: 多角度的去看待我们所学的数学内容, 数学定义定理公式都应当
是自然的, 我想和大家一起探索前人走过的路, 学数学不应该是死记硬背, 很多
知识都是通过直观的例子得到的启发, 我个人比较喜欢这种学习方式, 天才有天
才的路要走, 我们这种普通人也有自己的学习方式, 如果可以, 请大家多读几遍
我的那两篇认知心理学和数学方法论的博客, 希望你们能在学习的道路上越走
越远.
如果说论几何学方面的学习, 微分流形是基础, 代数拓扑也要懂一些, 微分
拓扑也要懂一些, 黎曼曲面相当于复变的曲面微分几何学, 以及黎曼几何, 复流
形, 子流形几何, 共形微分几何等. 下学期我会陆陆续续完成黎曼几何和子流形
几何的笔记. 我们下学期再见. 最后, 附上一些我喜欢的经典名言.
非淡泊无以明志, 非宁静无以致远.(诸葛孔明)
为将之道, 当先冶心. 泰山崩于眼前而色不改, 麋鹿兴于左而目不瞬.(苏洵)
天行健, 君子以自强不息. 地势坤, 君子以厚德载物.(周易)
是非审之于己, 毁誉听之于人, 得失安之于数.(岳麓书院)
发上等愿, 结中等缘, 享下等福.(左宗棠)
男儿立志出乡关, 学不成名誓不还.(毛泽东)
不为圣贤, 便为禽兽. 但问耕耘, 莫问收获.(曾国藩)
古之立大事者, 不惟有超世之才, 亦必有坚忍不拔之志.(苏轼)
鹰立如睡, 虎行似病. 神光内敛, 韬光养晦. 地低成海, 人低成王.(菜根谭)
若我求佛, 佛去求谁. 佛若渡我, 谁来渡佛.
海纳百川, 有容乃大. 壁立于仞, 无欲则刚.(林则徐)

159
CHAPTER 8. 后记

160
参考文献

[1] 陈省身. 微分几何讲义. 科学出版社, 北京, 2001.

[2] 郭震. 拓扑与流形. 云南科技出版社, 昆明, 2002.

[3] 陈维桓. 微分几何引论. 高等教育出版社, 北京, 2013.

[4] 梅加强. 流形与几何初步. 科学出版社, 北京, 2013.

161

You might also like